OB Final Prep U questions

Lakukan tugas rumah & ujian kamu dengan baik sekarang menggunakan Quizwiz!

The nurse who is caring for a woman hospitalized for hyperemesis gravidarum would expect the initial treatment to involve what? a. Intravenous (IV) therapy to correct fluid and electrolyte imbalances b. Antiemetic medication, such as pyridoxine, to control nausea and vomiting c. Enteral nutrition to correct nutritional deficits d. Corticosteroids to reduce inflammation

A

A woman at 10 weeks of gestation who is seen in the prenatal clinic with presumptive signs and symptoms of pregnancy likely will have: A. Amenorrhea. B. Chadwick's sign. C. Positive pregnancy test. D. Hegar's sign.

A (Amenorrhea is a presumptive sign of pregnancy. Presumptive signs of pregnancy are felt by the woman. A positive pregnancy test, the presence of Chadwick's sign, and the presence of Hegar's sign all are probable signs of pregnancy.)

A first-time mother at 18 weeks of gestation comes for her regularly scheduled prenatal visit. The client tells the nurse that she is afraid that she is going into premature labor because she is beginning to have regular contractions. The nurse explains that this is the Braxton Hicks sign and teaches the client that this type of contraction. A. Is painless. B. Causes cervical dilation. B. Increases with walking. D. Impedes oxygen flow to the fetus.

A (Uterine contractions can be felt through the abdominal wall soon after the fourth month of gestation. Braxton Hicks contractions are regular and painless and continue throughout the pregnancy. Although they are not painful, some women complain that they are annoying. Braxton Hicks contractions usually cease with walking or exercise. They can be mistaken for true labor; however, they do not increase in intensity or frequency or cause cervical dilation. In addition, they facilitate uterine blood flow through the intervillous spaces of the placenta and promote oxygen delivery to the fetus.)

The nurse providing care for the laboring woman should understand that amnioinfusion is used to treat: A. Variable decelerations B. Late decelerations C. Fetal bradycardia D. Fetal tachycardia

A A. Correct: Amnioinfusion is used during labor either to dilute meconium-stained amniotic fluid or to supplement the amount of amniotic fluid to reduce the severity of variable decelerations caused by cord compression. B. Incorrect: Amnioinfusion has no bearing on this alteration in FHR tracings. C. Incorrect: Amnioinfusion has no bearing on this alteration in FHR tracings. D. Incorrect: Amnioinfusion has no bearing on this alteration in FHR tracings. p. 508

Because a full bladder prevents the uterus from contracting normally, nurses intervene to help the woman spontaneously empty her bladder as soon as possible. If all else fails, what tactic might the nurse use? a. Inserting a sterile catheter b. Pouring water from a squeeze bottle over the woman's perineum c. Placing oil of peppermint in a bedpan under the woman d. Asking the physician to prescribe analgesic agents

A If all else fails, the woman will be straight cathed to relieve bladder distension.

A woman in preterm labor at 30 weeks of gestation receives two 12-mg intramuscular (IM) doses of betamethasone. What is the purpose of this pharmacologic intervention? a. To stimulate fetal surfactant production b. To suppress uterine contractions c. To maintain adequate maternal respiratory effort and ventilation during magnesium sulfate therapy d. To reduce maternal and fetal tachycardia associated with ritodrine administration

A Important as baby may deliver preterm.

The nurse is working with a 36-year-old, married client, with 6 children who smokes. The woman states, "I don't expect to have any more kids, but I hate the thought of being sterile." Which of the following contraceptive methods would be best for the nurse to recommend to this client? a. Intrauterine contraceptive device (IUD) b. Contraceptive patch c. Bilateral tubal ligation d. Birth control pills with estrogen and progestin

A Intrauterine contraception device (IUD) is the recommended method for this patient. IUD has a low failure rate and provides long-term contraception for 3-10 years. Bilateral tubal ligation (BTL) is a surgical procedure which results in sterilization. Due to her history of smoking, neither birth control pills with estrogen nor contraceptive patch is recommended due to the increased risks for blood clots, heart disease, and strokes, also associated with smoking.

What are the most common causes for subinvolution of the uterus? a. Retained placental fragments and infection b. Uterine tetany and overproduction of oxytocin c. Postpartum hemorrhage and infection d. Multiple gestation and postpartum hemorrhage

A Involution is the return of the uterus to a nonpregnant state after birth. Subinvolution is the failure of the uterus to return to a non-pregnant state. The most common causes of subinvolution are retained placental fragments and infection

A nurse working with an infertile couple has made the following nursing diagnosis: Sexual dysfunction related to decreased libido. Which of the following assessments is the likely reason for this diagnosis? a. The couple has established a set schedule for their sexual encounters. b. The couple has been married for more than 8 years. c. The couple lives with one set of parents. d. The couple has close friends who gave birth within the last year.

A Pregnancy occurs with ovulation; sexual intercourse outside the time of ovulation will not result in conception.

A couple who has been attempting to become pregnant for 5 years is seeking assistance from an infertility clinic. The nurse assesses the clients' emotional responses to their infertility. Which of the following responses would the nurse expect to find? (Select all that apply.) a. Anger at others who have babies. b. Feelings of failure because they cannot make a baby. c. Sexual excitement because they want to conceive a baby. d. Guilt on the part of one partner because he or she is unable to give the other a baby.

A, B, D Infertility can be seen as a crisis in the couple's lives and relationship. The diagnosis and treatment of infertility can cause anger with others who have babies, feelings of failure, and feelings of guile. Sexual dysfunction can occur based on the type of infertility testing required and the method of treatment.

Pre-surgical nursing care for the woman who is going to have a hysterectomy for ovarian cancer includes which of the following? (Select all that apply). a. administer enema if ordered b. identification band is in place c. Sips of water for hydration d. ensure lab results are available to the surgeon prior to surgery e. teach post-op routine care such as need for early ambulation

A, B, D, E

16 y/o Susie presents to the clinic, wanting a pregnancy test. What questions would be appropriate for the professional nurse to ask Susie prior to completing the pregnancy test? (Select one or more) a. Have you been using birth control? b. What is the first day of your last menstrual period? c. Does your mother know you are wanting a pregnancy test? d. Are you excited that you might be pregnant? e. Have you done a home pregnancy test?

A, B, E

Which of the following lab tests are routinely performed on pregnant women? (Select all that apply). Select one or more: a. Syphilis test b. Rubella titer c. Genetic testing for cystic fibrosis d. 3 hr GTT e. Blood type

A, B, E 3 hr GTT is only done for abnormal 1 hrGTT or increased risk for diabetes in pregnancy

Which congenital anomalies can occur as a result of the use of antiepileptic drugs (AEDs) in pregnancy? (Select all that apply.) a. Neural tube defects b. Gastroschisis c. Congenital heart disease d. Cleft lip e. Diaphragmatic hernia

A, C, D

Which of the following nursing interventions are important in the prenatal care of the woman with prenatal depression? (Select all of the following). a. Maintain a caring relationship b. Counsel her on the importance of medication if the woman refuses it. c. Recommend she see a psychiatrist for management d. Educate the woman about depression and plan of care

A, C, D

25 year old Susan (G0000) is thinking about getting pregnant this next year, but wants an effective method right now. She currently uses condoms. She denies medical problems. Which of the following birth control methods are most appropriate for the nurse to discuss with Susan? (Select all that apply). a. The birth control patch b. The implant c. Vaginal ring d. Condoms (continue) e. Oral contraceptive pills

A, C, D, E Long-acting contraception is not the best choice for a woman who desires pregnancy within one year. More appropriate methods include birth control pills and the patch, and the vaginal ring as they are not long acting and are reversible once the woman stops using them. Condoms are short acting with no hormones.

The nurse suspects that her postpartum client is experiencing hemorrhagic shock. Which observation indicates or would confirm this diagnosis? a.Absence of cyanosis in the buccal mucosa b.Cool, dry skin c.Calm mental status d.Urinary output of at least 30 ml/hr

ANS: D Hemorrhage may result in hemorrhagic shock. Shock is an emergency situation during which the perfusion of body organs may become severely compromised, and death may occur. The presence of adequate urinary output indicates adequate tissue perfusion. The assessment of the buccal mucosa for cyanosis can be subjective. The presence of cool, pale, clammy skin is associated with hemorrhagic shock. Hemorrhagic shock is associated with lethargy, not restlessness.

1. A woman gave birth vaginally to a 9-lb, 12-oz girl yesterday. Her primary health care provider has written orders for perineal ice packs, use of a sitz bath tid, and a stool softener. What information is most closely correlated with these orders? a. The woman is a gravida 2, para 2 b. The woman had a vacuum-assisted birth c. The woman received epidural anesthesia d. The woman has an episiotomy

ANS: D These orders are typical interventions for a woman who has had an episiotomy, lacerations, and hemorrhoids. A multiparous classification is not an indication for these orders. A vacuum-assisted birth may be used in conjunction with an episiotomy, which would indicate these interventions. Use of epidural anesthesia has no correlation with these orders.

The nurse caring for the laboring woman should understand that early decelerations are caused by:

Altered fetal cerebral blood flow. [Early decelerations are the fetus's response to fetal head compression]

Another common pregnancy-specific condition is pruritic urticarial papules and plaques of pregnancy (PUPPP). A client asks the nurse why she has developed this condition and what can be done. What is the nurse's best response? a. PUPPP is associated with decreased maternal weight gain. b. The goal of therapy is to relieve discomfort. c. This common pregnancy-specific condition is associated with a poor fetal outcome. d. The rate of hypertension decreases with PUPPP.

B

In caring for the preterm infant, what complication is thought to be a result of high arterial blood oxygen level? a. BPD b. ROP c. IVH d. NEC

B

What is the highest priority nursing intervention when admitting a pregnant woman who has experienced a bleeding episode in late pregnancy? a. Performing a venipuncture for hemoglobin and hematocrit levels b. Assessing FHR and maternal vital signs c. Placing clean disposable pads to collect any drainage d. Monitoring uterine contractions

B

On examining a woman who gave birth 5 hours ago, the nurse finds that the woman has completely saturated a perineal pad within 15 minutes. What is the nurse's highest priority at this time? a. Assessing the woman's vital signs b. Massaging the woman's fundus c. Calling the woman's primary health care provider d. Beginning an intravenous (IV) infusion of Ringer's lactate solution

B The initial intervention in management of excessive postpartum bleeding due to uterine atony is firm massage of the uterine fundus. Expression of any clots in the uterus, elimination of bladder distention, and continuous IV infusion of 10 to 40 units of oxytocin added to 1000ml of lactated ringers or normal saline solution also are primary interventions.

Which of the following perinatal complications have been associated with untreated asymptomatic bacteriuria? (Select all that apply). a. congenital birth defects b. Preterm birth c. Low birth weight d. chorioamniotis

B, D

A 36 y/o pregnant woman has been diagnosed with polyhydraminos. The nurse knows this is based on which of the following? a. Amniotic fluid index of 20 cm b. Amniotic fluid index of 7 cm c. Amniotic fluid index of 30 cm d. Amniotic fluid index of 10 cm

C

An infant at 36 weeks of gestation has increasing respirations (80 to 100 breaths per minute with significant substernal retractions). The infant is given oxygen by continuous nasal positive airway pressure (CPAP). What level of partial pressure of arterial oxygen (PaO2) indicates hypoxia? a. 89 mmHg b. 73 mmHg c. 45 mmHg d. 67 mmHg

C

The nurse caring for the woman in labor should understand that increased variability of the fetal heart rate may be caused by: a. Narcotics. c. Methamphetamines. b. Barbiturates. d. Tranquilizers.

C Narcotics, barbiturates, and tranquilizers may be causes of decreased variability; methamphetamines may cause increased variability.

25. How many kilocalories per kilogram (kcal/kg) of body weight does a breastfed term infant require each day? a.50 to 65 b.75 to 90 c.95 to 110 d.150 to 200

C (For the first 3 months, the infant needs 110 kcal/kg/day. At ages 3 to 6 months, the requirement is 100 kcal/kg/day. This level decreases slightly to 95 kcal/kg/day from 6 to 9 months and increases again to 100 kcal/kg/day until the baby reaches 12 months.)

6. At a 2-month well-baby examination, it was discovered that an exclusively breastfed infant had only gained 10 ounces in the past 4 weeks. The mother and the nurse develop a feeding plan for the infant to increase his weight gain. Which change in dietary management will assist the client in meeting this goal? a.Begin solid foods. b.Have a bottle of formula after every feeding. c.Have one extra breastfeeding session every 24 hours. d.Start iron supplements.

C (Usually the solution to slow weight gain is to improve the feeding technique. Position and the latch-on technique are evaluated, and adjustments are made. Adding a feeding or two within a 24-hour period might help. Solid foods should not be introduced to an infant for at least 4 to 6 months. Bottle feeding may cause nipple confusion and may limit the supply of milk. Iron supplements have no bearing on weight gain.)

For which of the following conditions is colposcopy used to further evaluate need for intervention? (Select all that apply). a. uterine fibroids b. bacterial vaginosis c. vaginal cancer d. uterine cancer e. cervical cancer

C, E

Normal uterine activity pattern in labor is characterized by:

Contractions every 2 to 5 minutes. [Contractions normally occur every 2 to 5 minutes and last less than 90 seconds with about 30 seconds in between.]

A first-time dad is concerned that his 3-day-old daughter's skin looks "yellow." In the nurse's explanation of physiologic jaundice, what fact should be included? a. Physiologic jaundice is also known as breast milk jaundice. b. Physiologic jaundice is caused by blood incompatibilities between the mother and the infant blood types. c. Physiologic jaundice occurs during the first 24 hours of life. d. Physiologic jaundice becomes visible when serum bilirubin levels peak between the second and fourth days of life

D

A primigravida is being monitored at the prenatal clinic for preeclampsia. Which finding is of greatest concern to the nurse? a. Weight gain of 0.5 kg during the past 2 weeks b. Pitting pedal edema at the end of the day c. Blood pressure (BP) increase to 138/86 mm Hg d. Dipstick value of 3+ for protein in her urine

D

A woman at 28 weeks of gestation experiences blunt abdominal trauma as the result of a fall. The nurse must closely observe the client for what? a. Alteration in maternal vital signs, especially blood pressure b. Complaints of abdominal pain c. Hemorrhage d. Placental abruption

D

An infant was born 2 hours ago at 37 weeks of gestation and weighs 4.1 kg. The infant appears chubby with a flushed complexion and is very tremulous. The tremors are most likely the result of what condition? a. Seizures b. Birth injury c. Hypocalcemia d. Hypoglycemia

D

In contrast to placenta previa, what is the most prevalent clinical manifestation of Grade 3 abruptio placentae? a. Bleeding b. Cramping c. Intermittent uterine contractions d. Intense abdominal pain

D

Postoperative care of the pregnant woman who requires abdominal surgery for appendicitis includes which additional assessment? a. Signs and symptoms of infection b. Vital signs and incision c. Intake and output (I&O) and intravenous (IV) site d. Fetal heart rate (FHR) and uterine activity

D

Which conditions are infants of diabetic mothers (IDMs) at a higher risk for developing? a. Hyponatremia b. Sepsis c. Iron deficiency anemia d. Respiratory distress syndrome

D

Which statement regarding the laboratory test for glycosylated hemoglobin Alc is correct? a. This laboratory test is a snapshot of glucose control at the moment. b. This laboratory test is performed on the woman's urine, not her blood. c. The laboratory test for glycosylated hemoglobin Alc is performed for all pregnant women, not only those with or likely to have diabetes. d. This laboratory test measures the levels of hemoglobin Alc, which should remain at less than 7%

D

A patient, G1 P0, is admitted to the labor and delivery unit for induction of labor. The following assessments were made on admission: Bishop score of 4, fetal heart rate 140s with moderate variability and no decelerations, TPR 98.6F, 88, 20, BP 120/80, negative obstetrical history. A prostaglandin suppository was inserted at that time. Which of the following findings, 6 hours after insertion, would warrant the removal of the Cervidil (dinoprostone)? a. Bishop score of 5 b. Fetal heart of 152 bpm c. Respiratory rate of 24 rpm d. More than 5 contractions in 10 minutes

D This is hyperstimulation

Which deceleration of the fetal heart rate would not require the nurse to change the maternal position?

Early decelerations [Early decelerations and accelerations generally do not need any nursing intervention.]

Peaking at 40 to 70 mm Hg in the first stage of labor

Strength

What is an advantage of external electronic fetal monitoring?

The tocotransducer is especially valuable for measuring uterine activity during the first stage of labor.

A nurse may be called on to stimulate the fetal scalp:

To elicit an acceleration in the fetal heart rate (FHR). [The scalp can be stimulated using digital pressure during a vaginal examination]

All infants born to mothers with diabetes are at some risk for complications. True or false?

True The degree of risk is influenced by the severity and duration of maternal disease. p. 996

The nurse providing care for the laboring woman understands that accelerations with fetal movement: a) Are reassuring b) Are caused by umbilical cord compression c) Warrant close observation d) Are caused by uteroplacental insufficiency

a) Are reassuring

A nurse caring for a woman in labor understands that increased variability of the fetal heart rate might be caused by: a) Narcotics b) Barbiturates c) Methamphetamines d) Tranquilizers

c) Methamphetamines

A new client and her partner arrive on the labor, delivery, recovery, and postpartum unit for the birth of their first child. You apply the electronic fetal monitor (EFM) to the woman. Her partner asks you to explain what is printing on the graph, referring to the EFM strip. He wants to know what the baby's heart rate should be. Your best response is:

"The top line graphs the baby's heart rate. Generally the heart rate is between 110 and 160. The heart rate will fluctuate in response to what is happening during labor."

MATCHING Because most pregnant women continue their usual activities, trauma remains a common complication during pregnancy. Approximately 1 in 12 pregnancies in the United States are complicated by trauma each year. As a result of the physiologic alterations that accompany pregnancy, special considerations for the mother and fetus are necessary when trauma occurs. Match the maternal system adaptation in pregnancy with the clinical response to trauma. a. Increased oxygen consumption b. Increased heart rate c. Decreased gastric motility d. Displacement of abdominal viscera e. Increase in clotting factors 1. Decreased placental perfusion in the supine position 2. Increased risk of thrombus formation 3. Altered pain referral 4. Increased risk of acidosis 5. Increased risk of aspiration

1. ANS: B DIF: Cognitive Level: Apply REF: p. 732 TOP: Nursing Process: Assessment MSC: Client Needs: Physiologic Integrity NOT: Immediate priorities for the stabilization of the pregnant woman after trauma should be identical to that of the nonpregnant client after trauma. Fetal survival depends on maternal survival, and stabilization of the mother improves the chance of fetal well-being. Trauma may affect a number of systems within the body, and being aware of normal system alterations in the pregnant woman is important for the nurse who is caring for this client. Care should be adapted according to the body system that has been injured. The effects of trauma on pregnancy are also influenced by the length of gestation, type and severity of the injuries, and the degree of disruption of uterine and fetal physiologic features. 2. ANS: E DIF: Cognitive Level: Apply REF: p. 732 TOP: Nursing Process: Assessment MSC: Client Needs: Physiologic Integrity NOT: Immediate priorities for the stabilization of the pregnant woman after trauma should be identical to that of the nonpregnant client after trauma. Fetal survival depends on maternal survival, and stabilization of the mother improves the chance of fetal well-being. Trauma may affect a number of systems within the body, and being aware of normal system alterations in the pregnant woman is important for the nurse who is caring for this client. Care should be adapted according to the body system that has been injured. The effects of trauma on pregnancy are also influenced by the length of gestation, type and severity of the injuries, and the degree of disruption of uterine and fetal physiologic features. 3. ANS: D DIF: Cognitive Level: Apply REF: p. 732 TOP: Nursing Process: Assessment MSC: Client Needs: Physiologic Integrity NOT: Immediate priorities for the stabilization of the pregnant woman after trauma should be identical to that of the nonpregnant client after trauma. Fetal survival depends on maternal survival, and stabilization of the mother improves the chance of fetal well-being. Trauma may affect a number of systems within the body, and being aware of normal system alterations in the pregnant woman is important for the nurse who is caring for this client. Care should be adapted according to the body system that has been injured. The effects of trauma on pregnancy are also influenced by the length of gestation, type and severity of the injuries, and the degree of disruption of uterine and fetal physiologic features. 4. ANS: A DIF: Cognitive Level: Apply REF: p. 732 TOP: Nursing Process: Assessment MSC: Client Needs: Physiologic Integrity NOT: Immediate priorities for the stabilization of the pregnant woman after trauma should be identical to that of the nonpregnant client after trauma. Fetal survival depends on maternal survival, and stabilization of the mother improves the chance of fetal well-being. Trauma may affect a number of systems within the body, and being aware of normal system alterations in the pregnant woman is important for the nurse who is caring for this client. Care should be adapted according to the body system that has been injured. The effects of trauma on pregnancy are also influenced by the length of gestation, type and severity of the injuries, and the degree of disruption of uterine and fetal physiologic features. 5. ANS: C DIF: Cognitive Level: Apply REF: p. 732 TOP: Nursing Process: Assessment MSC: Client Needs: Physiologic Integrity NOT: Immediate priorities for the stabilization of the pregnant woman after trauma should be identical to that of the nonpregnant client after trauma. Fetal survival depends on maternal survival, and stabilization of the mother improves the chance of fetal well-being. Trauma may affect a number of systems within the body, and being aware of normal system alterations in the pregnant woman is important for the nurse who is caring for this client. Care should be adapted according to the body system that has been injured. The effects of trauma on pregnancy are also influenced by the length of gestation, type and severity of the injuries, and the degree of disruption of uterine and fetal physiologic features.

Biophysical risks include factors that originate with either the mother or the fetus and affect the functioning of either one or both. The nurse who provides prenatal care should have an understanding of these risk factors. Match the specific pregnancy problem with the related risk factor. a.Polyhydramnios b.IUGR (maternal cause) c.Oligohydramnios d.Chromosomal abnormalities e.IUGR (fetoplacental cause) 1. Premature rupture of membranes 2. Advanced maternal age 3. Fetal congenital anomalies 4. Abnormal placenta development 5. Smoking, alcohol, and illicit drug use

1. ANS: C DIF: Cognitive Level: Understand REF: p. 635 TOP: Nursing Process: Implementation MSC: Client Needs: Health Promotion and Maintenance 2. ANS: D DIF: Cognitive Level: Understand REF: p. 635 TOP: Nursing Process: Implementation MSC: Client Needs: Health Promotion and Maintenance 3. ANS: A DIF: Cognitive Level: Understand REF: p. 635 TOP: Nursing Process: Implementation MSC: Client Needs: Health Promotion and Maintenance 4. ANS: E DIF: Cognitive Level: Understand REF: p. 635 TOP: Nursing Process: Implementation MSC: Client Needs: Health Promotion and Maintenance 5. ANS: B DIF: Cognitive Level: Understand REF: p. 635

PPH may be sudden and result in rapid blood loss. The nurse must be alert to the symptoms of hemorrhage and hypovolemic shock and be prepared to act quickly to minimize blood loss. Astute assessment of the client's circulatory status can be performed with noninvasive monitoring. Match the type of noninvasive assessment that the nurse would perform with the appropriate clinical manifestation or body system. a.Palpation b.Auscultation c.Inspection d.Observation e.Measurement 1. Pulse oximetry 2. Heart sounds 3. Arterial pulses 4. Skin color, temperature, and turgor 5. Presence or absence of anxiety

1. ANS: E DIF: Cognitive Level: Apply REF: p. 808 TOP: Nursing Process: Assessment MSC: Client Needs: Physiologic Integrity NOT: To perform a complete noninvasive assessment of the circulatory status in postpartum clients who are bleeding, the nurse must perform the following: palpation (rate, quality, equality) of arterial pulses; auscultation of heart sounds or murmurs and breath sounds; inspection of skin color, temperature, and turgor; level of consciousness; capillary refill, neck veins, and mucous membranes; observation of either the presence or absence of anxiety, apprehension, restlessness, and disorientation; and measurement of blood pressure, pulse oximetry, and urinary output. 2. ANS: B DIF: Cognitive Level: Apply REF: p. 808 TOP: Nursing Process: Assessment MSC: Client Needs: Physiologic Integrity NOT: To perform a complete noninvasive assessment of the circulatory status in postpartum clients who are bleeding, the nurse must perform the following: palpation (rate, quality, equality) of arterial pulses; auscultation of heart sounds or murmurs and breath sounds; inspection of skin color, temperature, and turgor; level of consciousness; capillary refill, neck veins, and mucous membranes; observation of either the presence or absence of anxiety, apprehension, restlessness, and disorientation; and measurement of blood pressure, pulse oximetry, and urinary output. 3. ANS: A DIF: Cognitive Level: Apply REF: p. 808 TOP: Nursing Process: Assessment MSC: Client Needs: Physiologic Integrity NOT: To perform a complete noninvasive assessment of the circulatory status in postpartum clients who are bleeding, the nurse must perform the following: palpation (rate, quality, equality) of arterial pulses; auscultation of heart sounds or murmurs and breath sounds; inspection of skin color, temperature, and turgor; level of consciousness; capillary refill, neck veins, and mucous membranes; observation of either the presence or absence of anxiety, apprehension, restlessness, and disorientation; and measurement of blood pressure, pulse oximetry, and urinary output. 4. ANS: C DIF: Cognitive Level: Apply REF: p. 808 TOP: Nursing Process: Assessment MSC: Client Needs: Physiologic Integrity NOT: To perform a complete noninvasive assessment of the circulatory status in postpartum clients who are bleeding, the nurse must perform the following: palpation (rate, quality, equality) of arterial pulses; auscultation of heart sounds or murmurs and breath sounds; inspection of skin color, temperature, and turgor; level of consciousness; capillary refill, neck veins, and mucous membranes; observation of either the presence or absence of anxiety, apprehension, restlessness, and disorientation; and measurement of blood pressure, pulse oximetry, and urinary output. 5. ANS: D DIF: Cognitive Level: Apply REF: p. 808 TOP: Nursing Process: Assessment MSC: Client Needs: Physiologic Integrity NOT: To perform a complete noninvasive assessment of the circulatory status in postpartum clients who are bleeding, the nurse must perform the following: palpation (rate, quality, equality) of arterial pulses; auscultation of heart sounds or murmurs and breath sounds; inspection of skin color, temperature, and turgor; level of consciousness; capillary refill, neck veins, and mucous membranes; observation of either the presence or absence of anxiety, apprehension, restlessness, and disorientation; and measurement of blood pressure, pulse oximetry, and urinary output.

A woman is 6 weeks pregnant. She has had a previous spontaneous abortion at 14 weeks of gestation and a pregnancy that ended at 38 weeks with the birth of a stillborn girl. What is her gravidity and parity using the GTPAL system? ___________________

3-1-0-1-0 (The correct calculation of this woman's gravidity and parity is 3-1-0-1-0. Using the GPTAL system, this client's gravidity and parity information is calculated as follows: G: Total number of times the woman has been pregnant (she is pregnant for the third time) T: Number of pregnancies carried to term (she has had only one pregnancy that resulted in a fetus at term) P: Number of pregnancies that resulted in a preterm birth (none) A: Abortions or miscarriages before the period of viability (she has had one) L: Number of children born who are currently living (she has no living children))

A woman is 6 weeks pregnant. She has had a previous spontaneous abortion at 14 weeks of gestation and a pregnancy that ended at 38 weeks with the birth of a stillborn girl. Record her gravidity and parity using the GTPAL system as x-x-x-x-x.

3-1-0-1-0 (Using the GPTAL system , this woman's gravidity and parity information is calculated as follows: G: Total number of times the woman has been pregnant (she is pregnant for the third time); T: Number of pregnancies carried to term (she has one stillborn); P: Number of pregnancies that resulted in a preterm birth (she has none); A: Abortions or miscarriages before the period of viability (she has had one); L: Number of children born who are currently living (she has no living children))

A pregnant woman is the mother of two children. Her first pregnancy ended in a stillbirth at 32 weeks of gestation, her second pregnancy with the birth of her daughter at 36 weeks, and her third pregnancy with the birth of her son at 41 weeks. Using the 5-digit system (#-#-#-#-#) to describe this woman's current obstetric history, the nurse records ____________________________.

4-1-2-0-2 (Gravida (the first number) is 4 because this woman is now pregnant and was pregnant 3 times before. Para (the next 4 numbers) represents the outcomes of the pregnancies and is described as: 4T: 1 = term birth at 41 weeks of gestation (son) 4P: 2 = preterm birth at 32 weeks of gestation (stillbirth) and 36 weeks of gestation (daughter) 4A: 0 = abortion: none occurred 4L: 2 = living children: her son and her daughter.)

2. Which condition would require prophylaxis to prevent subacute bacterial endocarditis (SBE) both antepartum and intrapartum? a. Valvular heart disease b. Congestive heart disease c. Arrhythmias d. Postmyocardial infarction

: A Prophylaxis for intrapartum endocarditis and pulmonary infection may be provided for women who have mitral valve prolapse. Prophylaxis for intrapartum endocarditis is not indicated for a client with congestive heart disease, underlying arrhythmias, or postmyocardial infarction. DIF: Cognitive Level: Understand REF: p. 712 TOP: Nursing Process: Implementation

MULTIPLE RESPONSE 1. Which congenital anomalies can occur as a result of the use of antiepileptic drugs (AEDs) in pregnancy? (Select all that apply.) a. Cleft lip b. Congenital heart disease c. Neural tube defects d. Gastroschisis e. Diaphragmatic hernia

: A, B, C Congenital anomalies that can occur with AEDs include cleft lip or palate, congenital heart disease, urogenital defects, and neural tube defects. Carbamazepine and valproate should be avoided if all possible; they may cause neural tube defects. Congenital anomalies of gastroschisis and diaphragmatic hernia are not associated with the use of AEDs. DIF: Cognitive Level: Understand REF: p. 725 TOP: Nursing Process: Planning

3. In caring for a pregnant woman with sickle cell anemia, the nurse must be aware of the signs and symptoms of a sickle cell crisis. What do these include? (Select all that apply.) a. Fever b. Endometritis c. Abdominal pain d. Joint pain e. Urinary tract infection (UTI)

: A, C, D Women with sickle cell anemia have recurrent attacks (crises) of fever and pain, most often in the abdomen, joints, and extremities. These attacks are attributed to vascular occlusion when red blood cells (RBCs) assume the characteristic sickled shape. Crises are usually triggered by dehydration, hypoxia, or acidosis. Women with the sickle cell trait are usually at a greater risk for postpartum endometritis (uterine wall infection); however, this development is not likely to occur during the pregnancy and is not a sign for the disorder. Although women with sickle cell anemia are at an increased risk for UTIs, these infections are not an indication of a sickle cell crisis. DIF: Cognitive Level: Understand REF: p. 721 TOP: Nursing Process: Assessment

2. A lupus flare-up during pregnancy or early postpartum occurs in 15% to 60% of women with this disorder. Which conditions associated with systemic lupus erythematosus (SLE) are maternal risks? (Select all that apply.) a. Miscarriage b. Intrauterine growth restriction (IUGR) c. Nephritis d. Preeclampsia e. Cesarean birth

: A, C, D, E Maternal risks associated with SLE include miscarriage, nephritis, preeclampsia, and cesarean birth. IUGR is a fetal risk related to SLE. Other fetal risks include stillbirth and prematurity. DIF: Cognitive Level: Understand REF: p. 727 TOP: Nursing Process: Assessment

3. Which information should the nurse take into consideration when planning care for a postpartum client with cardiac disease? a. The plan of care for a postpartum client is the same as the plan for any pregnant woman. b. The plan of care includes rest, stool softeners, and monitoring of the effect of activity. c. The plan of care includes frequent ambulating, alternating with active range-of-motion exercises. d. The plan of care includes limiting visits with the infant to once per day.

: B Bed rest may be ordered, with or without bathroom privileges. Bowel movements without stress or strain for the woman are promoted with stool softeners, diet, and fluids. Care of the woman with cardiac disease in the postpartum period is tailored to the woman's functional capacity. The woman will be on bed rest to conserve energy and to reduce the strain on the heart. Although the woman may need help caring for the infant, breastfeeding and infant visits are not contraindicated. DIF: Cognitive Level: Understand REF: pp. 718-719 TOP: Nursing Process: Planning

9. Which information regarding the care of antepartum women with cardiac conditions is mostimportant for the nurse to understand? a. Stress on the heart is greatest in the first trimester and the last 2 weeks before labor. b. Women with class II cardiac disease should avoid heavy exertion and any activity that causes even minor symptoms. c. Women with class III cardiac disease should get 8 to 10 hours of sleep every day and limit housework, shopping, and exercise. d. Women with class I cardiac disease need bed rest through most of the pregnancy and face the possibility of hospitalization near term.

: B Class II cardiac disease is symptomatic with ordinary activity. Women in this category need to avoid heavy exertion and limit regular activities as symptoms dictate. Stress is greatest between weeks 28 and 32 of gestation, when hemodynamic changes reach their maximum. Class III cardiac disease is symptomatic with less-than-ordinary activity. These women need bed rest most of the day and face the possibility of hospitalization near term. Class I cardiac disease is asymptomatic at normal levels of activity. These women can perform limited normal activities with discretion, although they still need a good amount of sleep. DIF: Cognitive Level: Understand REF: p. 711 TOP: Nursing Process: Planning

12. The client makes an appointment for preconception counseling. The woman has a known heart condition and is unsure if she should become pregnant. Which is the only cardiac condition that would cause concern? a. Marfan syndrome b. Eisenmenger syndrome c. Heart transplant d. Ventricular septal defect (VSD)

: B Pregnancy is contraindicated in clients with Eisenmenger syndrome. Women who have had heart transplants are successfully having babies. However, conception should be postponed for at least 1 year after transplantation. Management of the client with Marfan syndrome during pregnancy includes bed rest, beta-blockers, and surgery before conception. VSD is usually corrected early in life and is therefore not a contraindication to pregnancy. DIF: Cognitive Level: Understand REF: p. 714 TOP: Nursing Process: Assessment

15. Bell palsy is an acute idiopathic facial paralysis, the cause for which remains unknown. Which statement regarding this condition is correct? a. Bell palsy is the sudden development of bilateral facial weakness. b. Women with Bell palsy have an increased risk for hypertension. c. Pregnant women are affected twice as often as nonpregnant women. d. Bell palsy occurs most frequently in the first trimester.

: B The clinical manifestations of Bell palsy include the development of unilateral facial weakness, pain surrounding the ears, difficulty closing the eye, and hyperacusis. The cause is unknown; however, Bell palsy may be related to a viral infection. Pregnant women are affected at a rate of three to five times that of nonpregnant women. The incidence rate peaks during the third trimester and puerperium. Women who develop Bell palsy in pregnancy have an increased risk for hypertension. DIF: Cognitive Level: Understand REF: p. 726 TOP: Nursing Process: Assessment

18. It is extremely rare for a woman to die in childbirth; however, it can happen. In the United States, the annual occurrence of maternal death is 12 per 100,000 cases of live birth. What are the leading causes of maternal death? a. Embolism and preeclampsia b. Trauma and motor vehicle accidents (MVAs) c. Hemorrhage and infection d. Underlying chronic conditions

: B Trauma is the leading cause of obstetric death in women of childbearing age. Most maternal injuries are the result of MVAs and falls. Although preeclampsia and embolism are significant contributors to perinatal morbidity, these are not the leading cause of maternal mortality. Maternal death caused by trauma may occur as the result of hemorrhagic shock or abruptio placentae. In these cases, the hemorrhage is the result of trauma, not childbirth. The wish to become a parent is not eliminated by a chronic health problem, and many women each year risk their lives to have a baby. Because of advanced pediatric care, many women are surviving childhood illnesses and reaching adulthood with chronic health problems such as cystic fibrosis, diabetes, and pulmonary disorders. DIF: Cognitive Level: Understand REF: p. 731 TOP: Nursing Process: Assessment

4. Autoimmune disorders often occur during pregnancy because a large percentage of women with an autoimmune disorder are of childbearing age. Which disorders fall into the category of collagen vascular disease? (Select all that apply.) a. Multiple sclerosis b. SLE c. Antiphospholipid syndrome d. Rheumatoid arthritis e. Myasthenia gravis

: B, C, D, E Multiple sclerosis is not an autoimmune disorder. This patchy demyelination of the spinal cord may be a viral disorder. Autoimmune disorders (collagen vascular disease) make up a large group of conditions that disrupt the function of the immune system of the body. These disorders include those listed, as well as systemic sclerosis. DIF: Cognitive Level: Comprehend REF: p. 726 TOP: Nursing Process: Assessment

6. Which important component of nutritional counseling should the nurse include in health teaching for a pregnant woman who is experiencing cholecystitis? a. Assess the woman's dietary history for adequate calories and proteins. b. Teach the woman that the bulk of calories should come from proteins. c. Instruct the woman to eat a low-fat diet and to avoid fried foods. d. Instruct the woman to eat a low-cholesterol, low-salt diet.

: C Eating a low-fat diet and avoiding fried foods is appropriate nutritional counseling for this client. Caloric and protein intake do not predispose a woman to the development of cholecystitis. The woman should be instructed to limit protein intake and choose foods that are high in carbohydrates. A low-cholesterol diet may be the result of limiting fats. However, a low-salt diet is not indicated. DIF: Cognitive Level: Apply REF: p. 728 TOP: Nursing Process: Implementation

10. A woman at 28 weeks of gestation experiences blunt abdominal trauma as the result of a fall. The nurse must closely observe the client for what? a. Alteration in maternal vital signs, especially blood pressure b. Complaints of abdominal pain c. Placental absorption d. Hemorrhage

: C Electronic fetal monitoring (EFM) tracings can help evaluate maternal status after trauma and can reflect fetal cardiac responses to hypoxia and hypoperfusion. Signs and symptoms of placental absorption include uterine irritability, contractions, vaginal bleeding, and changes in FHR characteristics. Hypoperfusion may be present in the pregnant woman before the onset of clinical signs of shock. EFM tracings show the first signs of maternal compromise, such as when the maternal heart rate, blood pressure, and color appear normal, yet the EFM printout shows signs of fetal hypoxia. Abdominal pain, in and of itself, is not the most important symptom. However, if it is accompanied by contractions, changes in the FHR, rupture of membranes, or vaginal bleeding, then the client should be evaluated for abruptio placentae. Clinical signs of hemorrhage do not appear until after a 30% loss of circulating volume occurs. Careful monitoring of fetal status significantly assists in maternal assessment, because the fetal monitor tracing works as an oximeter of internal well-being. DIF: Cognitive Level: Apply REF: p. 732 TOP: Nursing Process: Assessment

16. A pregnant woman at term is transported to the emergency department (ED) after a severe vehicular accident. The obstetric nurse responds and rushes to the ED with a fetal monitor. Cardiopulmonary arrest occurs as the obstetric nurse arrives. What is the highest priority for the trauma team? a. Obtaining IV access, and starting aggressive fluid resuscitation b. Quickly applying the fetal monitor to determine whether the fetus viability c. Starting cardiopulmonary resuscitation (CPR) d. Transferring the woman to the surgical unit for an emergency cesarean delivery in case the fetus is still alive

: C In a situation of severe maternal trauma, the systematic evaluation begins with a primary survey and the initial ABCs (airway, breathing, and circulation) of resuscitation. CPR is initiated first, followed by intravenous (IV) replacement fluid. After immediate resuscitation and successful stabilization measures, a more detailed secondary survey of the mother and fetus should be accomplished. Attempts at maternal resuscitation are made, followed by a secondary survey of the fetus. In the presence of multisystem trauma, a cesarean delivery may be indicated to increase the chance for maternal survival. DIF: Cognitive Level: Apply REF: p. 734 TOP: Nursing Process: Implementation

5. A woman with asthma is experiencing a postpartum hemorrhage. Which drug should be avoided when treating postpartum bleeding to avoid exacerbating asthma? a. Oxytocin (Pitocin) b. Nonsteroidal antiinflammatory drugs (NSAIDs) c. Hemabate d. Fentanyl

: C Prostaglandin derivatives should not be used to treat women with asthma, because they may exacerbate symptoms. Oxytocin is the drug of choice to treat this woman's bleeding; it will not exacerbate her asthma. NSAIDs are not used to treat bleeding. Fentanyl is used to treat pain, not bleeding. DIF: Cognitive Level: Analyze REF: p. 722 TOP: Nursing Process: Planning

4. A woman has experienced iron deficiency anemia during her pregnancy. She had been taking iron for 3 months before the birth. The client gave birth by cesarean 2 days earlier and has been having problems with constipation. After assisting her back to bed from the bathroom, the nurse notes that the woman's stools are dark (greenish-black). What should the nurse's initial action be? a. Perform a guaiac test, and record the results. b. Recognize the finding as abnormal, and report it to the primary health care provider. c. Recognize the finding as a normal result of iron therapy. d. Check the woman's next stool to validate the observation.

: C The nurse should recognize that dark stools are a common side effect in clients who are taking iron replacement therapy. A guaiac test would be indicated if gastrointestinal (GI) bleeding was suspected. GI irritation, including dark stools, is also a common side effect of iron therapy. Observation of stool formation is a normal nursing activity. DIF: Cognitive Level: Apply REF: p. 716 TOP: Nursing Process: Evaluation

11. Which neurologic condition would require preconception counseling, if at all possible? a. Eclampsia b. Bell palsy c. Epilepsy d. Multiple sclerosis

: C Women with epilepsy should receive preconception counseling, if at all possible. Achieving seizure control before becoming pregnant is a desirable state. Medication should also be carefully reviewed. Eclampsia may sometimes be confused with epilepsy, and Bell palsy is a form of facial paralysis; preconception counseling for either condition is not essential to care. Multiple sclerosis is a patchy demyelination of the spinal cord that does not affect the normal course of pregnancy or birth. DIF: Cognitive Level: Understand REF: p. 725 TOP: Nursing Process: Planning

7. Postoperative care of the pregnant woman who requires abdominal surgery for appendicitis includes which additional assessment? a. Intake and output (I&O) and intravenous (IV) site b. Signs and symptoms of infection c. Vital signs and incision d. Fetal heart rate (FHR) and uterine activity

: D Care of a pregnant woman undergoing surgery for appendicitis differs from that for a nonpregnant woman in one significant aspect: the presence of the fetus. Continuous fetal and uterine monitoring should take place. An assessment of I&O levels, along with an assessment of the IV site, are normal postoperative care procedures. Evaluating the client for signs and symptoms of infection is also part of routine postoperative care. Routine vital signs and evaluation of the incision site are expected components of postoperative care. DIF: Cognitive Level: Apply REF: p. 730 TOP: Nursing Process: Assessment

8. Since the gene for cystic fibrosis was identified in 1989, data can be collected for the purposes of genetic counseling for couples regarding carrier status. According to the most recent statistics, how often does cystic fibrosis occur in Caucasian live births? a. 1 in 100 b. 1 in 1000 c. 1 in 2000 d. 1 in 3200

: D Cystic fibrosis occurs in approximately 1 in 3200 Caucasian live births. 1 in 100, 1 in 1000, and 1 in 2000 occurrences of cystic fibrosis in live births are all too frequent rates. DIF: Cognitive Level: Remember REF: p. 722 TOP: Nursing Process: Assessment

13. What form of heart disease in women of childbearing years generally has a benign effect on pregnancy? a. Cardiomyopathy b. Rheumatic heart disease c. Congenital heart disease d. Mitral valve prolapse

: D Mitral valve prolapse is a benign condition that is usually asymptomatic. Cardiomyopathy produces congestive heart failure during pregnancy. Rheumatic heart disease can lead to heart failure during pregnancy. Some congenital heart diseases produce pulmonary hypertension or endocarditis during pregnancy. DIF: Cognitive Level: Remember REF: p. 713 TOP: Nursing Process: Assessment

14. A pregnant woman at 33 weeks of gestation is brought to the birthing unit after a minor automobile accident. The client is experiencing no pain and no vaginal bleeding, her vital signs are stable, and the FHR is 132 beats per minute with variability. What is the nurse's highestpriority? a. Monitoring the woman for a ruptured spleen b. Obtaining a physician's order to discharge her home c. Monitoring her for 24 hours d. Using continuous EFM for a minimum of 4 hours

: D Monitoring the external FHR and contractions is recommended after blunt trauma in a viable gestation for a minimum of 4 hours, regardless of injury severity. Fetal monitoring should be initiated as soon as the woman is stable. In this scenario, no clinical findings indicate the possibility of a ruptured spleen. If the maternal and fetal findings are normal, then EFM should continue for a minimum of 4 hours after a minor trauma or a minor automobile accident. Once the monitoring has been completed and the health care provider is reassured of fetal well-being, the client may be discharged home. Monitoring for 24 hours is unnecessary unless the ERM strip is abnormal or nonreassuring. DIF: Cognitive Level: Apply REF: p. 732 TOP: Nursing Process: Planning

17. Another common pregnancy-specific condition is pruritic urticarial papules and plaques of pregnancy (PUPPP). A client asks the nurse why she has developed this condition and what can be done. What is the nurse's bestresponse? a. PUPPP is associated with decreased maternal weight gain. b. The rate of hypertension decreases with PUPPP. c. This common pregnancy-specific condition is associated with a poor fetal outcome. d. The goal of therapy is to relieve discomfort.

: D PUPPP is associated with increased maternal weight gain, increased rate of twin gestation, and hypertension. It is not, however, associated with poor maternal or fetal outcomes. The goal of therapy is simply to relieve discomfort. Antipruritic topical medications, topical steroids, and antihistamines usually provide relief. PUPPP usually resolves before childbirth or shortly thereafter. DIF: Cognitive Level: Apply REF: p. 724 TOP: Nursing Process: Planning

1. When caring for a pregnant woman with cardiac problems, the nurse must be alert for the signs and symptoms of cardiac decompensation. Which critical findings would the nurse find on assessment of the client experiencing this condition? a. Regular heart rate and hypertension b. Increased urinary output, tachycardia, and dry cough c. Shortness of breath, bradycardia, and hypertension d. Dyspnea, crackles, and an irregular, weak pulse

: D Signs of cardiac decompensation include dyspnea; crackles; an irregular, weak, and rapid pulse; rapid respirations; a moist and frequent cough; generalized edema; increasing fatigue; and cyanosis of the lips and nailbeds. A regular heart rate and hypertension are not generally associated with cardiac decompensation. Of the symptoms of increased urinary output, tachycardia, and dry cough, only tachycardia is indicative of cardiac decompensation. Of the symptoms of shortness of breath, bradycardia, and hypertension, only dyspnea is indicative of cardiac decompensation. DIF: Cognitive Level: Understand REF: p. 716 TOP: Nursing Process: Assessment

A pregnant woman at 37 weeks of gestation has had ruptured membranes for 26 hours. A cesarean section is performed for failure to progress. The fetal heart rate (FHR) before birth is 180 beats per minute with limited variability. At birth the newborn has Apgar scores of 6 and 7 at 1 and 5 minutes and is noted to be pale and tachypneic. Based on the maternal history, what is the most likely cause of this newborn's distress? a. Sepsis b. Phrenic nerve injury c. Hypoglycemia d. Respiratory distress syndrome

A

According to research, which risk factor for PPD is likely to have the greatest effect on the client postpartum? a. Prenatal depression b. Single-mother status c. Low socioeconomic status d. Unplanned or unwanted pregnancy

A

For an infant experiencing symptoms of drug withdrawal, which intervention should be included in the plan of care? a. Snugly swaddling the infant and tightly holding the baby b. Playing soft music during feeding c. Administering chloral hydrate for sedation d. Feeding every 4 to 6 hours to allow extra rest between feedings

A

Having a genetic mutation may create an 85% chance of developing breast cancer in a woman's lifetime. Which condition does not increase a client's risk for breast cancer? a. Paget disease b. Li-Fraumeni syndrome c. BRCA1 or BRCA2 gene mutation d. Cowden syndrome

A

In caring for an immediate postpartum client, the nurse notes petechiae and oozing from her intravenous (IV) site. The client would be closely monitored for which clotting disorder? a. Disseminated Intravascular Coagulation (DIC) b. Hemorrhage c. HELLP syndrome d. Amniotic fluid embolism (AFE)

A

On day 3 of life, a newborn continues to require 100% oxygen by nasal cannula. The parents ask if they may hold their infant during his next gavage feeding. Considering that this newborn is physiologically stable, what response should the nurse provide? a. "You may hold your baby during the feeding." b. "You may only hold your baby's hand during the feeding." c. "Parents are not allowed to hold their infants who are dependent on oxygen." d. "Feedings cause more physiologic stress; therefore, the baby must be closely monitored. I don't think you should hold the baby."

A

Parents are often asked if they would like to have an autopsy performed on their infant. Nurses who are assisting parents with this decision should be aware of which information? a. Some religions prohibit autopsy. b. In the current litigious society, more autopsies are performed than in the past. c. Autopsies must be performed within a few hours after the infant's death. d. Autopsies are usually covered by insurance

A

Preeclampsia begins at what point during the pregnancy? a. At placental implantation b. After 20 weeks gestation c. At conception d. At 12 weeks gestation

A

Screening at 24 weeks of gestation reveals that a pregnant woman has gestational diabetes mellitus (GDM). In planning her care, the nurse and the client mutually agree that an expected outcome is to prevent injury to the fetus as a result of GDM. This fetus is at the greatest risk for which condition? a. Macrosomia b. Congenital anomalies of the central nervous system c. Preterm birth d. Low birth weight

A

What is one of the initial signs and symptoms of puerperal infection in the postpartum client? a. Temperature of 38° C (100.4° F) or higher on 2 successive days b. Fatigue continuing for longer than 1 week c. Profuse vaginal lochia with ambulation d. Pain with voiding

A

What is the primary purpose for magnesium sulfate administration for clients with preeclampsia and eclampsia? a. To prevent convulsions b. To prevent a boggy uterus and lessen lochial flow c. To shorten the duration of labor d. To improve patellar reflexes and increase respiratory efficiency

A

When caring for clients with neoplasms of the reproductive system, the nurse must begin by assessing the woman's knowledge of the disorder, its management, and prognosis. This assessment should be followed by a nursing diagnosis. Which diagnosis fails to address the psychologic effect of these disorders? a. Risk for injury, related to lack of skill for self-care b. Interrupted family processes c. Disturbed body image, as a result of changes in anatomy d. Anxiety, related to surgical procedures

A

Which clinical findings would alert the nurse that the neonate is expressing pain? a. Cry face; eyes squeezed; increase in blood pressure b. High-pitched, shrill cry; withdrawal; change in heart rate c. Low-pitched crying; tachycardia; eyelids open wide d. Cry face; flaccid limbs; closed mouth

A

Which condition is considered a medical emergency that requires immediate treatment? a. Inversion of the uterus b. Hypotonic uterus c. ITP d. Uterine atony

A

Which diagnostic test is used to confirm a suspected diagnosis of breast cancer? a. Needle-localization biopsy b. Ultrasound c. Magnetic resonance imaging (MRI) d. Mammogram

A

Which explanation will assist the parents in their decision on whether they should circumcise their son? Select one: a. The circumcision procedure has pros and cons during the prenatal period. b. Circumcision is rarely painful, and any discomfort can be managed without medication. c. American Academy of Pediatrics (AAP) recommends that all male newborns be routinely circumcised. d. The infant will likely be alert and hungry shortly after the procedure

A

Which of the following antepartum tests is (are) used to evaluate a Maternal Serum Screening test that is negative for increased risk? a. Further followup test not necessary b. Biophysical profile c. Amniocentesis d. CVS

A

A woman who has completed one pregnancy with a fetus (or fetuses) reaching the stage of fetal viability is called a: A. Primipara. B. Multipara. C. Primigravida. D. Nulligravida.

A (A primipara is a woman who has completed one pregnancy with a viable fetus. To remember terms, keep in mind: gravida is a pregnant woman; para comes from parity, meaning a viable fetus; primi means first; multi means many; and null means none. A primigravida is a woman pregnant for the first time. A multipara is a woman who has completed two or more pregnancies with a viable fetus. A nulligravida is a woman who has never been pregnant.)

During a client's physical examination the nurse notes that the lower uterine segment is soft on palpation. The nurse would document this finding as: A. Hegar's sign B. Chadwick's sign C. McDonald's sign D. Goodell's sign

A (At approximately 6 weeks of gestation, softening and compressibility of the lower uterine segment occur; this is called Hegar's sign. McDonald's sign indicates a fast food restaurant. Chadwick's sign is the blue-violet coloring of the cervix caused by increased vascularity; this occurs around the fourth week of gestation. Softening of the cervical tip is called Goodell's sign, which may be observed around the sixth week of pregnancy.)

During a woman's physical examination, the nurse notes that the lower uterine segment is soft on palpation. The nurse documents this finding as the: A. Hegar sign. B. McDonald sign. C. Chadwick sign. D. Goodell sign

A (At approximately six weeks of gestation, softening and compressibility of the lower uterine segment occur; this is called the Hegar sign. The McDonald sign is flexibility of the uterus at the junction of the cervix and uterus and usually can be detected at seven to eight weeks of gestation. The Chadwick sign is a blue-violet cervix caused by increased vascularity; this occurs around the fourth week of gestation. Softening of the cervical tip is called the Goodell sign, which may be observed around the sixth week of pregnancy.)

A patient who is pregnant used a home pregnancy test that showed a negative result. What will the nurse check for in the medication history of the patient? A. Diuretics B. Analgesics C. Tranquilizers D. Anticonvulsants

A (Diuretics are the medications that are usually prescribed to a patient with hypertension. These drugs may interfere with the levels of human chorionic gonadotropin (hCG) hormone. This may give a false-negative home pregnancy test result. Analgesics are the group of drugs used for pain relief. These drugs do not affect the hCG levels and therefore do not show a false report in the home pregnancy test. Tranquilizers are the drugs used for reducing anxiety, fear, and tension. The use of a tranquilizer results in a false-positive pregnancy test result because it increases hCG levels. Anticonvulsants are a group of drugs used in treating epileptic seizures; they affect the hCG levels and create a false-positive test result.)

A woman is in her seventh month of pregnancy. She has been complaining of nasal congestion and occasional epistaxis. The nurse suspects that: A. This is a normal respiratory change in pregnancy caused by elevated levels of estrogen. B. This is an abnormal cardiovascular change, and the nosebleeds are an ominous sign. C. The woman is a victim of domestic violence and is being hit in the face by her partner. D. The woman has been using cocaine intranasally.

A (Elevated levels of estrogen cause capillaries to become engorged in the respiratory tract. This may result in edema in the nose, larynx, trachea, and bronchi. This congestion may cause nasal stuffiness and epistaxis. Cardiovascular changes in pregnancy may cause edema in lower extremities. Determining that the woman is a victim of domestic violence and was hit in the face cannot be made on the basis of the sparse facts provided. If the woman had been hit in the face, she most likely would have additional physical findings. Determination of the use of cocaine by the woman cannot be made on the basis of the sparse facts provided.)

The nurse is assessing a pregnant patient who has a positive Hegar sign. The fetal heartbeat is evident by Doppler ultrasound stethoscope, and there is about 5 mm Hg decrease of carbon dioxide partial pressure. The nurse suspects the patient is in which week of gestation? A. Week 10 B. Week 20 C. Week 30 D. Week 35

A (Hegar sign is the softening and compressibility of the lower uterine segment that is seen during weeks 6 to 12 of the gestation period. The fetal heartbeat is detected using ultrasound stethoscope between 8 and 17 weeks. A decrease in the partial pressure of carbon dioxide by 5 mm Hg is seen at week 10 of gestation. Therefore it is most likely that the fetus is in the tenth week of gestation. Fetal heartbeat can be detected by fetal stethoscope after week 17 of pregnancy. Therefore, at weeks 20, 30, and 35 of gestation, Doppler ultrasound stethoscope is not required. Hegar sign is not seen in weeks 20, 30, or 35.)

While reviewing the laboratory reports of a pregnant female, the nurse finds that the patient's urine glucose levels fluctuate. What does the nurse infer from the assessment? The patient has: A. A normal pregnancy. B. Decreased fat absorption. C. Decreased glucose metabolism. D. Sensitive pancreatic β-cells.

A (In pregnant woman, the tubular reabsorption of glucose is impaired causing glucosuria to occur. This urine glucose level can vary from 0 to 20 mg/dL in a pregnant female. Although glucosuria is a normal finding in pregnancy, the possibility of diabetes should be considered. Decrease in fat absorption does not affect the glucose reabsorption in the kidneys but may lead to malnutrition. A decreased rate of metabolism does not affect glomerular filtration process. β-cells of the islets of Langerhans help in the production of insulin; they are not involved in the glucose absorption by the kidneys.)

Appendicitis may be difficult to diagnose in pregnancy because the appendix is: A. Displaced upward and laterally, high and to the right. B. Displaced upward and laterally, high and to the left. C. Deep at McBurney point. D. Displaced downward and laterally, low and to the right.

A (The appendix is displaced high and to the right, beyond McBurney point.)

The mucous plug that forms in the endocervical canal is called the: A. Operculum. B. Funic souffle. C. Leukorrhea. D. Ballottement.

A (The operculum protects against bacterial invasion. Leukorrhea is the mucus that forms the endocervical plug (the operculum). The funic souffle is the sound of blood flow)

A normal uterine activity (UA) pattern in labor is characterized by: A. Contractions every 2 to 5 minutes B. Contractions lasting about 2 minutes C. Contractions about 1 minute apart D. A contraction intensity of about 1000 mm Hg with relaxation at 50 mm Hg

A A. Correct: Contractions normally occur every 2 to 5 minutes and last less than 90 seconds (intensity 800 mm Hg) with about 30 seconds in between (20 mm Hg or less). B. Incorrect: Contractions normally occur every 2 to 5 minutes and last less than 90 seconds (intensity 800 mm Hg) with about 30 seconds in between (20 mm Hg or less). C. Incorrect: Contractions normally occur every 2 to 5 minutes and last less than 90 seconds (intensity 800 mm Hg) with about 30 seconds in between (20 mm Hg or less). D. Incorrect: Contractions normally occur every 2 to 5 minutes and last less than 90 seconds (intensity 800 mm Hg) with about 30 seconds in between (20 mm Hg or less). p. 498

Which deceleration of the FHR would NOT require the nurse to change the maternal position? A. Early decelerations B. Late decelerations C. Variable decelerations D. It is always a good idea to change the woman's position.

A A. Correct: Early decelerations (and accelerations) generally do not need any nursing intervention. B. Incorrect: Late decelerations suggest that the nurse should change the maternal position (lateral); variable decelerations also require a maternal position change (side to side). C. Incorrect: Late decelerations suggest that the nurse should change the maternal position (lateral); variable decelerations also require a maternal position change (side to side). D. Incorrect: Although changing positions throughout labor is recommended, it is not required in response to early decelerations. p. 505

The nurse caring for the laboring woman should understand that early decelerations are caused by: A. Altered fetal cerebral blood flow B. Umbilical cord compression C. Uteroplacental insufficiency D. Spontaneous rupture of membranes

A A. Correct: Early decelerations are the fetus's response to fetal head compression. B. Incorrect: Variable decelerations are associated with umbilical cord compression. C. Incorrect: Late decelerations are associated with uteroplacental insufficiency. D. Incorrect: Spontaneous rupture of membranes has no bearing on the FHR unless the umbilical cord prolapses, which would result in variable or prolonged bradycardia. p. 507

The nurse providing care for the laboring woman should understand that accelerations with fetal movement: A. Are reassuring B. Are caused by umbilical cord compression C. Warrant close observation D. Are caused by uteroplacental insufficiency

A A. Correct: Episodic accelerations in the FHR occur during fetal movement and are indications of fetal well-being. B. Incorrect: Umbilical cord compression results in variable decelerations in the FHR. C. Incorrect: Accelerations in the FHR are an indication of fetal well-being and do not warrant close observation. D. Incorrect: Uteroplacental insufficiency would result in late decelerations in the FHR. p. 504

Fetal tachycardia is most common during: A. Maternal fever B. Umbilical cord prolapse C. Regional anesthesia D. MgSO4 administration

A A. Correct: Fetal tachycardia can be considered an early sign of fetal hypoxemia and can also result from maternal or fetal infection. B. Incorrect: This situation most likely would result in fetal bradycardia, not tachycardia. C. Incorrect: This situation most likely would result in fetal bradycardia, not tachycardia. D. Incorrect: This situation most likely would result in fetal bradycardia, not tachycardia. p. 505

Fetal well-being during labor is assessed by: A. The response of the FHR to uterine contractions (UCs) B. Maternal pain control C. Accelerations in the FHR D. An FHR above 110 beats/min

A A. Correct: Fetal well-being during labor can be measured by the response of the FHR to UCs. In general, reassuring FHR patterns are characterized by an FHR baseline in the range of 110 to 160 beats/min with no periodic changes and a moderate baseline variability, and accelerations with fetal movement. B. Incorrect: Maternal pain control is not the measure used to determine fetal well-being in labor. C. Incorrect: Although FHR accelerations are a reassuring pattern, they are only one component of the criteria by which fetal well-being is assessed. D. Incorrect: Although an FHR above 110 beats/min may be reassuring, it is only one component of the criteria by which fetal well-being is assessed. More information would be needed to determine fetal well-being. p. 498

While evaluating an external monitor tracing of a woman in active labor, the nurse notes that the fetal heart rate (FHR) for five sequential contractions begins to decelerate late in the contraction, with the nadir of the decelerations occurring after the peak of the contraction. The nurse's first priority is to: A. Change the woman's position B. Notify the care provider C. Assist with amnioinfusion D. Insert a scalp electrode

A A. Correct: Late decelerations may be caused by maternal supine hypotension syndrome. They usually are corrected when the woman turns on her side to displace the weight of the gravid uterus from the vena cava. B. Incorrect: If the fetus does not respond to primary nursing interventions for late decelerations, the nurse would continue with subsequent intrauterine resuscitation measures, including notifying the care provider. C. Incorrect: An amnioinfusion may be used to relieve pressure on an umbilical cord that has not prolapsed. The fetal heart rate pattern associated with this situation most likely reveals variable deceleration. D. Incorrect: A fetal scalp electrode would provide accurate data for evaluating the well-being of the fetus; however, this is not a nursing intervention that would alleviate late decelerations, nor is it the nurse's first priority. p. 507

The nurse caring for a laboring woman is aware that maternal cardiac output can be increased by: A. Change in position B. Oxytocin administration C. Regional anesthesia D. Intravenous analgesic

A A. Correct: Maternal supine hypotension syndrome is caused by the weight and pressure of the gravid uterus on the ascending vena cava when the woman is in a supine position. This reduces venous return to the woman's heart, as well as cardiac output, and subsequently reduces her blood pressure. The nurse can encourage the woman to change positions and to avoid the supine position. B. Incorrect: This intervention may reduce maternal cardiac output. C. Incorrect: This intervention may reduce maternal cardiac output. D. Incorrect: This intervention may reduce maternal cardiac output. p. 503

Perinatal nurses are legally responsible for: A. Correctly interpreting FHR patterns, initiating appropriate nursing interventions, and documenting the outcomes B. Greeting the client on arrival, assessing her, and starting an IV line C. Applying the external fetal monitor and notifying the care provider D. Making sure the woman is comfortable

A A. Correct: Nurses who care for women during childbirth are legally responsible for correctly interpreting FHR patterns, initiating appropriate nursing interventions based on those patterns, and documenting the outcomes of those interventions. B. Incorrect: This may be an activity that a nurse performs, but it is not an activity for which the nurse is legally responsible. C. Incorrect: This may be an activity that a nurse performs, but it is not an activity for which the nurse is legally responsible. D. Incorrect: This is one aspect of caring for a woman in labor, but it is not an activity for which the nurse is legally responsible. p. 511

The nurse caring for the woman in labor should understand that decreased variability of the fetal heart rate would be considered benign if caused by: A. A periodic fetal sleep state B. Uterine palpation C. Uterine contractions D. Maternal activity

A A. Correct: Periodic fetal sleep states usually last 20 to 30 minutes. B. Incorrect: Uterine palpations and contractions, as well as maternal activity, might be (probably) benign signs of increased variability. C. Incorrect: Uterine palpations and contractions, as well as maternal activity, might be (probably) benign signs of increased variability. D. Incorrect: Uterine palpations and contractions, as well as maternal activity, might be (probably) benign signs of increased variability. p. 502

When using IA to assess uterine activity, nurses should be aware that: A. The examiner's hand should be placed over the fundus before, during, and after contractions. B. The frequency and duration of contractions is measured in seconds for consistency. C. Contraction intensity is given a judgment number of 1 to 7 by the nurse and client together. D. The resting tone between contractions is described as either placid or turbulent.

A A. Correct: The assessment is done by palpation; duration, frequency, intensity, and resting tone must be assessed. B. Incorrect: The duration of contractions is measured in seconds; the frequency is measured in minutes. C. Incorrect: The intensity of contractions usually is described as mild, moderate, or strong. D. Incorrect: The resting tone usually is characterized as soft or relaxed. p. 500

In assisting with the two factors that have an effect on fetal status, namely pushing and positioning, nurses should: A. Encourage the woman's cooperation in avoiding the supine position B. Advise the woman to avoid the semi-Fowler position C. Encourage the woman to hold her breath and tighten her abdominal muscles to produce a vaginal response D. Instruct the woman to open her mouth and close her glottis, letting air escape after the push

A A. Correct: The woman should maintain a side-lying position. B. Incorrect: The semi-Fowler position is the recommended side-lying position with a lateral tilt to the uterus. C. Incorrect: This is the Valsalva maneuver, which should be avoided. D. Incorrect: Both the mouth and glottis should be open, letting air escape during the push. p. 516

Fetal tachycardia is most common during: A. Maternal fever B. Umbilical cord prolapse C. Regional anesthesia D. MgSO4 administration

A A. Maternal fever Correct: Fetal tachycardia can be considered an early sign of fetal hypoxemia and can also result from maternal or fetal infection. B. Umbilical cord prolapse Incorrect: This situation most likely would result in fetal bradycardia, not tachycardia. C. Regional anesthesia Incorrect: This situation most likely would result in fetal bradycardia, not tachycardia. D. MgSO4 administration Incorrect: This situation most likely would result in fetal bradycardia, not tachycardia. p. 505

Maddy, a G3 P1 woman, gave birth 12 hours ago to a 9 lb. 13 oz. daughter. She experiences severe cramps with breastfeeding. The perinatal nurse best describes this condition as: a. Afterpains b. Uterine hypertonia c. Bladder hypertonia d. Rectus abdominis diastasis

A Afterpains (afterbirth pains) are intermittent uterine contractions that occur during the process of involution. Afterpains are more pronounced in patients with decreased uterine tone due to overdistension, which is associated with multiparity and macrosomia. Patients often describe the sensation as a discomfort similar to menstrual cramps. Afterpains are also related to the increase of oxytocin released in response to infant suckling.

The perinatal nurse notes a rapid decrease in the fetal heart rate that does not recover immediately following an amniotomy. The most likely cause of this obstetrical emergency is: a. Prolapsed umbilical cord b. Vasa previa c. Oligohydramnios d. Placental abruption

A Amniotomy is the artificial rupture of membranes (AROM) to induce or augment labor. This is a common procedure seen in obstetrics. Risks associated with amniotomy include umbilical cord prolapse when the presenting part is not engaged. Vasa previa or rupture of fetal vessels unsupported by the placenta is a very rare situation and usually results in rapid fetal exsanguination in the presence of bloody fluid seen following AROM.

Which statement regarding the postpartum uterus is correct? a. After 2 weeks postpartum, it should be abdominally nonpalpable. b. At the end of the third stage of labor, the postpartum uterus weighs approximately 500 g. c. Postpartum uterus returns to its original (prepregnancy) size by 6 weeks postpartum. d. After 2 weeks postpartum, it weighs 100 g.

A By the end of the third stage of labor, the uterus weighs 1000g, 350 grams 2 weeks after labor and returns to its nonpregnant location by 6 weeks after birth when it will weigh 60 to 80 grams.

Tanya, a 30-year-old woman, is being prepared for a planned cesarean birth. The perinatal nurse assists the anesthesiologist with the spinal block and then positions Tanya in a supine position. Tanya's blood pressure drops to 90/52, and there is a decrease in the fetal heart rate to 110 bpm. The perinatal nurse's best response is to: a. Place Tanya in a left lateral tilt b. Discontinue Tanya's intravenous administration. c. Have naloxone (Narcan) ready for administration. d. Have epinephrine ready for administration.

A Reposition the woman after epidural or spinal anesthesia in a supine position with a left lateral tilt to decrease the pressure from the uterus on the inferior vena cava and to maintain placental perfusion.

Which client is at greatest risk for early PPH? a. Woman with severe preeclampsia on magnesium sulfate whose labor is being induced b. Primigravida in spontaneous labor with preterm twins c. Multiparous woman (G 3, P 2-0-0-2) with an 8-hour labor d. Primiparous woman (G 2, P 1-0-0-1) being prepared for an emergency cesarean birth for fetal distress

A The effects of magnesium sulfate puts this patient at greater risk for PPH than the other scenarios.

A woman gave birth to a 3200 g baby girl with an estimated gestational age of 40 weeks. The baby is 1 hour of age. In preparation for administration of Vitamin K to the infant, the nurse will explain to the parents that an injection of this medication: a. Influences the activation of coagulation factors to prevent delayed clotting and hemorrhagic disease b. Prevents high levels of unconjugated bilirubin in the newborn's blood c. Prevents the excessive loss of RBCs d. Aids the liver in regulation of blood glucose

A Vitamin K (phytonadione) influences the activation of coagulation factors II, VII, IX, and X. After birth, the neonate experiences a decrease in Vitamin K and is at risk for delayed clotting and for hemorrhage. An injection of Vitamin K is given as a prophylaxis to decreased the risk of bleeding.

A couple who has sought infertility counseling has been told that the man's sperm count is very low. The nurse advises the couple that spermatogenesis is impaired when which of the following occur? a. The testes are overheated. b. The vas deferens is ligated. c. The prostate gland is enlarged. d. alcohol use

A see p 204 under nonmedical therapy

23 y/o Amy delivered a baby 2 months ago and presents to the clinic with complaint of stool coming out of her vagina for the past week. The nurse suspects recto-vaginal fistula. What is the next most appropriate question the nurse should ask Amy? a. "Did you deliver your baby vaginally?" b. "How long has this been going on?" c. "Are you sure it is stool?" d. "How big was your baby?"

A this question starts the nurse's assessment for potential trauma during childbirth that increased Amy's risk for the fistula.

A major nursing intervention for an infant born with myelomeningocele is to: A. Protect the sac from injury B. Prepare the parents for the child's paralysis from the waist down C. Prepare the parents for closure of the sac at around 2 years of age D. Assess for cyanosis

A A. Correct: A major preoperative nursing intervention for a neonate with a myelomeningocele is protection of the protruding sac from injury to prevent its rupture and the resultant risk of CNS infection. B. Incorrect: The long-term prognosis in an affected infant can be determined to a large extent at birth, with the degree of neurologic dysfunction related to the level of the lesion, which determines the nerves involved. C. Incorrect: A myelomeningocele should be surgically closed within 24 hours. D. Incorrect: Although the nurse would assess for multiple potential problems in this infant, the major nursing intervention would be to protect the sac from injury. p. 1036

The abuse of which of the following substances during pregnancy is the leading cause of mental retardation in the United States? A. Alcohol B. Tobacco C. Marijuana D. Heroin

A A. Correct: Alcohol abuse during pregnancy is recognized as one of the leading causes of mental retardation in the United States. B. Incorrect: Alcohol abuse during pregnancy is recognized as one of the leading causes of mental retardation in the United States. C. Incorrect: Alcohol abuse during pregnancy is recognized as one of the leading causes of mental retardation in the United States. D. Incorrect: Alcohol abuse during pregnancy is recognized as one of the leading causes of mental retardation in the United States. p. 1013

Which TORCH infection could be contracted by the infant because the mother owned a cat? A. Toxoplasmosis B. Varicella zoster (chicken pox) C. Parvovirus B19 D. Rubella

A A. Correct: Cats that eat birds infected with the Toxoplasma gondii protozoan excrete infective oocysts. Humans (including pregnant women) can become infected if they fail to wash their hands after cleaning the litter box. The infection is passed through the placenta. B. Incorrect: Cats that eat birds infected with the Toxoplasma gondii protozoan excrete infective oocysts. Humans (including pregnant women) can become infected if they fail to wash their hands after cleaning the litter box. The infection is passed through the placenta. C. Incorrect: Cats that eat birds infected with the Toxoplasma gondii protozoan excrete infective oocysts. Humans (including pregnant women) can become infected if they fail to wash their hands after cleaning the litter box. The infection is passed through the placenta. D. Incorrect: Cats that eat birds infected with the Toxoplasma gondii protozoan excrete infective oocysts. Humans (including pregnant women) can become infected if they fail to wash their hands after cleaning the litter box. The infection is passed through the placenta. p. 1004

With regard to injuries to the infant's plexus during labor and birth, nurses should be aware that: A. If the nerves are stretched with no avulsion, they should recover completely in 3 to 6 months. B. Erb palsy is damage to the lower plexus. C. Parents of children with brachial palsy are taught to pick up the child from under the axillae. D. Breastfeeding is not recommended for infants with facial nerve paralysis until the condition resolves.

A A. Correct: However, if the ganglia are disconnected completely from the spinal cord, the damage is permanent. B. Incorrect: Erb palsy is damage to the upper plexus and is less serious than brachial palsy. C. Incorrect: Parents of children with brachial palsy are taught to avoid picking up the child under the axillae or by pulling on the arms. D. Incorrect: Breastfeeding is not contraindicated, but both the mother and infant will need help from the nurse at the start. p. 994

Which infant would be more likely to have Rh incompatibility? A. Infant of an Rh-negative mother and a father who is Rh positive and homozygous for the Rh factor B. Infant who is Rh negative and whose mother is Rh negative C. Infant of an Rh-negative mother and a father who is Rh positive and heterozygous for the Rh factor D. Infant who is Rh positive and whose mother is Rh positive

A A. Correct: If the mother is Rh negative and the father is Rh positive and homozygous for the Rh factor, all the offspring will be Rh positive. Only Rh-positive offspring of an Rh-negative mother are at risk for Rh incompatibility. B. Incorrect: Only the Rh-positive offspring of an Rh-negative mother are at risk. C. Incorrect: If the mother is Rh negative and the father is Rh positive and heterozygous for the factor, there is a 50% chance that each infant born of the union will be Rh positive and a 50% chance that each will be born Rh negative. D. Incorrect: Only the Rh-positive offspring of an Rh-negative mother are at risk. p. 1026

A 3.8-kg infant was delivered vaginally at 39 weeks after a 30-minute second stage. There was a nuchal cord. After birth, the infant is noted to have petechiae over the face and upper back. Information given to the infant's parents should be based on the knowledge that petechiae: A. Are benign if they disappear within 48 hours of birth B. Result from increased blood volume C. Should always be further investigated D. Usually occur with forceps delivery

A A. Correct: Petechiae, or pinpoint hemorrhagic areas, acquired during birth may extend over the upper portion of the trunk and face. These lesions are benign if they disappear within 2 days of birth and no new lesions appear. B. Incorrect: Petechiae may result from decreased platelet formation. C. Incorrect: In this situation, the presence of petechiae is most likely a soft-tissue injury resulting from the nuchal cord at birth. Unless they do not dissipate in 2 days, there is no reason to alarm the family. D. Incorrect: Petechiae usually occur with a breech presentation vaginal birth. p. 993

A careful review of the literature on the various recreational and illicit drugs reveals that: A. More, longer term studies are needed to assess the lasting effects on infants when mothers have taken or are taking illegal drugs. B. Heroin and methadone cross the placenta; marijuana, cocaine, and PCP do not. C. Mothers should get off heroin (detox) any time they can during pregnancy. D. Methadone withdrawal for infants is less severe and shorter than heroin withdrawal.

A A. Correct: Studies on the effects of marijuana and cocaine use by mothers are somewhat contradictory. More, longer range studies are needed. B. Incorrect: Just about all of these drugs cross the placenta, including marijuana, cocaine, and PCP. C. Incorrect: Drug withdrawal is accompanied by fetal withdrawal, which can lead to fetal death. Therefore, detoxification from heroin is not recommended, particularly later, in pregnancy. D. Incorrect: Methadone withdrawal is more severe and more prolonged than heroin withdrawal. p. 1015

With regard to congenital anomalies of the cardiovascular and respiratory systems, nurses should be aware that: A. Cardiac disease may be manifested by respiratory signs and symptoms. B. Screening for congenital anomalies of the respiratory system need only be done for infants having respiratory distress. C. Choanal atresia can be corrected by a suction catheter. D. Congenital diaphragmatic hernias are diagnosed and treated after birth.

A A. Correct: The cardiac and respiratory systems function together. B. Incorrect: Screening for congenital respiratory system anomalies is necessary even for infants who appear normal at birth. C. Incorrect: Choanal atresia requires emergency surgery. D. Incorrect: Congenital diaphragmatic hernias are discovered prenatally on ultrasound. p. 1033

A pregnant woman presents in labor at term, having had no prenatal care. After birth, her infant is noted to be small for gestational age with small eyes and a thin upper lip. The infant also is microcephalic. Based on her infant's physical findings, this woman should be questioned about her use of which substance during pregnancy? A. Alcohol B. Cocaine C. Heroin D. Marijuana

A A. Correct: The description of the infant suggests fetal alcohol syndrome, which is consistent with maternal alcohol consumption during pregnancy. B. Incorrect: Fetal brain, kidney, and urogenital system malformations have been associated with maternal cocaine ingestions. C. Incorrect: Heroin use in pregnancy frequently results in IUGR. The infant may have a shrill cry and sleep cycle disturbances and may present with poor feeding, tachypnea, vomiting, diarrhea, hypothermia or hyperthermia, and sweating. D. Incorrect: Studies have found a higher incidence of meconium staining in infants born of mothers who used marijuana during pregnancy. p. 1013

During a prenatal examination, the woman reports having two cats at home. The nurse informs her that she should not be cleaning the litter box while she is pregnant. When the woman questions the nurse as to why, the nurse's best response would be: A. "Your cats could be carrying toxoplasmosis. This is a zoonotic parasite that can infect you and have severe effects on your unborn child." B. "You and your baby can be exposed to the HIV virus in your cats' feces." C. "It's just gross. You should make your husband clean the litter boxes." D. "Cat feces are known to carry E. coli, which can cause a severe infection in both you and your baby."

A A. Correct: Toxoplasmosis is a multisystem disease caused by the protozoal Toxoplasma gondii parasite, commonly found in cats, dogs, pigs, sheep, and cattle. About 30% of women who contract toxoplasmosis during gestation transmit the disease to their offspring. Clinical features ascribed to toxoplasmosis include hydrocephalus or microcephaly, chorioretinitis, seizures, or cerebral calcifications. B. Incorrect: HIV is not transmitted by cats. C. Incorrect: Although this may be a valid statement, it is not appropriate, does not answer the client's question, and is not the nurse's best response. D. Incorrect: E. coli is found in normal human fecal flora. It is not transmitted by cats. p. 1004

The most important nursing action in preventing neonatal infection is: A. Good handwashing B. Isolation of infected infants C. Separate gown technique D. Standard Precautions

A A. Correct: Virtually all controlled clinical trials have demonstrated that effective handwashing is responsible for the prevention of nosocomial infection in nursery units. B. Incorrect: Measures to be taken include Standard Precautions, careful and thorough cleaning, frequent replacement of used equipment, and disposal of excrement and linens in an appropriate manner. Overcrowding must be avoided in nurseries. However, the most important nursing action for preventing neonatal infection is effective handwashing. C. Incorrect: Measures to be taken include Standard Precautions, careful and thorough cleaning, frequent replacement of used equipment, and disposal of excrement and linens in an appropriate manner. Overcrowding must be avoided in nurseries. However, the most important nursing action for preventing neonatal infection is effective handwashing. D. Incorrect: Measures to be taken include Standard Precautions, careful and thorough cleaning, frequent replacement of used equipment, and disposal of excrement and linens in an appropriate manner. Overcrowding must be avoided in nurseries. However, the most important nursing action for preventing neonatal infection is effective handwashing. p. 1002

16. Which explanation will assist the parents in their decision on whether they should circumcise their son? a. The circumcision procedure has pros and cons during the prenatal period. b. American Academy of Pediatrics (AAP) recommends that all male newborns be routinely circumcised. c. Circumcision is rarely painful, and any discomfort can be managed without medication. d. The infant will likely be alert and hungry shortly after the procedure.

A (Parents need to make an informed choice regarding newborn circumcision, based on the most current evidence and recommendations. Health care providers and nurses who care for childbearing families should provide factual, unbiased information regarding circumcision and give parents opportunities to discuss the risks and benefits of the procedure. The AAP and other professional organizations note the benefits but stop short of recommending routine circumcision. Circumcision is painful and must be managed with environmental, nonpharmacologic, and pharmacologic measures. After the procedure, the infant may be fussy for several hours, or he may be sleepy and difficult to awaken for feeding.)

21. A 3.8-kg infant was vaginally delivered at 39 weeks of gestation after a 30-minute second stage. A nuchal cord occurred. After the birth, the infant is noted to have petechiae over the face and upper back. Based on the nurses knowledge, which information regarding petechiae should be shared with the parents? a. Petechiae (pinpoint hemorrhagic areas) are benign if they disappear within 48 hours of childbirth. b.These hemorrhagic areas may result from increased blood volume. c.Petechiae should always be further investigated. d. Petechiae usually occur with a forceps delivery.

A (Petechiae that are acquired during birth may extend over the upper portion of the trunk and face. These lesions are benign if they disappear within 2 days of birth and no new lesions appear. Petechiae may result from decreased platelet formation. In this infant, the presence of petechiae is more likely a soft-tissue injury resulting from the nuchal cord at birth. Unless the lesions do not dissipate in 2 days, no reason exists to alarm the family. Petechiae usually occur with a breech presentation vaginal birth.)

28. Nursing follow-up care often includes home visits for the new mother and her infant. Which information related to home visits is correct? a. Ideally, the visit is scheduled within 72 hours after discharge. b. Home visits are available in all areas. c. Visits are completed within a 30-minute time frame. d. Blood draws are not a part of the home visit.

A (The home visit is ideally scheduled within 72 hours after discharge. This timing allows early assessment and intervention for problems with feedings, jaundice, newborn adaptation, and maternal-infant interaction. Because of geographic distances, home visits are not available in all locales. Visits are usually 60 to 90 minutes in length to allow enough time for assessment and teaching. When jaundice is found, the nurse can discuss the implications and check the transcutaneous bilirubin level or draw blood for testing.)

The nurse providing care for the laboring woman should understand that amnioinfusion is used to treat: a. Variable decelerations. c. Fetal bradycardia. b. Late decelerations. d. Fetal tachycardia.

A Amnioinfusion is used during labor either to dilute meconium-stained amniotic fluid or to supplement the amount of amniotic fluid to reduce the severity of variable decelerations caused by cord compression. Amnioinfusion has no bearing on late decelerations, fetal bradycardia, or fetal tachycardia alterations in fetal heart rate (FHR) tracings.

A normal uterine activity pattern in labor is characterized by: a. Contractions every 2 to 5 minutes. b. Contractions lasting about 2 minutes. c. Contractions about 1 minute apart. d. A contraction intensity of about 1000 mm Hg with relaxation at 50 mm Hg.

A Contractions normally occur every 2 to 5 minutes and last less than 90 seconds (intensity 800 mm Hg) with about 30 seconds in between (20 mm Hg or less).

Which deceleration of the fetal heart rate would not require the nurse to change the maternal position? a. Early decelerations b. Late decelerations c. Variable decelerations d. It is always a good idea to change the woman's position.

A Early decelerations (and accelerations) generally do not need any nursing intervention. Late decelerations suggest that the nurse should change the maternal position (lateral); variable decelerations also require a maternal position change (side to side). Although changing positions throughout labor is recommended, it is not required in response to early decelerations.

The nurse caring for the laboring woman should understand that early decelerations are caused by: a. Altered fetal cerebral blood flow. c. Uteroplacental insufficiency. b. Umbilical cord compression. d. Spontaneous rupture of membranes.

A Early decelerations are the fetus's response to fetal head compression. Variable decelerations are associated with umbilical cord compression. Late decelerations are associated with uteroplacental insufficiency. Spontaneous rupture of membranes has no bearing on the fetal heart rate unless the umbilical cord prolapses, which would result in variable or prolonged bradycardia.

The nurse providing care for the laboring woman comprehends that accelerations with fetal movement: a. Are reassuring. b. Are caused by umbilical cord compression. c. Warrant close observation. d. Are caused by uteroplacental insufficiency.

A Episodic accelerations in the fetal heart rate (FHR) occur during fetal movement and are indications of fetal well-being. Umbilical cord compression results in variable decelerations in the FHR. Accelerations in the FHR are an indication of fetal well-being and do not warrant close observation. Uteroplacental insufficiency would result in late decelerations in the FHR.

Fetal well-being during labor is assessed by: a. The response of the fetal heart rate (FHR) to uterine contractions (UCs). b. Maternal pain control. c. Accelerations in the FHR. d. An FHR above 110 beats/min.

A Fetal well-being during labor can be measured by the response of the FHR to UCs. In general, reassuring FHR patterns are characterized by an FHR baseline in the range of 110 to 160 beats/min with no periodic changes, a moderate baseline variability, and accelerations with fetal movement. Maternal pain control is not the measure used to determine fetal well-being in labor. Although FHR accelerations are a reassuring pattern, they are only one component of the criteria by which fetal well-being is assessed. Although an FHR above 110 beats/min may be reassuring, it is only one component of the criteria by which fetal well-being is assessed. More information would be needed to determine fetal well-being.

Which maternal condition is considered a contraindication for the application of internal monitoring devices? a. Unruptured membranes b. External monitors in current use c. Cervix dilated to 4 cm d. Fetus with a known heart defect

A In order to apply internal monitoring devices, the membranes must be ruptured. Cervical dilation of 4 cm permits the insertion of fetal scalp electrodes and intrauterine catheter. The external monitor can be discontinued after the internal ones are applied. A compromised fetus should be monitored with the most accurate monitoring devices.

While evaluating an external monitor tracing of a woman in active labor, the nurse notes that the fetal heart rate (FHR) for five sequential contractions begins to decelerate late in the contraction, with the nadir of the decelerations occurring after the peak of the contraction. The nurse's first priority is to: a. Change the woman's position. c. Assist with amnioinfusion. b. Notify the care provider. d. Insert a scalp electrode.

A Late decelerations may be caused by maternal supine hypotension syndrome. They usually are corrected when the woman turns on her side to displace the weight of the gravid uterus from the vena cava. If the fetus does not respond to primary nursing interventions for late decelerations, the nurse would continue with subsequent intrauterine resuscitation measures, including notifying the care provider. An amnioinfusion may be used to relieve pressure on an umbilical cord that has not prolapsed. The FHR pattern associated with this situation most likely reveals variable deceleration. A fetal scalp electrode would provide accurate data for evaluating the well-being of the fetus; however, this is not a nursing intervention that would alleviate late decelerations, nor is it the nurse's first priority.

The nurse caring for a laboring woman is aware that maternal cardiac output can be increased by: a. Change in position. c. Regional anesthesia. b. Oxytocin administration. d. Intravenous analgesic.

A Maternal supine hypotension syndrome is caused by the weight and pressure of the gravid uterus on the ascending vena cava when the woman is in a supine position. This reduces venous return to the woman's heart, as well as cardiac output, and subsequently reduces her blood pressure. The nurse can encourage the woman to change positions and avoid the supine position. Oxytocin administration, regional anesthesia, and intravenous analgesic may reduce maternal cardiac output.

Perinatal nurses are legally responsible for: a. Correctly interpreting fetal heart rate (FHR) patterns, initiating appropriate nursing interventions, and documenting the outcomes. b. Greeting the client on arrival, assessing her, and starting an intravenous line. c. Applying the external fetal monitor and notifying the care provider. d. Making sure that the woman is comfortable.

A Nurses who care for women during childbirth are legally responsible for correctly interpreting FHR patterns, initiating appropriate nursing interventions based on those patterns, and documenting the outcomes of those interventions. Greeting the client, assessing her, and starting an IV; applying the external fetal monitor and notifying the care provider; and making sure the woman is comfortable may be activities that a nurse performs, but they are not activities for which the nurse is legally responsible.

When using intermittent auscultation (IA) to assess uterine activity, the nurse should be cognizant that: a. The examiner's hand should be placed over the fundus before, during, and after contractions. b. The frequency and duration of contractions is measured in seconds for consistency. c. Contraction intensity is given a judgment number of 1 to 7 by the nurse and client together. d. The resting tone between contractions is described as either placid or turbulent.

A The assessment is done by palpation; duration, frequency, intensity, and resting tone must be assessed. The duration of contractions is measured in seconds; the frequency is measured in minutes. The intensity of contractions usually is described as mild, moderate, or strong. The resting tone usually is characterized as soft or relaxed.

The nurse knows that proper placement of the tocotransducer for electronic fetal monitoring is located: a. Over the uterine fundus. c. Inside the uterus. b. On the fetal scalp. d. Over the mother's lower abdomen.

A The tocotransducer monitors uterine activity and should be placed over the fundus, where the most intensive uterine contractions occur. The tocotransducer is for external use.

In assisting with the two factors that have an effect on fetal status (i.e., pushing and positioning), nurses should: a. Encourage the woman's cooperation in avoiding the supine position. b. Advise the woman to avoid the semi-Fowler position. c. Encourage the woman to hold her breath and tighten her abdominal muscles to produce a vaginal response. d. Instruct the woman to open her mouth and close her glottis, letting air escape after the push.

A The woman should maintain a side-lying position. The semi-Fowler position is the recommended side-lying position with a lateral tilt to the uterus. The Valsalva maneuver, which encourages the woman to hold her breath and tighten her abdominal muscles, should be avoided. Both the mouth and glottis should be open, letting air escape during the push.

10. A mother is changing the diaper of her newborn son and notices that his scrotum appears large and swollen. The client is concerned. What is the best response from the nurse? a. A large scrotum and swelling indicate a hydrocele, which is a common finding in male newborns. b. I don't know, but I'm sure it is nothing. c. Your baby might have testicular cancer. d. Your babys urine is backing up into his scrotum.

A (Explaining what a hydrocele is and its characteristics is the most appropriate response by the nurse. The swelling usually decreases without intervention. Telling the mother that the condition is nothing important is inappropriate and does not address the mothers concern. Furthermore, if the nurse is unaware of any abnormal-appearing condition, then she should seek assistance from additional resources. Telling the mother that her newborn might have testicular cancer is inaccurate, inappropriate, and could cause the new mother undue worry. Urine will not back up into the scrotum if the infant has a hydrocele. Any nurse caring for the normal newborn should understand basic anatomy.)

5. The nurse is using the New Ballard Scale to determine the gestational age of a newborn. Which assessment finding is consistent with a gestational age of 40 weeks? a. Flexed posture b. Abundant lanugo c. Smooth, pink skin with visible veins d. Faint red marks on the soles of the feet

A (Term infants typically have a flexed posture. Abundant lanugo; smooth, pink skin with visible veins; and faint red marks are usually observed on preterm infants.)

25. The nurse should be cognizant of which important statement regarding care of the umbilical cord? a. The stump can become easily infected. b. If bleeding occurs from the vessels of the cord, then the nurse should immediately call for assistance. c. The cord clamp is removed at cord separation. d. The average cord separation time is 5 to 7 days.

A (The cord stump is an excellent medium for bacterial growth. The nurse should first check the clamp (or tie) and apply a second one. If bleeding occurs and does not stop, then the nurse should call for assistance. The cord clamp is removed after 24 hours when it is dry. The average cord separation time is 10 to 14 days.)

17. The most serious complication of an infant heelstick is necrotizing osteochondritis resulting from lancet penetration of the bone. What approach should the nurse take when performing the test to prevent this complication? a. Lancet should penetrate at the outer aspect of the heel. b. Lancet should penetrate the walking surface of the heel. c. Lancet should penetrate the ball of the foot. d. Lancet should penetrate the area just below the fifth toe.

A (The stick should be made at the outer aspect of the heel and should penetrate no deeper than 2.4 mm. Repeated trauma to the walking surface of the heel can cause fibrosis and scarring that can lead to problems with walking later in life. The ball of the foot and the area below the fifth toe are inappropriate sites for a heelstick.)

8. A woman gave birth to a 7-pound, 6-ounce infant girl 1 hour ago. The birth was vaginal and the estimated blood loss (EBL) was 1500 ml. When evaluating the womans vital signs, which finding would be of greatest concern to the nurse? a. Temperature 37.9 C, heart rate 120 beats per minute (bpm), respirations 20 breaths per minute, and blood pressure 90/50 mm Hg b. Temperature 37.4 C, heart rate 88 bpm, respirations 36 breaths per minute, and blood pressure 126/68 mm Hg c. Temperature 38 C, heart rate 80 bpm, respirations 16 breaths per minute, and blood pressure 110/80 mm Hg d. Temperature 36.8 C, heart rate 60 bpm, respirations 18 breaths per minute, and blood pressure 140/90 mm Hg

A (An EBL of 1500 ml with tachycardia and hypotension suggests hypovolemia caused by excessive blood loss. Temperature 37.4 C, heart rate 88 bpm, respirations 36 breaths per minute, and blood pressure 126/68 mm Hg are normal vital signs except for an increased respiratory rate, which may be secondary to pain from the birth. Temperature 38 C, heart rate 80 bpm, respirations 16 breaths per minute, and blood pressure 110/80 mm Hg are normal vital signs except for the temperature, which may increase to 38 C during the first 24 hours as a result of the dehydrating effects of labor. Temperature 36.8 C, heart rate 60 bpm, respirations 18 breaths per minute, and blood pressure 140/90 mm Hg are normal vital signs, although the blood pressure is slightly elevated, which may be attributable to the use of oxytocic medications.)

10. After delivery, excess hypertrophied tissue in the uterus undergoes a period of self-destruction. What is the correct term for this process? a. Autolysis b. Subinvolution c. Afterpains d. Diastasis

A (Autolysis is caused by a decrease in hormone levels. Subinvolution is failure of the uterus to return to a nonpregnant state. Afterpains are caused by uterine cramps 2 to 3 days after birth. Diastasis refers to the separation of muscles.)

9. A nurse is discussing the storage of breast milk with a mother whose infant is preterm and in the special care nursery. Which statement indicates that the mother requires additional teaching? a.I can store my breast milk in the refrigerator for 3 months. b.I can store my breast milk in the freezer for 3 months. c.I can store my breast milk at room temperature for 4 hours. d.I can store my breast milk in the refrigerator for 3 to 5 days.

A (Breast milk for the hospitalized infant can be stored in the refrigerator for only 8 days, not for 3 months. Breast milk can be stored in the freezer for 3 months, in a deep freezer for 6 months, or at room temperature for 4 hours. Human milk for the healthy or preterm hospitalized infant can be kept in the refrigerator for up to 8 days or in the freezer for up to 3 months, but only for 4 hours or less at room temperature.)

10. A new mother asks the nurse what the experts say about the best way to feed her infant. Which recommendation of the American Academy of Pediatrics (AAP) regarding infant nutrition should be shared with this client? a.Infants should be given only human milk for the first 6 months of life. b.Infants fed on formula should be started on solid food sooner than breastfed infants. c.If infants are weaned from breast milk before 12 months, then they should receive cows milk, not formula. d.After 6 months, mothers should shift from breast milk to cows milk.

A (Breastfeeding and human milk should also be the sole source of milk for the first 12 months, not for only the first 6 months. Infants should be started on solids when they are ready, usually at 6 months, whether they start on formula or breast milk. If infants are weaned from breast milk before 12 months, then they should receive iron-fortified formula, not cows milk.)

11. Which statement is the best rationale for recommending formula over breastfeeding? a.Mother has a medical condition or is taking drugs that could be passed along to the infant via breast milk. b.Mother lacks confidence in her ability to breastfeed. c.Other family members or care providers also need to feed the baby. d.Mother sees bottle feeding as more convenient.

A (Breastfeeding is contraindicated when mothers have certain viruses, tuberculosis, are undergoing chemotherapy, or are using or abusing drugs. Some women lack confidence in their ability to produce breast milk of adequate quantity or quality. The key to encouraging these mothers to breastfeed is anticipatory guidance beginning as early as possible during the pregnancy. A major barrier for many women is the influence of family and friends. She may view formula feeding as a way to ensure that the father and other family members can participate. Each encounter with the family is an opportunity for the nurse to educate, dispel myths, and clarify information regarding the benefits of breastfeeding. Many women see bottle feeding as more convenient and less embarrassing than breastfeeding. They may also see breastfeeding as incompatible with an active social life. Although modesty issues related to feeding the infant in public may exist, these concerns are not legitimate reasons to formula-feed an infant. Often, the decision to formula feed rather than breastfeed is made without complete information regarding the benefits of breastfeeding.)

29. A new mother asks whether she should feed her newborn colostrum, because it is not real milk. What is the nurses most appropriate answer? a.Colostrum is high in antibodies, protein, vitamins, and minerals. b.Colostrum is lower in calories than milk and should be supplemented by formula. c.Giving colostrum is important in helping the mother learn how to breastfeed before she goes home. d.Colostrum is unnecessary for newborns.

A (Colostrum is important because it has high levels of the nutrients needed by the neonate and helps protect against infection. Supplementation is not necessary and will decrease stimulation to the breast and decrease the production of milk. It is important for the mother to feel comfortable in this role before discharge; however, the importance of the colostrum to the infant is the top priority. Colostrum provides immunities and enzymes necessary to cleanse the gastrointestinal system, among other things.)

2. A pregnant woman wants to breastfeed her infant; however, her husband is not convinced that there are any scientific reasons to do so. The nurse can give the couple printed information comparing breastfeeding and bottle feeding. Which statement regarding bottle feeding using commercially prepared infant formulas might influence their choice? a.Bottle feeding using a commercially prepared formula increases the risk that the infant will develop allergies. b.Bottle feeding helps the infant sleep through the night. c.Commercially prepared formula ensures that the infant is getting iron in a form that is easily absorbed. d.Bottle feeding requires that multivitamin supplements be given to the infant.

A (Exposure to cows milk poses a risk of developing allergies, eczema, and asthma. Newborns should be fed during the night, regardless of the feeding method. Iron is better absorbed from breast milk than from formula. Commercial formulas are designed to meet the nutritional needs of the infant and to resemble breast milk. No supplements are necessary.)

13. The nurse should be cognizant of which statement regarding the unique qualities of human breast milk? a.Frequent feedings during predictable growth spurts stimulate increased milk production. b.Milk of preterm mothers is the same as the milk of mothers who gave birth at term. c.Milk at the beginning of the feeding is the same as the milk at the end of the feeding. d.Colostrum is an early, less concentrated, less rich version of mature milk.

A (Growth spurts (at 10 days, 3 weeks, 6 weeks, and 3 months) usually last 24 to 48 hours, after which the infants resume normal feeding. The milk of mothers of preterm infants is different from that of mothers of full-term infants to meet the needs of these newborns. Milk changes composition during feeding. The fat content of the milk increases as the infant feeds. Colostrum precedes mature milk and is more concentrated and richer in proteins and minerals (but not fat).)

7. Parents have been asked by the neonatologist to provide breast milk for their newborn son, who was born prematurely at 32 weeks of gestation. The nurse who instructs them regarding pumping, storing, and transporting the milk needs to assess their knowledge of lactation. Which statement is valid? a.Premature infants more easily digest breast milk than formula. b.A glass of wine just before pumping will help reduce stress and anxiety. c.The mother should only pump as much milk as the infant can drink. d.The mother should pump every 2 to 3 hours, including during the night.

A (Human milk is the ideal food for preterm infants, with benefits that are unique, in addition to those benefits received by full-term, healthy infants. Greater physiologic stability occurs with breastfeeding, compared with formula feeding. Consumption of alcohol during lactation is approached with caution. Excessive amounts can have serious effects on the infant and can adversely affect the mothers milk ejection reflex. To establish an optimal milk supply, the most appropriate instruction for the mother should be to pump 8 to 10 times a day for 10 to 15 minutes on each breast.)

14. A nurse providing couplet care should understand the issue of nipple confusion. In which situation might this condition occur? a.Breastfeeding babies receive supplementary bottle feedings. b.Baby is too abruptly weaned. c.Pacifiers are used before breastfeeding is established. d.Twins are breastfed together.

A (Nipple confusion can result when babies go back and forth between bottles and breasts, especially before breastfeeding is established in 3 to 4 weeks; bottle feeding and breastfeeding require different skills. Abrupt weaning can be distressing to the mother and/or baby but should not lead to nipple confusion. Pacifiers used before breastfeeding is established can be disruptive but do not lead to nipple confusion. Breastfeeding twins require some logistical adaptations but should not lead to nipple confusion.)

1. A woman gave birth to an infant boy 10 hours ago. Where does the nurse expect to locate this womans fundus? a. 1 centimeter above the umbilicus b. 2 centimeters below the umbilicus c. Midway between the umbilicus and the symphysis pubis d. Nonpalpable abdominally

A (The fundus descends approximately 1 to 2 cm every 24 hours. Within 12 hours after delivery the fundus may be approximately 1 cm above the umbilicus. By the sixth postpartum week the fundus is normally halfway between the symphysis pubis and the umbilicus. The fundus should be easily palpated using the maternal umbilicus as a reference point.)

15. Several delivery changes in the integumentary system that appear during pregnancy disappear after birth, although not always completely. What change is almost certain to be completely reversed? a. Nail brittleness b. Darker pigmentation of the areolae and linea nigra c. Striae gravidarum on the breasts, abdomen, and thighs d. Spider nevi

A (The nails return to their prepregnancy consistency and strength. Some women have permanent darker pigmentation of the areolae and linea nigra. Striae gravidarum (stretch marks) usually do not completely disappear. For some women, spider nevi persist indefinitely.)

During labor a fetus with an average heart rate of 135 beats/min over a 10-minute period would be considered to have:

A normal baseline heart rate.

A woman is in for a routine prenatal checkup. You are assessing her urine for proteinuria. Which findings are considered normal (Select all that apply)? A. Dipstick assessment of trace to +1 B. <300 mg/24 hours C. Dipstick assessment of +2 D. >300 mg/24 hours

A, B (Small amounts of protein in the urine are acceptable during pregnancy. The presence of protein in greater amounts may indicate renal problems. A dipstick assessment of +2 and >300 mg/24 hours are excessive amounts of protein in the urine and should be evaluated further.)

A couple is undergoing an infertility workup. The semen analysis indicates a decreased number of sperm and immature sperm. Which of the following factors can have a potential effect on sperm maturity? (Select all that apply.) a. The man rides a bike to and from work each day. b. The man takes a calcium channel blocker for the treatment of hypertension c. The man drinks 6 cups of coffee a day. d. The man was treated for prostatitis 12 months ago and has been symptom free since treatment.

A, B The daily riding of a bike can be the cause of prolonged heat exposure to the testicles. Prolonged heat exposure is a gonadotoxin. A number of medications, such as calcium channel blockers, can have an effect on sperm production. Coffee has not been associated with low sperm counts. Prostatitis or other infections within the last 3 months may have an effect on the sperm analysis. This man's episode of prostatitis was 12 months prior.

Reports have linked third trimester use of selective serotonin uptake inhibitors (SSRIs) with a constellation of neonatal signs. The nurse is about to perform an assessment on the infant of a mother with a history of a mood disorder. Which signs and symptoms in the neonate may be the result of maternal SSRI use? (Select all that apply.) a. Irritability b. Shivering c. Fever d. Hyperglycemia e. Hypotonia

A, B, C

3. The Baby Friendly Hospital Initiative endorsed by the World Health Organization (WHO) and the United Nations Childrens Fund (UNICEF) was founded to encourage institutions to offer optimal levels of care for lactating mothers. Which actions are included in the Ten Steps to Successful Breastfeeding for Hospitals? (Select all that apply.) a.Give newborns no food or drink other than breast milk. b.Have a written breastfeeding policy that is communicated to all staff members. c.Help mothers initiate breastfeeding within hour of childbirth. d.Give artificial teats or pacifiers as necessary. e.Return infants to the nursery at night.

A, B, C (No artificial teats or pacifiers (also called dummies or soothers) should be given to breastfeeding infants. Although pacifiers have been linked to a reduction in SIDs, they should not be introduced until the infant is 3 to 4 weeks old and breastfeeding is well established. No other food or drink should be given to the newborn unless medically indicated. The breastfeeding policy should be routinely communicated to all health care staff members. All staff should be trained in the skills necessary to maintain this policy. Breastfeeding should be initiated within hour of childbirth, and all mothers need to be shown how to maintain lactation even if separated from their babies. The facility should practice rooming in and keep mothers and babies together 24 hours a day.)

1. Which actions are examples of appropriate techniques to wake a sleepy infant for breastfeeding? (Select all that apply.) a.Unwrapping the infant b.Changing the diaper c.Talking to the infant d.Slapping the infants hands and feet e.Applying a cold towel to the infants abdomen

A, B, C (Unwrapping the infant, changing the diaper, and talking to the infant are appropriate techniques to use when trying to wake a sleepy infant. The parent can rub, never slap, the infants hands or feet to wake the infant. Applying a cold towel to the infants abdomen may lead to cold stress in the infant. The parent may want to apply a cool cloth to the infants face to wake the infant.)

Tachysystole, previously referred to as hyperstimulation, is defined as: (Select one or more) a. Contractions lasting 2 minutes or longer b. Five or more contractions in 10 minutes over a 30-minute window c. Contractions occurring within 1 minute of each other d. Uterine resting tone below 20 mm/Hg

A, B, C Contractions lasting more than 2 minutes, five or more contractions in 10 minutes, and contractions occurring within 1 minute of each other describe the criteria for tachysystole. Uterine resting tone below 20 mm/Hg reflects normal uterine resting tone

The perinatal nurse is caring for Christy following the birth of her first child. Based on Christy's history, the RN recognizes that risk factors for postpartum depression include: Select one or more: a. Loss of friends based on upcoming divorce; family is unable to assist b. Separated from spouse pending divorce c. Unplanned cesarean delivery secondary to d. Good prenatal care with uneventful pregnancy

A, B, C Recognized risk factors for postpartum depression include a history of depression before pregnancy, depression or anxiety during pregnancy, poor quality relationship with partner, life/child care stresses, and complications of pregnancy/childbirth.

The nurse is teaching the parents of a healthy newborn about infant safety. Which of the following should be included in the teaching plan? (Select all that apply). a. Water temperature for the infant's bath should be 100.4 degrees F. b. Do not cook while holding an infant c. Cover electrical outlets d. Remove strings from infant sleepwear, bedding, and pacifiers to prevent strangulation.

A, B, C, D

The perinatal nurse is teaching the new mother who has chosen to formula feed her infant. Appropriate instructions to be given to this mother include (select all that apply): a. Mix the formula according to manufacturer's instructions; do not overdilute or underdilute b. Periodically check the nipple for slow flow. c. Prepare only enough formula to last for 24 hours and discard open containers or prepared formula after 24 hours.

A, B, C, D Parents should be advised to read and follow the manufacturer's instructions explicitly when preparing the formula, because some require no water and some need to be diluted with water. Cold water should be used to mix the powder, only the amount to be used for each feeding should be prepared, and any unused formula should be discarded. The nipples should be checked periodically during feedings for correct flow and should be replaced regularly.

Many common drugs of abuse cause significant physiologic and behavioral problems in infants who are breastfed by mothers currently using (choose all that apply): A. Amphetamine B. Heroin C. Nicotine D. PCP E. Morphine

A, B, C, D These drugs of abuse are contraindicated during breastfeeding because of the reported effects on the infant. Morphine is a medication that often is used to treat neonatal abstinence syndrome. p. 1019

Which of the following are management options for couples faced with infertility challenges, depending of the cause? (Select all that apply). a. adoption b. remain child-free c. assisted reproductive technology procedures d. Medication

A, B, C, D see p 204 for child-free option

1. Pain should be regularly assessed in all newborns. If the infant is displaying physiologic or behavioral cues that indicate pain, then measures should be taken to manage the pain. Which interventions are examples of nonpharmacologic pain management techniques? (Select all that apply.) a. Swaddling b. Nonnutritive sucking c. Skin-to-skin contact with the mother d. Sucrose e. Acetaminophen

A, B, C, D (Swaddling, nonnutritive sucking, skin-to-skin contact with the mother, and sucrose are all appropriate nonpharmacologic techniques used to manage pain in neonates. Acetaminophen is a pharmacologic method of treating pain.)

Approximately 10% to 15% of all clinically recognized pregnancies end in miscarriage. What are possible causes of early miscarriage? (Select all that apply.) a. Chromosomal abnormalities b. Nausea and vomiting in early pregnancy c. Endocrine imbalance d. Systemic disorders e. Varicella

A, B, C, D, E Although most N/V in early pregnancy is not likely to relate to miscarriage, severe dehydration can reduce uterine circulation severely enough to impact a pregnancy

2. A nurse is discussing the signs and symptoms of mastitis with a mother who is breastfeeding. Which findings should the nurse include in the discussion? (Select all that apply.) a.Breast tenderness b.Warmth in the breast c.Area of redness on the breast often resembling the shape of a pie wedge d.Small white blister on the tip of the nipple e.Fever and flulike symptoms

A, B, C, E (Breast tenderness, warmth in the breast, redness on the breast, and fever and flulike symptoms are commonly associated with mastitis and should be included in the nurses discussion of mastitis. A small white blister on the tip of the nipple generally is not associated with mastitis but is commonly seen in women who have a plugged milk duct.)

According to the CDC, which of the following are significant causes of maternal death in the United States? (Select all that apply). a. hemorrhage b. cardio-vascular disease c. non-cardiovascular conditions d. street drug use e. ski accidents f. hypertensive disorders

A, B, C, F

Following the complete assessment and review of the medical reports of a pregnant female, the nurse concludes that the female is in week 32 of pregnancy. What findings are consistent with the nurse's conclusion? Select all that apply. A. Fetal movements are clearly visible. B. Cardiac output of the patient is increased. C. Uterus is almost the size of a grapefruit. D. Braxton Hicks contractions are observed. E. Fetal heart tone is detected by ultrasound.

A, B, D (The fetal movements are clearly visible on ultrasound during week 32 of pregnancy as the fetus is developed and active. An increase in the cardiac output around 30% to 50% is seen in week 32, which later declines by about 20% in week 40. Braxton Hicks contractions are irregular, painless contractions, which become definite after week 28 of pregnancy. The uterus is almost the size of a grapefruit in week 12, which increases later because of mechanical pressure of the fetus. Fetal heart tones are detected by ultrasound in week 6, but later can be easily detected by a fetal stethoscope.)

The hormonal reports of a pregnant female reveal increased estrogen levels in the body. Which related signs would the nurse find in the patient? Select all that apply. A. Mucoid discharge from the cervix B. Heaviness in the patient's breasts C. Milk discharge from the patient's nipples D. Decreased chest expansion of the patient E. Well-defined pink blotches on the palm

A, B, E (High levels of estrogen during pregnancy increase the production of cervical mucus. Therefore pregnant women have copious white or gray cervical discharge. Increased estrogen levels increase the blood supply to the breasts, thereby causing breast heaviness. The presence of well-defined pink blotches on the palm, referred to as palmar erythema, is also the effect of increased estrogen levels during pregnancy. Milk production is possible only when the baby has been delivered and there is a decreased estrogen level in the body. High levels of estrogen cause laxity of the ligaments of the rib cage, which increases the chest expansion.)

3. The Period of Purple Crying is a program developed to educate new parents about infant crying and the dangers of shaking a baby. Each letter in the acronym PURPLE represents a key concept of this program. Which concepts are accurate? (Select all that apply.) a. P: peak of crying and painful expression b. U: unexpected c. R: baby is resting at last d. L: extremely loud e. E: evening

A, B, E (P: peak of crying; U: unexpectedcomes and goes; R: resists soothing; P: painline face; L: longlasting up to 5 hours a day; and E: evening or late afternoon. Many hospitals now provide parents with an educational DVD and provide education before discharge.)

Yolanda is 6 weeks pregnant by dates and is considering abortion. What options might be appropriate for her at this point? (Select all that apply). a. medical abortion with mifepristone and misopristol b. emergency contraception c. surgical abortion with aspiration d. it is too late for her to have an abortion

A, C, see pp 194-195 of text

The CHOICE Project removed 3 key barriers to contraception for many women. These included which of the following? (Select one or more) a. cost b. parental permission c. lack of access to preferred method d. state laws that restrict methods e. knowledge deficit

A, C, D

The diagnosis of pregnancy is based on which positive signs of pregnancy (Select all that apply)? A. Identification of fetal heartbeat B. Palpation of fetal outline C. Visualization of the fetus D. Verification of fetal movement E. Positive hCG test

A, C, D (Identification of fetal heartbeat, visualization of the fetus, and verification of fetal movement all are positive, objective signs of pregnancy. Palpation of fetal outline and a positive hCG test are probable signs of pregnancy. A tumor also can be palpated. Medication and tumors may lead to false-positive results on pregnancy tests.)

2. As recently as 2005, the AAP revised safe sleep practices to assist in the prevention of SIDS. The nurse should model these practices in the hospital and incorporate this information into the teaching plan for new parents. Which practices are ideal for role modeling? (Select all that apply.) a. Fully supine position for all sleep b. Side-sleeping position as an acceptable alternative c. Tummy time for play d. Infant sleep sacks or buntings e. Soft mattress

A, C, D (The back to sleep position is now recommended as the only position for every sleep period. To prevent positional plagiocephaly (flattening of the head) the infant should spend time on his or her abdomen while awake and for play. Loose sheets and blankets may be dangerous because they could easily cover the babys head. The parents should be instructed to tuck any bedding securely around the mattress or use sleep sacks or bunting bags instead. The side-sleeping position is no longer an acceptable alternative position, according to the AAP. Infants should always sleep on a firm surface, ideally a firm crib mattress covered by a sheet only. Quilts and sheepskins, among other bedding, should not be placed under the infant.)

Cleft lip or palate is a common congenital midline fissure, or opening, in the lip or palate resulting from failure of the primary palate to fuse. Multiple genetic and, to a lesser extent, environmental factors may lead to the development of a cleft lip or palate. Such factors include (choose all that apply): A. Alcohol consumption B. Female gender C. Use of some antiepileptics D. Maternal cigarette smoking E. Antibiotic use in pregnancy

A, C, D Factors that are associated with the potential development of cleft lip or palate are maternal infections, radiation exposure, corticosteroids, anticonvulsants, male gender, Native American or Asian descent, and smoking during pregnancy. Cleft lip is more common in male infants. Antibiotic use in pregnancy is not associated with the development of cleft lip or palate. p. 1039

4. Which statements concerning the benefits or limitations of breastfeeding are accurate? (Select all that apply.) a.Breast milk changes over time to meet the changing needs as infants grow. b.Breastfeeding increases the risk of childhood obesity. c, Breast milk and breastfeeding may enhance cognitive development. d.Long-term studies have shown that the benefits of breast milk continue after the infant is weaned. e.Benefits to the infant include a reduced incidence of SIDS

A, C, D, E (Breastfeeding actually decreases the risk of childhood obesity. Human milk is the perfect food for human infants. Breast milk changes over time to meet the demands of the growing infant. Scientific evidence is clear that human milk provides the best nutrients for infants with continued benefits long after weaning. Fatty acids in breast milk promote brain growth and development and may lead to enhanced cognition. Infants who are breastfed experience a reduced incidence of SIDS.)

4. Hearing loss is one of the genetic disorders included in the universal screening program. Auditory screening of all newborns within the first month of life is recommended by the AAP. What is the rationale for having this testing performed? (Select all that apply.) a. Prevents or reduces developmental delays b. Reassures concerned new parents c. Provides early identification and treatment d. Helps the child communicate better e. Is recommended by the Joint Committee on Infant Hearing

A, C, D, E (New parents are often anxious regarding auditory screening and its impending results; however, parental anxiety is not the reason for performing the screening test. Auditory screening is usually performed before hospital discharge. Importantly, the nurse ensures the parents that the infant is receiving appropriate testing and fully explains the test to the parents. For infants who are referred for further testing and follow-up, providing further explanation and emotional support to the parents is an important responsibility for the nurse. All other responses are appropriate reasons for auditory screening of the newborn. Infants who do not pass the screening test should have it repeated. If the infant still does not pass the test, then he or she should have a full audiologic and medical evaluation by 3 months of age. If necessary, the infant should be enrolled in an early intervention program by 6 months of age.)

The induction of labor is considered an acceptable obstetric procedure if it is in the best interest to deliver the fetus. The charge nurse on the labor and delivery unit is often asked to schedule clients for this procedure and therefore must be cognizant of the specific conditions appropriate for labor induction. What are appropriate indications for induction? (Select all that apply) a. Fetal death b. Convenience of the woman or her physician c. Postterm pregnancy d. Maternal fatigue and frustration at 38 weeks gestation e. Rupture of membranes at or near term

A, C, E

5. The AAP recommends pasteurized donor milk for preterm infants if the mothers own milk in not available. Which statements regarding donor milk and milk banking are important for the nurse to understand and communicate to her client? (Select all that apply.) a.All milk bank donors are screened for communicable diseases. b.Internet milk sharing is an acceptable source for donor milk. c.Donor milk may be given to transplant clients. d.Donor milk is used in neonatal intensive care units (NICUs) for severely low-birth-weight infants only. e.Donor milk may be used for children with immunoglobulin A (IgA) deficiencies.

A, C, E (Because of the antiinfective and growth promotion properties for donor milk, donor milk is highly recommended for preterm and sick infants, as well as for term newborns. Human donor milk has also been used for older children with short gut syndrome, immunodeficiencies, metabolic disorders, or congenital anomalies. Human donor milk has also been used in the adult populationposttransplant clients and for those with colitis, ulcers, or cirrhosis of the liver. Some mothers acquire milk through Internet-based or community-based milk sharing. The U.S. Food and Drug Administration (FDA) has issued a warning regarding this practice. Samples of milk from these sources are higher in contaminants and infectious disease. A milk bank that belongs to the Human Milk Banking Association of North America should always be used for donor milk. All donors are scrupulously screened, and the milk is tested to determine its safety for use.)

Karen, a G2 P2, experienced a precipitous birth 90 minutes ago. Her infant is 4200 grams and a repair of a second-degree laceration was needed following the birth. As part of the nursing assessment, the nurse discovers that Karen's uterus is boggy. Furthermore, it is noted that Karen's vaginal bleeding has increased. What is the nurse's most appropriate actions? Select one or more: a. Assess vital signs including blood pressure and pulse. b. Massage the uterine fundus with continual lower segment support. c. Measure and document each perineal pad changed in order to assess blood loss. d. Assess for bladder distention and encourage patient to void.

A, D Massaging the fundus and resovling bladder distention will help to improve uterine atony and are the first interventions needed. B, C are interventions that can be performed after re-establishing uterine tone.

Hypothyroidism occurs in 2 to 3 pregnancies per 1000. Because severe hypothyroidism is associated with infertility and miscarriage, it is not often seen in pregnancy. Regardless of this fact, the nurse should be aware of the characteristic symptoms of hypothyroidism. Which do they include? (Select all that apply.) a. Lethargy b. Weight loss c. Hot flashes d. Decrease in exercise capacity e. Cold intolerance

A, D, E

Researchers have found a number of common risk factors that increase a woman's chance of developing a breast malignancy. It is essential for the nurse who provides care to women of any age to be aware of which risk factors? (Select all that apply.) a. Family history b. Late menarche c. Early menopause d. Age e. Nulliparity or first pregnancy after age 40 years

A, D, E

The practice of the calendar rhythm method is based on the number of days in each menstrual cycle. The fertile period is determined after monitoring each cycle for 6 months. The beginning of the fertile period is estimated by subtracting 18 days from the longest cycle and 11 days from the shortest. If the womans cycles vary in length from 24 to 30 days, then her fertile period would be day _____ through day ______.

ANS: 6; 19 To avoid pregnancy, the couple must abstain from intercourse on days 6 through 19. Ovulation occurs on day 12 (plus or minus 2 days either way).

2. During the immediate postpartum period, saturation of one pad within 1 hour or less is considered ____________________ blood loss.

ANS: Heavy Any estimation of lochial flow is inaccurate and incomplete without consideration of the time factor. The woman who saturates a perineal pad in 1 hour or less is bleeding much more heavily than the woman who saturates a pad in 8 hours.

1. The _________________________ test is used to detect the amount of fetal blood in the maternal circulation.

ANS: Kleihauer-Betke This test is ordered if a large fetomaternal transfusion is suspected. If more than 15 ml of fetal blood is present in maternal circulation, a higher dose of Rh immune globulin must be given.

18. The nurse caring for a newborn checks the record to note clinical findings that occurred before her shift. Which finding related to the renal system would be of increased significance and require further action? a. The pediatrician should be notified if the newborn has not voided in 24 hours. b. Breastfed infants will likely void more often during the first days after birth. c. Brick dust or blood on a diaper is always cause to notify the physician. d. Weight loss from fluid loss and other normal factors should be made up in 4 to 7 days.

ANS: A A newborn who has not voided in 24 hours may have any of a number of problems, some of which deserve the attention of the pediatrician. Formula-fed infants tend to void more frequently in the first 3 days; breastfed infants will void less during this time because the mothers breast milk has not yet come in. Brick dust may be uric acid crystals; blood spotting could be attributable to the withdrawal of maternal hormones (pseudomenstruation) or a circumcision. The physician must be notified only if the cause of bleeding is not apparent. Weight loss from fluid loss might take 14 days to regain.

A client at 39 weeks of gestation has been admitted for an external version. Which intervention would the nurse anticipate the provider to order? a.Tocolytic drug b.Contraction stress test (CST) c.Local anesthetic d.Foley catheter

ANS: A A tocolytic drug will relax the uterus before and during the version, thus making manipulation easier. CST is used to determine the fetal response to stress. A local anesthetic is not used with external version. Although the bladder should be emptied, catheterization is not necessary.

7. A new mother states that her infant must be cold because the babys hands and feet are blue. This common and temporary condition is called what? a. Acrocyanosis b. Erythema toxicum neonatorum c. Harlequin sign d. Vernix caseosa

ANS: A Acrocyanosis, or the appearance of slightly cyanotic hands and feet, is caused by vasomotor instability, capillary stasis, and a high hemoglobin level. Acrocyanosis is normal and intermittently appears over the first 7 to 10 days after childbirth. Erythema toxicum neonatorum (also called erythema neonatorum) is a transient newborn rash that resembles flea bites. The harlequin sign is a benign, transient color change in newborns. One half of the body is pale, and the other one half is ruddy or bluish-red with a line of demarcation. Vernix caseosa is a cheeselike, whitish substance that serves as a protective covering for the newborn.

A woman is undergoing a nipple-stimulated CST. She is having contractions that occur every 3 minutes. The fetal heart rate (FHR) has a baseline heart rate of approximately 120 beats per minute without any decelerations. What is the correct interpretation of this test? a.Negative b.Positive c.Satisfactory d.Unsatisfactory

ANS: A Adequate uterine activity necessary for a CST consists of three contractions in a 10-minute time frame. If no decelerations are observed in the FHR pattern with the contractions, then the findings are considered to be negative. A positive CST indicates the presence of repetitive late FHR decelerations. The terms satisfactory or unsatisfactory are not applicable.

Which information is the highest priority for the nurse to comprehend regarding the BPP? a.BPP is an accurate indicator of impending fetal well-being. b.BPP is a compilation of health risk factors of the mother during the later stages of pregnancy. c.BPP consists of a Doppler blood flow analysis and an amniotic fluid index (AFI). d.BPP involves an invasive form of an ultrasonic examination.

ANS: A An abnormal BPP score is one indication that labor should be induced. The BPP evaluates the health of the fetus, requires many different measures, and is a noninvasive procedure.

A client asks her nurse, "My doctor told me that he is concerned with the grade of my placenta because I am overdue. What does that mean?" What is the nurse's best response? a."Your placenta changes as your pregnancy progresses, and it is given a score that indicates how well it is functioning." b."Your placenta isn't working properly, and your baby is in danger." c."We need to perform an amniocentesis to detect if you have any placental damage." d."Don't worry about it. Everything is fine."

ANS: A An explanation of what is meant by the "grade of my placenta" is the most appropriate response. If the client desires further information, the nurse can explain that calcium deposits are significant in postterm pregnancies, and ultrasonography can also be used to determine placental aging. Although stating that the client's placenta is not working properly and that the baby is in danger may be a valid response, it does not reflect therapeutic communication techniques and is likely to alarm the client. An ultrasound, not amniocentesis, is the method of assessment used to determine placental maturation. Telling the client not to worry is not appropriate and discredits her concerns.

31. Under which circumstance should the nurse immediately alert the pediatric provider? a. Infant is dusky and turns cyanotic when crying. b. Acrocyanosis is present 1 hour after childbirth. c. The infants blood glucose level is 45 mg/dl. d. The infant goes into a deep sleep 1 hour after childbirth.

ANS: A An infant who is dusky and becomes cyanotic when crying is showing poor adaptation to extrauterine life. Acrocyanosis is an expected finding during the early neonatal life and is within the normal range for a newborn.Infants enter the period of deep sleep when they are approximately 1 hour old.

A 39-year-old primigravida woman believes that she is approximately 8 weeks pregnant, although she has had irregular menstrual periods all her life. She has a history of smoking approximately one pack of cigarettes a day; however, she tells the nurse that she is trying to cut down. Her laboratory data are within normal limits. What diagnostic technique would be useful at this time? a.Ultrasound examination b.Maternal serum alpha-fetoprotein (MSAFP) screening c.Amniocentesis d.Nonstress test (NST)

ANS: A An ultrasound examination could be performed to confirm the pregnancy and to determine the gestational age of the fetus. An MSAFP screening is performed at 16 to 18 weeks of gestation; therefore, it is too early in the woman's pregnancy to perform this diagnostic test. An amniocentesis is performed if the MSAFP levels are abnormal or if fetal or maternal anomalies are detected. An NST is performed to assess fetal well-being in the third trimester.

A woman in preterm labor at 30 weeks of gestation receives two 12-mg intramuscular (IM) doses of betamethasone. What is the purpose of this pharmacologic intervention? a.To stimulate fetal surfactant production b.To reduce maternal and fetal tachycardia associated with ritodrine administration c.To suppress uterine contractions d.To maintain adequate maternal respiratory effort and ventilation during magnesium sulfate therapy

ANS: A Antenatal glucocorticoids administered as IM injections to the mother accelerate fetal lung maturity. Propranolol (Inderal) is given to reduce the effects of ritodrine administration. Betamethasone has no effect on uterine contractions. Calcium gluconate is given to reverse the respiratory depressive effects of magnesium sulfate therapy.

Which description most accurately describes the augmentation of labor? a.Is part of the active management of labor that is instituted when the labor process is unsatisfactory b.Relies on more invasive methods when oxytocin and amniotomy have failed c.Is a modern management term to cover up the negative connotations of forceps-assisted birth d.Uses vacuum cups

ANS: A Augmentation is part of the active management of labor that stimulates uterine contractions after labor has started but is not progressing satisfactorily. Augmentation uses amniotomy and oxytocin infusion, as well as some more gentle, noninvasive methods. Forceps-assisted births are less common than in the past and not considered a method of augmentation. A vacuum-assisted delivery occurs during childbirth if the mother is too exhausted to push. Vacuum extraction is not considered an augmentation methodology.

The client is being induced in response to worsening preeclampsia. She is also receiving magnesium sulfate. It appears that her labor has not become active, despite several hours of oxytocin administration. She asks the nurse, "Why is this taking so long?" What is the nurse's most appropriate response? a."The magnesium is relaxing your uterus and competing with the oxytocin. It may increase the duration of your labor." b."I don't know why it is taking so long." c."The length of labor varies for different women." d."Your baby is just being stubborn."

ANS: A Because magnesium sulfate is a tocolytic agent, its use may increase the duration of labor. The amount of oxytocin needed to stimulate labor may be more than that needed for the woman who is not receiving magnesium sulfate. The nurse should explain to the client the effects of magnesium sulfate on the duration of labor. Although the length of labor varies for different women, the most likely reason this woman's labor is protracted is the tocolytic effects of magnesium sulfate. The behavior of the fetus has no bearing on the length of labor.

28. Which intervention can nurses use to prevent evaporative heat loss in the newborn? a. Drying the baby after birth, and wrapping the baby in a dry blanket b. Keeping the baby out of drafts and away from air conditioners c. Placing the baby away from the outside walls and windows d. Warming the stethoscope and the nurses hands before touching the baby

ANS: A Because the infant is wet with amniotic fluid and blood, heat loss by evaporation quickly occurs. Heat loss by convection occurs when drafts come from open doors and air currents created by people moving around. If the heat loss is caused by placing the baby near cold surfaces or equipment, it is referred to as a radiation heat loss. Conduction heat loss occurs when the baby comes in contact with cold surfaces.

Of these psychosocial factors, which has the least negative effect on the health of the mother and/or fetus? a.Moderate coffee consumption b.Moderate alcohol consumption c.Cigarette smoke d.Emotional distress

ANS: A Birth defects in humans have not been related to caffeine consumption. Pregnant women who consume more than 300 mg of caffeine daily may be at increased risk for miscarriage or IUGR. Although the exact effects of alcohol in pregnancy have not been quantified, it exerts adverse effects on the fetus including fetal alcohol syndrome, fetal alcohol effects, learning disabilities, and hyperactivity. A strong, consistent, causal relation has been established between maternal smoking and reduced birth weight. Childbearing triggers profound and complex physiologic and psychologic changes on the mother. Evidence suggests a relationship between emotional distress and birth complications.

Which intervention is most important when planning care for a client with severe gestational hypertension? a.Induction of labor is likely, as near term as possible. b.If at home, the woman should be confined to her bed, even with mild gestational hypertension. c.Special diet low in protein and salt should be initiated. d.Vaginal birth is still an option, even in severe cases.

ANS: A By 34 weeks of gestation, the risk of continuing the pregnancy may be considered greater than the risks of a preterm birth. Strict bed rest is controversial for mild cases; some women in the hospital are even allowed to move around. Diet and fluid recommendations are essentially the same as for healthy pregnant women, although some authorities have suggested a diet high in protein. Women with severe gestational hypertension should expect a cesarean delivery.

12. A client is warm and asks for a fan in her room for her comfort. The nurse enters the room to assess the mother and her infant and finds the infant unwrapped in his crib with the fan blowing over him on high. The nurse instructs the mother that the fan should not be directed toward the newborn and that the newborn should be wrapped in a blanket. The mother asks why. How would the nurse respond? a. Your baby may lose heat by convection, which means that he will lose heat from his body to the cooler ambient air. You should keep him wrapped, and should prevent cool air from blowing on him. b. Your baby may lose heat by conduction, which means that he will lose heat from his body to the cooler ambient air. You should keep him wrapped, and should prevent cool air from blowing on him. c. Your baby may lose heat by evaporation, which means that he will lose heat from his body to the cooler ambient air. You should keep him wrapped, and should prevent cool air from blowing on him. d. Your baby will easily get cold stressed and needs to be bundled up at all times.

ANS: A Convection is the flow of heat from the body surface to cooler ambient air. Because of heat loss by convection, all newborns in open bassinets should be wrapped to protect them from the cold. Conduction is the loss of heat from the body surface to cooler surfaces, not air, in direct contact with the newborn. Evaporation is a loss of heat that occurs when a liquid is converted into a vapor. In the newborn, heat loss by evaporation occurs as a result of vaporization of moisture from the skin. Cold stress may occur from excessive heat loss; however, this does not imply that the infant will become stressed if not bundled at all times. Furthermore, excessive bundling may result in a rise in the infants temperature.

9. A woman is 16 weeks pregnant and has elected to terminate her pregnancy. Which is the most common technique used for the termination of a pregnancy in the second trimester? a.Dilation and evacuation (D&E) b.Methotrexate administration c.Prostaglandin administration d.Vacuum aspiration

ANS: A D&E can be performed at any point up to 20 weeks of gestation. It is more commonly performed between 13 and 16 weeks of gestation. Methotrexate is a cytotoxic drug that causes early abortion by preventing fetal cell division. Prostaglandins are also used for early abortion and work by dilating the cervix and initiating uterine wall contractions. Vacuum aspiration is used for abortions in the first trimester. DIF: Cognitive Level: Understand REF: p. 194

The nurse sees a woman for the first time when she is 30 weeks pregnant. The client has smoked throughout the pregnancy, and fundal height measurements now are suggestive of intrauterine growth restriction (IUGR) in the fetus. In addition to ultrasound to measure fetal size, what is another tool useful in confirming the diagnosis? a.Doppler blood flow analysis b.Contraction stress test (CST) c.Amniocentesis d.Daily fetal movement counts

ANS: A Doppler blood flow analysis allows the examiner to study the blood flow noninvasively in the fetus and the placenta. It is a helpful tool in the management of high-risk pregnancies because of IUGR, diabetes mellitus, multiple fetuses, or preterm labor. Because of the potential risk of inducing labor and causing fetal distress, a CST is not performed on a woman whose fetus is preterm. Indications for an amniocentesis include diagnosis of genetic disorders or congenital anomalies, assessment of pulmonary maturity, and the diagnosis of fetal hemolytic disease, not IUGR. Fetal kick count monitoring is performed to monitor the fetus in pregnancies complicated by conditions that may affect fetal oxygenation. Although this may be a useful tool at some point later in this woman's pregnancy, it is not used to diagnose IUGR.

7. Which statement regarding emergency contraception is correct? a.Emergency contraception requires that the first dose be taken within 72 hours of unprotected intercourse. b.Emergency contraception may be taken right after ovulation. c.Emergency contraception has an effectiveness rate in preventing pregnancy of approximately 50%. d.Emergency contraception is commonly associated with the side effect of menorrhagia.

ANS: A Emergency contraception should be taken as soon as possible or within 72 hours of unprotected intercourse to prevent pregnancy. If taken before ovulation, follicular development is inhibited, which prevents ovulation. The risk of pregnancy is reduced by as much as 75%. The most common side effect of postcoital contraception is nausea. DIF: Cognitive Level: Understand REF: p. 186

13. A first-time father is changing the diaper of his 1-day-old daughter. He asks the nurse, What is this black, sticky stuff in her diaper? What is the nurses best response? a. Thats meconium, which is your babys first stool. Its normal. b. Thats transitional stool. c. That means your baby is bleeding internally. d. Oh, dont worry about that. Its okay.

ANS: A Explaining what meconium is and that it is normal is an accurate statement and the most appropriate response. Transitional stool is greenish-brown to yellowish-brown and usually appears by the third day after the initiation of feeding. Telling the father that the baby is internally bleeding is not an accurate statement. Telling the father not to worry is not appropriate. Such responses are belittling to the father and do not teach him about the normal stool patterns of his daughter.

13. Which term best describes the conscious decision concerning when to conceive or avoid pregnancy as opposed to the intentional prevention of pregnancy during intercourse? a.Family planning b.Birth control c.Contraception d.Assisted reproductive therapy

ANS: A Family planning is the process of deciding when and if to have children. Birth control is the device and/or practice used to reduce the risk of conceiving or bearing children. Contraception is the intentional prevention of pregnancy during sexual intercourse. Assisted reproductive therapy is one of several possible treatments for infertility. DIF: Cognitive Level: Understand REF: p. 171

A woman with severe preeclampsia has been receiving magnesium sulfate by intravenous infusion for 8 hours. The nurse assesses the client and documents the following findings: temperature of 37.1° C, pulse rate of 96 beats per minute, respiratory rate of 24 breaths per minute, BP of 155/112 mm Hg, 3+ DTRs, and no ankle clonus. The nurse calls the provider with an update. The nurse should anticipate an order for which medication? a.Hydralazine b.Magnesium sulfate bolus c.Diazepam d.Calcium gluconate

ANS: A Hydralazine is an antihypertensive medication commonly used to treat hypertension in severe preeclampsia. Typically, it is administered for a systolic BP higher than 160 mm Hg or a diastolic BP higher than 110 mm Hg. An additional bolus of magnesium sulfate may be ordered for increasing signs of CNS irritability related to severe preeclampsia (e.g., clonus) or if eclampsia develops. Diazepam is sometimes used to stop or shorten eclamptic seizures. Calcium gluconate is used as the antidote for magnesium sulfate toxicity. The client is not currently displaying any signs or symptoms of magnesium toxicity.

2. Part of the health assessment of a newborn is observing the infants breathing pattern. What is the predominate pattern of newborns breathing? a. Abdominal with synchronous chest movements b. Chest breathing with nasal flaring c. Diaphragmatic with chest retraction d. Deep with a regular rhythm

ANS: A In a normal infant respiration, the chest and abdomen synchronously rise and infant breaths are shallow and irregular. Breathing with nasal flaring is a sign of respiratory distress. Diaphragmatic breathing with chest retraction is also a sign of respiratory distress.

The exact cause of preterm labor is unknown but believed to be multifactorial. Infection is thought to be a major factor in many preterm labors. Which type of infection has not been linked to preterm birth? a.Viral b.Periodontal c.Cervical d.Urinary tract

ANS: A Infections that increase the risk of preterm labor and birth are bacterial and include cervical, urinary tract, periodontal, and other bacterial infections. Therefore, early, continual, and comprehensive participation by the client in her prenatal care is important. Recent evidence has shown a link between periodontal infections and preterm labor. Researchers recommend regular dental care before and during pregnancy, oral assessment as a routine part of prenatal care, and scrupulous oral hygiene to prevent periodontal infections.

Which condition is considered a medical emergency that requires immediate treatment? a.Inversion of the uterus b.Hypotonic uterus c.ITP d.Uterine atony

ANS: A Inversion of the uterus is likely to lead to hypovolemic shock and therefore is considered a medical emergency. Although hypotonic uterus, ITP, and uterine atony are serious conditions, they are not necessarily medical emergencies that require immediate treatment.

What is the most common reason for late postpartum hemorrhage (PPH)? a.Subinvolution of the uterus b.Defective vascularity of the decidua c.Cervical lacerations d.Coagulation disorders

ANS: A Late PPH may be the result of subinvolution of the uterus. Recognized causes of subinvolution include retained placental fragments and pelvic infection. Although defective vascularity, cervical lacerations, and coagulation disorders of the decidua may also cause PPH, late PPH typically results from subinvolution of the uterus, pelvic infection, or retained placental fragments.

Which analysis of maternal serum may predict chromosomal abnormalities in the fetus? a.Multiple-marker screening b.L/S ratio c.BPP d.Blood type and crossmatch of maternal and fetal serum

ANS: A Maternal serum can be analyzed for abnormal levels of alpha-fetoprotein, human chorionic gonadotropin, and estriol. The multiple-marker screening may predict chromosomal defects in the fetus. The L/S ratio is used to determine fetal lung maturity. A BPP is used for evaluating fetal status during the antepartum period. Five variables are used, but none is concerned with chromosomal problems. The blood type and crossmatch would not predict chromosomal defects in the fetus.

Which neonatal complications are associated with hypertension in the mother? a.Intrauterine growth restriction (IUGR) and prematurity b.Seizures and cerebral hemorrhage c.Hepatic or renal dysfunction d.Placental abruption and DIC

ANS: A Neonatal complications are related to placental insufficiency and include IUGR, prematurity, and necrotizing enterocolitis. Seizures and cerebral hemorrhage are maternal complications. Hepatic and renal dysfunction are maternal complications of hypertensive disorders in pregnancy. Placental abruption and DIC are conditions related to maternal morbidity and mortality.

What is the most common medical complication of pregnancy? a.Hypertension b.Hyperemesis gravidarum c.Hemorrhagic complications d.Infections

ANS: A Preeclampsia and eclampsia are two noted deadly forms of hypertension. A large percentage of pregnant women will have nausea and vomiting, but a relatively few will have the severe form called hyperemesis gravidarum. Hemorrhagic complications are the second most common medical complication of pregnancy; hypertension is the most common. Infection is a risk factor for preeclampsia.

What nursing diagnosis is the most appropriate for a woman experiencing severe preeclampsia? a.Risk for injury to mother and fetus, related to central nervous system (CNS) irritability b.Risk for altered gas exchange c.Risk for deficient fluid volume, related to increased sodium retention secondary to the administration of magnesium sulfate d.Risk for increased cardiac output, related to the use of antihypertensive drugs

ANS: A Risk for injury is the most appropriate nursing diagnosis for this client scenario. Gas exchange is more likely to become impaired, attributable to pulmonary edema. A risk for excess, not deficient, fluid volume, related to increased sodium retention, is increased, and a risk for decreased, not increased, cardiac output, related to the use of antihypertensive drugs, also is increased.

12. A hospital has a number of different perineal pads available for use. A nurse is observed soaking several of them and writing down what she sees. This activity indicates that the nurse is trying to: a. Improve the accuracy of blood loss estimation, which usually is a subjective assessment b. Determine which pad is best c. Demonstrate that other nurses usually underestimate blood loss d. Reveal to the nurse supervisor that one of them needs some time off

ANS: A Saturation of perineal pads is a critical indicator of excessive blood loss, and anything done to aid in assessment is valuable. The nurse is noting the saturation volumes and soaking appearances. Instead of determining which pad is best, the nurse is more likely noting saturation volumes and soaking appearances to improve the accuracy of blood loss estimation. Nurses usually overestimate blood loss. Soaking perineal pads and writing down results does not indicate the need for time off from work.

35. Which newborn reflex is elicited by stroking the lateral sole of the infants foot from the heel to the ball of the foot? a. Babinski b. Tonic neck c. Stepping d. Plantar grasp

ANS: A The Babinski reflex causes the toes to flare outward and the big toe to dorsiflex. The tonic neck reflex (also called the fencing reflex) refers to the posture assumed by newborns when in a supine position. The stepping reflex occurs when infants are held upright with their heel touching a solid surface and the infant appears to be walking. Plantar grasp reflex is similar to the palmar grasp reflex; when the area below the toes is touched, the infants toes curl over the nurses finger.

Which information is an important consideration when comparing the CST with the NST? a.The NST has no known contraindications. b.The CST has fewer false-positive results when compared with the NST. c.The CST is more sensitive in detecting fetal compromise, as opposed to the NST. d.The CST is slightly more expensive than the NST.

ANS: A The CST has several contraindications. The NST has a high rate of false-positive results and is less sensitive than the CST but relatively inexpensive.

1. A woman has chosen the calendar method of conception control. Which is the most important action the nurse should perform during the assessment process? a.Obtain a history of the womans menstrual cycle lengths for the past 6 to 12 months. b.Determine the clients weight gain and loss pattern for the previous year. c.Examine skin pigmentation and hair texture for hormonal changes. d.Explore the clients previous experiences with conception control.

ANS: A The calendar method of conception control is based on the number of days in each cycle, counting from the first day of menses. The fertile period is determined after the lengths of menstrual cycles have been accurately recorded for 6 months. Weight gain or loss may be partly related to hormonal fluctuations, but it has no bearing on the use of the calendar method. Integumentary changes may be related to hormonal changes, but they are not indicators for use of the calendar method. Exploring previous experiences with conception control may demonstrate client understanding and compliancy, but these experiences are not the most important aspect to assess for the discussion of the calendar method. DIF: Cognitive Level: Analyze REF: p. 174

5. A primiparous woman is to be discharged from the hospital tomorrow with her infant girl. Which behavior indicates a need for further intervention by the nurse before the woman can be discharged? a. The woman is disinterested in learning about infant care. b. The woman continues to hold and cuddle her infant after she has fed her. c. The woman reads a magazine while her infant sleeps. d. The woman changes her infant's diaper and then shows the nurse the contents of the diaper.

ANS: A The client should be excited, happy, and interested or involved in infant care. A woman who is sad, tearful, or disinterested in caring for her infant may be exhibiting signs of depression or postpartum blues and require further intervention. Holding and cuddling her infant after feeding is an appropriate parent-infant interaction. Taking time for herself while the infant is sleeping is an appropriate maternal action. Showing the nurse the contents of the diaper is appropriate because the mother is seeking approval from the nurse and notifying the nurse of the infant's elimination patterns.

Which nursing intervention should be immediately performed after the forceps-assisted birth of an infant? a.Assessing the infant for signs of trauma b.Administering prophylactic antibiotic agents to the infant c.Applying a cold pack to the infant's scalp d.Measuring the circumference of the infant's head

ANS: A The infant should be assessed for bruising or abrasions at the site of application, facial palsy, and subdural hematoma. Prophylactic antibiotics are not necessary with a forceps delivery. A cold pack would place the infant at risk for cold stress and is contraindicated. Measuring the circumference of the head is part of the initial nursing assessment.

6. What will prevent early discharge of a postpartum woman? a. Hgb <10 g b. Birth at 38 weeks of gestation c. Voids about 200 to 300 ml per void d. Episiotomy that shows slight redness and edema and is dry and approximated

ANS: A The mother's hemoglobin should be greater than 10 g for early discharge. The birth of an infant at term is not a criterion that prevents early discharge. A voiding volume of 200 to 300 ml per void is normal and does not indicate that the woman should not be discharged early. An episiotomy that shows slight redness and edema and is dry and approximated is a normal finding and does not prevent a woman from being discharged early.

A pregnant woman's BPP score is 8. She asks the nurse to explain the results. How should the nurse respond at this time? a."The test results are within normal limits." b."Immediate delivery by cesarean birth is being considered." c."Further testing will be performed to determine the meaning of this score." d."An obstetric specialist will evaluate the results of this profile and, within the next week, will inform you of your options regarding delivery."

ANS: A The normal biophysical score ranges from 8 to 10 points if the amniotic fluid volume is adequate. A normal score allows conservative treatment of high-risk clients. Delivery can be delayed if fetal well-being is indicated. Scores less than 4 should be investigated, and delivery could be initiated sooner than planned. The results of the BPP are usually available immediately after the procedure is performed. Since this score is within normal range, no further testing is required at this time.

22. How would the nurse optimally reassure the parents of an infant who develops a cephalhematoma? a. A cephalhematoma may occur with a spontaneous vaginal birth. b. A cephalhematoma only happens as a result of a forceps- or vacuum-assisted delivery. c. It is present immediately after birth. d. The blood will gradually absorb over the first few months of life.

ANS: A The nurse should explain that bleeding between the skull and the periosteum of a newborn may occur during a spontaneous vaginal delivery as a result of the pressure against the maternal bony pelvis. Low forceps and other difficult extractions may result in bleeding. However, a cephalhematoma can also spontaneously occur. Swelling may appear unilaterally or bilaterally, is usually minimal or absent at birth, and increases over the first 2 to 3 days of life. Cephalhematomas gradually disappear over 2 to 3 weeks. A less common condition results in the calcification of the hematoma, which may persist for months.

12. Although reported in small numbers, toxic shock syndrome (TSS) can occur with the use of a diaphragm. If a client is interested in this form of conception control, then the nurse must instruct the woman on how best to reduce her risk of TSS. Which comment by the nurse would be most helpful in achieving this goal? a.You should always remove your diaphragm 6 to 8 hours after intercourse. Dont use the diaphragm during menses, and watch for danger signs of TSS, including a sudden onset of fever over 38.4 C, hypotension, and a rash. b.You should remove your diaphragm right after intercourse to prevent TSS. c.Its okay to use your diaphragm during your menstrual cycle. Just be sure to wash it thoroughly first to prevent TSS. d.Make sure you dont leave your diaphragm in for longer than 24 hours, or you may get TSS.

ANS: A The nurse should instruct the client on the proper use and removal of the diaphragm and include the danger signs of TSS. The diaphragm must remain against the cervix for 6 to 8 hours to prevent pregnancy, but it should not remain in place longer than 8 hours to avoid the risk of TSS. The diaphragm should not be used during menses. DIF: Cognitive Level: Apply REF: p. 180 TOP: Nursing Process: Planning

19. What is the correct term for the cheeselike, white substance that fuses with the epidermis and serves as a protective coating? a. Vernix caseosa b. Surfactant c. Caput succedaneum d. Acrocyanosis

ANS: A The protection provided by vernix caseosa is needed because the infants skin is so thin. Surfactant is a protein that lines the alveoli of the infants lungs. Caput succedaneum is the swelling of the tissue over the presenting part of the fetal head. Acrocyanosis is cyanosis of the hands and feet, resulting in a blue coloring.

17. Which nonpharmacologic contraceptive method has a failure rate of less than 25%? a.Standard days variation b.Periodic abstinence c.Postovulation d.Coitus interruptus

ANS: A The standard days variation on the calendar method has a failure rate of 12% and is a variation of the calendar rhythm method with a fixed number of days for fertility in each cycle. The periodic abstinence method has a failure rate of 25% or higher. The postovulation method has a failure rate of 25% or higher. The coitus interruptus method has a failure rate of 27% or higher. DIF: Cognitive Level: Remember REF: p. 174

26. Which component of the sensory system is the least mature at birth? a. Vision b. Hearing c. Smell d. Taste

ANS: A The visual system continues to develop for the first 6 months after childbirth. As soon as the amniotic fluid drains from the ear (in minutes), the infants hearing is similar to that of an adult. Newborns have a highly developed sense of smell and can distinguish and react to various tastes.

A pregnant woman's amniotic membranes have ruptured. A prolapsed umbilical cord is suspected. What intervention would be the nurse's highest priority? a.Placing the woman in the knee-chest position b.Covering the cord in sterile gauze soaked in saline c.Preparing the woman for a cesarean birth d.Starting oxygen by face mask

ANS: A The woman is assisted into a modified Sims position, Trendelenburg position, or the knee-chest position in which gravity keeps the pressure of the presenting part off the cord. Although covering the cord in sterile gauze soaked saline, preparing the woman for a cesarean, and starting oxygen by face mark are appropriate nursing interventions in the event of a prolapsed cord, the intervention of top priority would be positioning the mother to relieve cord compression.

2. The laboratory results for a postpartum woman are as follows: blood type, A; Rh status, positive; rubella titer, 1:8 (EIA 0.8); hematocrit, 30%. How would the nurse best interpret these data? a. Rubella vaccine should be given b. A blood transfusion is necessary c. Rh immune globulin is necessary within 72 hours of birth d. A Kleihauer-Betke test should be performed

ANS: A This client's rubella titer indicates that she is not immune and that she needs to receive a vaccine. These data do not indicate that the client needs a blood transfusion. Rh immune globulin is indicated only if the client has a negative Rh status and the infant has a positive Rh status. A Kleihauer-Betke test should be performed if a large fetomaternal transfusion is suspected, especially if the mother is Rh negative. The data do not provide any indication for performing this test.

A perinatal nurse is caring for a woman in the immediate postbirth period. Assessment reveals that the client is experiencing profuse bleeding. What is the most likely cause for this bleeding? a.Uterine atony b.Uterine inversion c.Vaginal hematoma d.Vaginal laceration

ANS: A Uterine atony is significant hypotonia of the uterus and is the leading cause of postpartum hemorrhage. Uterine inversion may lead to hemorrhage; however, it is not the most likely source of this client's bleeding. Further, if the woman were experiencing a uterine inversion, it would be evidenced by the presence of a large, red, rounded mass protruding from the introitus. A vaginal hematoma may be associated with hemorrhage. However, the most likely clinical finding for vaginal hematoma is pain, not the presence of profuse bleeding. A vaginal laceration should be suspected if vaginal bleeding continues in the presence of a firm, contracted uterine fundus.

3. Which statements regarding physiologic jaundice are accurate? (Select all that apply.) a. Neonatal jaundice is common; however, kernicterus is rare. b. Appearance of jaundice during the first 24 hours or beyond day 7 indicates a pathologic process. c. Because jaundice may not appear before discharge, parents need instruction on how to assess for jaundice and when to call for medical help. d. Jaundice is caused by reduced levels of serum bilirubin. e. Breastfed babies have a lower incidence of jaundice.

ANS: A, B, C Neonatal jaundice occurs in 60% of term newborns and in 80% of preterm infants. The complication called kernicterus is rare. Jaundice in the first 24 hours or that persists past day 7 is cause for medical concern. Parents need to be taught how to evaluate their infant for signs of jaundice. Jaundice is caused by elevated levels of serum bilirubin. Breastfeeding is associated with an increased incidence of jaundice.

Which assessments are included in the fetal BPP? (Select all that apply.) a.Fetal movement b.Fetal tone c.Fetal heart rate d.AFI e.Placental grade

ANS: A, B, C, D Fetal movement, tone, heart rate, and AFI are all assessed in a BPP. The placental grade is determined by ultrasound and is not included in the criteria of assessment factors for a BPP.

2. Infant abduction from hospitals in the United States has increased over the past few years. As a result many maternity units have put practices into place to protect infants from possible abduction. These practices include: (select all that apply) a. Limited-entry systems b. Photo identification badges c. Fingerprint identification of all newborns d. Infant should always be transported in a bassinet e. Staff wear special scrubs or unique ID badges

ANS: A, B, C, D, E Limited-entry systems, photo identification badges, fingerprint identification of all newborns, transport of the infant in a bassinet, and special staff scrubs or unique identification badges are all practices that limit the ability of an abductor to remove an infant from the hospital. Nurses must also teach new parents to check the identity of any person attempting to remove an infant from the room and question the reason why. Clients and staff must work together to ensure the safety of newborns in the hospital environment.

What are the complications and risks associated with cesarean births? (Select all that apply.) a.Pulmonary edema b.Wound dehiscence c.Hemorrhage d.Urinary tract infections e.Fetal injuries

ANS: A, B, C, D, E Pulmonary edema, wound dehiscence, hemorrhage, urinary tract infections, and fetal injuries are possible complications and risks associated with cesarean births.

Women who are obese are at risk for several complications during pregnancy and birth. Which of these would the nurse anticipate with an obese client? (Select all that apply.) a.Thromboembolism b.Cesarean birth c.Wound infection d.Breech presentation e.Hypertension

ANS: A, B, C, E A breech presentation is not a complication of pregnancy or birth for the client who is obese. Venous thromboembolism is a known risk for obese women. Therefore, the use of thromboembolism-deterrent (TED) hose and sequential compression devices may help decrease the chance for clot formation. Women should also be encouraged to ambulate as soon as possible. In addition to having an increased risk for complications with a cesarean birth, in general, obese women are also more likely to require an emergency cesarean birth. Many obese women have a pannus (i.e., large roll of abdominal fat) that overlies a lower transverse incision made just above the pubic area. The pannus causes the area to remain moist, which encourages the development of infection. Obese women are more likely to begin pregnancy with comorbidities such as hypertension and type 2 diabetes.

4. During life in utero, oxygenation of the fetus occurs through transplacental gas exchange. When birth occurs, four factors combine to stimulate the respiratory center in the medulla. The initiation of respiration then follows. What are these four essential factors? a. Chemical b. Mechanical c. Thermal d. Psychologic e. Sensory

ANS: A, B, C, E Chemical factors are essential to initiate breathing. During labor, decreased levels of oxygen and increased levels of carbon dioxide seem to have a cumulative effect that is involved in the initiation of breathing. Clamping of the cord may also contribute to the start of respirations and results in a drop in the level of prostaglandins, which are known to inhibit breathing. Mechanical factors are also necessary to initiate respirations. As the infant passes through the birth canal, the chest is compressed. After the birth, the chest is relaxed, which allows for negative intrathoracic pressure that encourages air to flow into the lungs. The profound change in temperature between intrauterine and extrauterine life stimulates receptors in the skin to communicate with the receptors in the medulla. The stimulation of these receptors also contributes to the initiation of breathing. Sensory factors include handling by the health care provider, drying by the nurse, lights, smells, and sounds. Psychologic factors do not contribute to the initiation of respirations.

IUGR is associated with which pregnancy-related risk factors? (Select all that apply.) a.Poor nutrition b.Maternal collagen disease c.Gestational hypertension d.Premature rupture of membranes e.Smoking

ANS: A, B, C, E Poor nutrition, maternal collagen disease, gestational hypertension, and smoking are risk factors associated with the occurrence of IUGR. Premature rupture of membranes is associated with preterm labor, not IUGR.

2. Which statements describe the first stage of the neonatal transition period? (Select all that apply.) a. The neonatal transition period lasts no longer than 30 minutes. b. It is marked by spontaneous tremors, crying, and head movements. c. Passage of the meconium occurs during the neonatal transition period. d. This period may involve the infant suddenly and briefly sleeping. e. Audible grunting and nasal flaring may be present during this time.

ANS: A, B, C, E The first stage is an active phase during which the baby is alert; this stage is referred to as the first period of reactivity. Decreased activity and sleep mark the second stage, the period of decreased responsiveness. The first stage is the shortest, lasting less than 30 minutes. Such exploratory behaviors include spontaneous startle reactions. Audible grunting, nasal flaring, and chest retractions may be present; however, these behaviors usually resolve within 1 hour of life.

Transvaginal ultrasonography is often performed during the first trimester. While preparing a 6-week gestational client for this procedure, she expresses concerns over the necessity for this test. The nurse should explain that this diagnostic test may be indicated for which situations? (Select all that apply.) a.Multifetal gestation b.Obesity c.Fetal abnormalities d.Amniotic fluid volume e. Ectopic pregnancy

ANS: A, B, C, E Transvaginal ultrasound is useful in women who are obese whose thick abdominal layers cannot be penetrated with traditional abdominal ultrasound. This procedure is also used to identify multifetal gestation, ectopic pregnancy, estimating gestational age, confirming fetal viability, and identifying fetal abnormalities. Amniotic fluid volume is assessed during the second and third trimester; conventional ultrasound would be used.

Which medications are used to manage PPH? (Select all that apply.) a.Oxytocin b.Methergine c.Terbutaline d.Hemabate e.Magnesium sulfate

ANS: A, B, D Oxytocin, Methergine, and Hemabate are medications used to manage PPH. Terbutaline and magnesium sulfate are tocolytic medications that are used to relax the uterus, which would cause or worsen PPH.

The nurse is responsible for providing health teaching regarding the side effects of COCs. These side effects are attributed to estrogen, progesterone, or both. Which side effects are related to the use of COCs? (Select all that apply.) a. Gallbladder disease b. Myocardial infarction and stroke c. Hypotension d. Breast tenderness and fluid retention e. Dry skin and scalp

ANS: A, B, D Serious side effects include stroke, myocardial infarction, hypertension, gallbladder disease, and liver tumors. More common side effects include nausea, breast tenderness, fluid retention, increased appetite, oily skin and scalp, and chloasma. DIF: Cognitive Level: Understand REF: p. 184

Cell-free deoxyribonucleic acid (DNA) screening is a new method of noninvasive prenatal testing (NIPT) that has recently become available in the clinical setting. This technology can provide a definitive diagnosis of which findings? (Select all that apply.) a.Fetal Rh status b.Fetal gender c.Maternally transmitted gene disorder d.Paternally transmitted gene disorder e.Trisomy 21

ANS: A, B, D, E The NIPT cannot actually distinguish fetal from maternal DNA. It can determine fetal Rh status, gender, trisomies 13, 18, and 21, as well as paternally transmitted gene disorders. The test can be performed any time after 10 weeks of gestation and is recommended for women who have previously given birth to a child with chromosomal abnormalities.

One of the most important components of the physical assessment of the pregnant client is the determination of BP. Consistency in measurement techniques must be maintained to ensure that the nuances in the variations of the BP readings are not the result of provider error. Which techniques are important in obtaining accurate BP readings? (Select all that apply.) a.The client should be seated. b.The client's arm should be placed at the level of the heart. c.An electronic BP device should be used. d.The cuff should cover a minimum of 60% of the upper arm. e.The same arm should be used for every reading.

ANS: A, B, E BP readings are easily affected by maternal position. Ideally, the client should be seated. An alternative position is left lateral recumbent with the arm at the level of the heart. The arm should always be held in a horizontal position at approximately the level of the heart. The same arm should be used at every visit. The manual sphygmomanometer is the most accurate device. If manual and electronic devices are used in the care setting, then the nurse must use caution when interpreting the readings. A proper size cuff should cover at least 80% of the upper arm or be approximately 1.5 times the length of the upper arm.

Lacerations of the cervix, vagina, or perineum are also causes of PPH. Which factors influence the causes and incidence of obstetric lacerations of the lower genital tract? (Select all that apply.) a.Operative and precipitate births b.Adherent retained placenta c.Abnormal presentation of the fetus d.Congenital abnormalities of the maternal soft tissue e.Previous scarring from infection

ANS: A, C, D, E Abnormal adherence of the placenta occurs for unknown reasons. Attempts to remove the placenta in the usual manner can be unsuccessful, and lacerations or a perforation of the uterine wall may result. However, attempts to remove the placenta do not influence lower genital tract lacerations. Lacerations of the perineum are the most common of all lower genital tract injuries and often occur with both precipitate and operative births and are classified as first-, second-, third-, and fourth-degree lacerations. An abnormal presentation or position of the fetus, the relative size of the presenting part, and the birth canal may contribute to lacerations of the lower genital tract. Congenital abnormalities, previous scarring from infection or injury, and a contracted pelvis may also influence injury to the lower genital tract, followed by hemorrhage.

Which adverse prenatal outcomes are associated with the HELLP syndrome? (Select all that apply.) a.Placental abruption b.Placenta previa c.Renal failure d.Cirrhosis e.Maternal and fetal death

ANS: A, C, E The HELLP syndrome is associated with an increased risk for adverse perinatal outcomes, including placental abruption, acute renal failure, subcapsular hepatic hematoma, hepatic rupture, recurrent preeclampsia, preterm birth, and fetal and maternal death. The HELLP syndrome is associated with an increased risk for placental abruption, not placenta previa. It is also associated with an increased risk for hepatic hematoma, not cirrhosis.

1. Many new mothers experience some type of nipple pain during the first weeks of initiating breastfeeding. Should this pain be severe or persistent, it may discourage or inhibit breastfeeding altogether. The nurse should be aware of a variety of factors that may contribute to nipple pain. These include: (select all that apply) a. Improper feeding position b. Large-for-gestational age infant c. Fair skin d. Progesterone deficiency e. Flat or retracted nipples

ANS: A, C, E Nipple lesions may manifest as chapped, cracked, bleeding, sore, erythematous, edematous, or blistered. Factors that contribute to nipple pain include improper positioning or failure to break suction before removing the baby from the breast. Flat or retracted nipples along with the use of nipple shields, breast shells, or plastic breast pads also contribute. Women with fair skin are more likely to develop sore and cracked nipples. Prevention of nipple soreness is preferable to treatment after it appears. Vigorous feeding may be a contributing factor. This may be the case with any size infant, not just those who are large for gestational age. Estrogen or dietary deficiencies can contribute to nipple soreness.

The nurse is reviewing the educational packet provided to a client about tubal ligation. Which information regarding this procedure is important for the nurse to share? (Select all that apply.) a. It is highly unlikely that you will become pregnant after the procedure. b. Tubal ligation is an effective form of 100% permanent sterilization. You wont be able to get pregnant. c. Sterilization offers some form of protection against STIs. d. Sterilization offers no protection against STIs. e. Your menstrual cycle will greatly increase after your sterilization.

ANS: A, D A woman is unlikely to become pregnant after tubal ligation. However, sterilization offers no protection against STIs and is not 100% effective. Typically, the menstrual cycle remains the same after a tubal ligation. DIF: Cognitive Level: Apply REF: p. 190

What is a maternal indication for the use of vacuum-assisted birth? a.Wide pelvic outlet b.Maternal exhaustion c.History of rapid deliveries d.Failure to progress past station 0

ANS: B A mother who is exhausted may be unable to assist with the expulsion of the fetus. The client with a wide pelvic outlet will likely not require vacuum extraction. With a rapid delivery, vacuum extraction is not necessary. A station of 0 is too high for a vacuum-assisted birth

The client being cared for has severe preeclampsia and is receiving a magnesium sulfate infusion. Which new finding would give the nurse cause for concern? a.Sleepy, sedated affect b.Respiratory rate of 10 breaths per minute c.DTRs of 2 d.Absent ankle clonus

ANS: B A respiratory rate of 10 breaths per minute indicates the client is experiencing respiratory depression from magnesium toxicity. Because magnesium sulfate is a CNS depressant, the client will most likely become sedated when the infusion is initiated. DTRs of 2 and absent ankle clonus are normal findings.

3. A woman gave birth 48 hours ago to a healthy infant girl. She has decided to bottle-feed. During your assessment you notice that both her breasts are swollen, warm, and tender on palpation. The woman should be advised that this condition can best be treated by: a. Running warm water on her breasts during a shower b. Applying ice to the breasts for comfort c. Expressing small amounts of milk from the breasts to relieve pressure d. Wearing a loose-fitting bra to prevent nipple irritation

ANS: B Applying ice to the breasts for comfort is an appropriate intervention for treating engorgement in a mother who is bottle feeding. Ice should be applied for 15 minutes on and 45 minutes off to avoid rebound engorgement. This woman is experiencing engorgement, which can be treated by using ice packs (since she is not breastfeeding) and cabbage leaves. A bottle-feeding mother should avoid any breast stimulation, including pumping or expressing milk. A bottle-feeding mother should wear a well-fitted support bra or breast binder continuously for at least the first 72 hours after giving birth. A loose-fitting bra will not aid lactation suppression. Furthermore, the shifting of the bra against the breasts may stimulate the nipples and thereby stimulate lactation.

37. What is the rationale for evaluating the plantar crease within a few hours of birth? a. Newborn has to be footprinted. b. As the skin dries, the creases will become more prominent. c. Heel sticks may be required. d. Creases will be less prominent after 24 hours.

ANS: B As the infants skin begins to dry, the creases will appear more prominent, and the infants gestation could be misinterpreted. Footprinting nor heel sticks will not interfere with the creases. The creases will appear more prominent after 24 hours.

A number of methods can be used for inducing labor. Which cervical ripening method falls under the category of mechanical or physical? a.Prostaglandins are used to soften and thin the cervix. b.Labor can sometimes be induced with balloon catheters or laminaria tents. c.Oxytocin is less expensive and more effective than prostaglandins but creates greater health risks. d.Amniotomy can be used to make the cervix more favorable for labor.

ANS: B Balloon catheters or laminaria tents are mechanical means of ripening the cervix. Ripening the cervix, making it softer and thinner, increases the success rate of induced labor. Prostaglandin E1 is less expensive and more effective than oxytocin but carries a greater risk. Amniotomy is the artificial rupture of membranes, which is used to induce labor only when the cervix is already ripe.

18. Which contraceptive method best protects against STIs and the HIV? a.Periodic abstinence b.Barrier methods c.Hormonal methods d.Same protection with all methods

ANS: B Barrier methods, such as condoms, protect against STIs and the HIV the best of all contraceptive methods. Periodic abstinence and hormonal methods, such as birth control pills, offer no protection against STIs or the HIV. DIF: Cognitive Level: Apply REF: p. 177 TOP: Nursing Process: Planning

3. A woman who has a seizure disorder and takes barbiturates and phenytoin sodium daily asks the nurse about the pill as a contraceptive choice. What is the nurses best response? a.Oral contraceptives are a highly effective method, but they have some side effects. b.Your current medications will reduce the effectiveness of the pill. c.Oral contraceptives will reduce the effectiveness of your seizure medication. d.The pill is a good choice for a woman of your age and with your personal history.

ANS: B Because the liver metabolizes oral contraceptives, their effectiveness is reduced when they are simultaneously taken with anticonvulsants. Stating that the pill is an effective birth control method with side effects is a true statement, but this response is not the most appropriate. The anticonvulsant reduces the effectiveness of the pill, not the other way around. Stating that the pill is a good choice for a woman of her age and personal history does not teach the client that the effectiveness of the pill may be reduced because of her anticonvulsant therapy. DIF: Cognitive Level: Apply REF: p. 184 TOP: Nursing Process: Planning

The nurse who elects to work in the specialty of obstetric care must have the ability to distinguish between preterm birth, preterm labor, and low birth weight. Which statement regarding this terminology is correct? a.Terms preterm birth and low birth weight can be used interchangeably. b.Preterm labor is defined as cervical changes and uterine contractions occurring between 20 and 37 weeks of gestation. c.Low birth weight is a newborn who weighs below 3.7 pounds. d.Preterm birth rate in the United States continues to increase.

ANS: B Before 20 weeks of gestation, the fetus is not viable (miscarriage); after 37 weeks, the fetus can be considered term. Although these terms are used interchangeably, they have different meanings: preterm birth describes the length of gestation (before 37 weeks), regardless of the newborn's weight; low birth weight describes only the infant's weight at the time of birth (2500 g or less), whenever it occurs. Low birth weight is anything below 2500 g or approximately pounds. In 2011, the preterm birth rate in the United States was 11.7 %; it has dropped every year since 2008.

13. What is not a reliable indicator of impending shock from early hemorrhage? a. Respirations b. Blood pressure c. Skin condition d. Urinary output

ANS: B Blood pressure is not a reliable indicator; several more sensitive signs are available. Respiratory rate is a more sensitive and reliable indicator than blood pressure. Skin condition is a more sensitive and reliable indicator than blood pressure. Urinary output is a more sensitive and reliable indicator than blood pressure.

14. Which statement best describes the transition period between intrauterine and extrauterine existence for the newborn? a. Consists of four phases, two reactive and two of decreased responses b. Lasts from birth to day 28 of life c. Applies to full-term births only d. Varies by socioeconomic status and the mothers age

ANS: B Changes begin immediately after birth; the cutoff time when the transition is considered over (although the baby keeps changing) is 28 days. This transition period has three phases: first reactivity, decreased response, and second reactivity. All newborns experience this transition period, regardless of age or type of birth. Although stress can cause variations in the phases, the mothers age and wealth do not disturb the pattern.

34. The process during which bilirubin is changed from a fat-soluble product to a water-soluble product is known as what? a. Enterohepatic circuit b. Conjugation of bilirubin c. Unconjugated bilirubin d. Albumin binding

ANS: B Conjugation of bilirubin is the process of changing the bilirubin from a fat-soluble to a water-soluble product and is the route by which part of the bile produced by the liver enters the intestine, is reabsorbed by the liver, and is then recycled into the intestine. Unconjugated bilirubin is a fat-soluble product. Albumin binding is the process during which something attaches to a protein molecule.

23. The nurse is circulating during a cesarean birth of a preterm infant. The obstetrician requests that cord clamping be delayed. What is the rationale for this directive? a. To reduce the risk for jaundice b. To reduce the risk of intraventricular hemorrhage c. To decrease total blood volume d. To improve the ability to fight infection

ANS: B Delayed cord clamping provides the greatest benefits to the preterm infant. These benefits include a significant reduction in intraventricular hemorrhage, a reduced need for a blood transfusion, and improved blood cell volume. The risk of jaundice can increase, requiring phototherapy. Although no difference in the newborns infection fighting ability occurs, iron status is improved, which can provide benefits for 6 months.

Nurses, certified nurse-midwives, and other advanced practice nurses have the knowledge and expertise to assist women in making informed choices regarding contraception. A multidisciplinary approach should ensure that the womans social, cultural, and interpersonal needs are met. Which action should the nurse first take when meeting with a new client to discuss contraception? a. Obtain data about the frequency of coitus. b. Determine the womans level of knowledge concerning contraception and her commitment to any particular method. c. Assess the womans willingness to touch her genitals and cervical mucus. d. Evaluate the womans contraceptive life plan.

ANS: B Determining the womans level of knowledge concerning contraception and her commitment to any particular method is the primary step of this nursing assessment and necessary before completing the process and moving on to a nursing diagnosis. Once the clients level of knowledge is determined, the nurse can interact with the woman to compare options, reliability, cost, comfort level, protection from STIs, and her partners willingness to participate. Although important, obtaining data about the frequency of coitus is not the first action that the nurse should undertake when completing an assessment. Data should include not only the frequency of coitus but also the number of sexual partners, level of contraceptive involvement, and the partners objections. Assessing the womans willingness to touch herself is a key factor for the nurse to discuss should the client express an interest in using one of the fertility awareness methods of contraception. The nurse must be aware of the clients plan regarding whether she is attempting to prevent conception, delay conception, or conceive. DIF: Cognitive Level: Analyze REF: pp. 171-172

8. An unmarried young woman describes her sex life as active and involving many partners. She wants a contraceptive method that is reliable and does not interfere with sex. She requests an intrauterine device (IUD). Which information is most important for the nurse to share? a.The IUD does not interfere with sex. b.The risk of pelvic inflammatory disease will be higher with the IUD. c.The IUD will protect you from sexually transmitted infections. d.Pregnancy rates are high with the IUD.

ANS: B Disadvantages of IUDs include an increased risk of pelvic inflammatory disease (PID) in the first 20 days after insertion, as well as the risks of bacterial vaginosis and uterine perforation. The IUD offers no protection against sexually transmitted infections (STIs) or the human immunodeficiency virus (HIV), as does a barrier method. Because this woman has multiple sex partners, she is at higher risk of developing an STI. Stating that an IUD does not interfere with sex may be correct; however, it is not the most appropriate response. The typical failure rate of the IUD is approximately 1%. DIF: Cognitive Level: Apply REF: p. 187 TOP: Nursing Process: Planning

2. A married couple is discussing alternatives for pregnancy prevention and has asked about fertility awareness methods (FAMs). Which response by the nurse is most appropriate? a.Theyre not very effective, and it is very likely that youll get pregnant. b.FAMs can be effective for many couples; however, they require motivation. c.These methods have a few advantages and several health risks. d.You would be much safer going on the pill and not having to worry.

ANS: B FAMs are effective with proper vigilance about ovulatory changes in the body and with adherence to coitus intervals. FAMs are effective if correctly used by a woman with a regular menstrual cycle. The typical failure rate for all FAMs is 24% during the first year of use. FAMs have no associated health risks. The use of birth control has associated health risks. In addition, taking a pill daily requires compliance on the clients part. DIF: Cognitive Level: Apply REF: pp. 173-174 TOP: Nursing Process: Planning

An MSAFP screening indicates an elevated level of alpha-fetoprotein. The test is repeated, and again the level is reported as higher than normal. What is the next step in the assessment sequence to determine the well-being of the fetus? a.PUBS b.Ultrasound for fetal anomalies c.BPP for fetal well-being d.Amniocentesis for genetic anomalies

ANS: B If MSAFP findings are abnormal, then follow-up procedures include genetic counseling for families with a history of NTD, repeated MSAFP screenings, an ultrasound examination, and possibly amniocentesis. Indications for the use of PUBS include prenatal diagnosis of inherited blood disorders, karyotyping of malformed fetuses, detection of fetal infection, determination of the acid-base status of fetuses with IUGR, and assessment and treatment of isoimmunization and thrombocytopenia in the fetus. A BPP is a method of assessing fetal well-being in the third trimester. Before an amniocentesis, the client would have an ultrasound for direct visualization of the fetus.

Which client is at greatest risk for early PPH? a.Primiparous woman (G 2, P 1-0-0-1) being prepared for an emergency cesarean birth for fetal distress b.Woman with severe preeclampsia on magnesium sulfate whose labor is being induced c.Multiparous woman (G 3, P 2-0-0-2) with an 8-hour labor d.Primigravida in spontaneous labor with preterm twins

ANS: B Magnesium sulfate administration during labor poses a risk for PPH. Magnesium acts as a smooth muscle relaxant, thereby contributing to uterine relaxation and atony. A primiparous woman being prepared for an emergency cesarean birth for fetal distress, a multiparous woman with an 8-hour labor, and a primigravida in spontaneous labor with preterm twins do not indicate risk factors or causes of early PPH

A male client asks the nurse why it is better to purchase condoms that are not lubricated with nonoxynol-9 (a common spermicide). Which response by the nurse is the most accurate? a.The lubricant prevents vaginal irritation. b.Nonoxynol-9 does not provide protection against STIs as originally thought; it has also been linked to an increase in the transmission of the HIV and can cause genital lesions. c.The additional lubrication improves sex. d.Nonoxynol-9 improves penile sensitivity.

ANS: B Nonoxynol-9 does not provide protection against STIs as originally thought; it has also been linked to an increase in the transmission of the HIV and can cause genital lesions. Nonoxynol-9 may cause vaginal irritation, has no effect on the quality of sexual activity, and has no effect on penile sensitivity.

The nurse is planning the care for a laboring client with diabetes mellitus. This client is at greater risk for which clinical finding? a.Oligohydramnios b.Polyhydramnios c.Postterm pregnancy d.Chromosomal abnormalities

ANS: B Polyhydramnios or amniotic fluid in excess of 2000 ml is 10 times more likely to occur in the client with diabetes mellitus rather than in nondiabetic pregnancies. This complication places the mother at risk for premature rupture of membranes, premature labor, and postpartum hemorrhage. Prolonged rupture of membranes, IUGR, intrauterine fetal death, and renal agenesis (Potter syndrome) place the client at risk for developing oligohydramnios. Anencephaly, placental insufficiency, and perinatal hypoxia contribute to the risk for postterm pregnancy. Maternal age older than 35 years and balanced translocation (maternal and paternal) are risk factors for chromosomal abnormalities.

11. What is not a postpartum practice for preventing infections? a. Not letting the mother walk barefoot at the hospital b. Educating the client to wipe from back to front after voiding c. Having staff members with conditions such as strep throat, conjunctivitis, and diarrhea stay home d. Instructing the mother to change her perineal pad from front to back each time she voids or defecates

ANS: B Proper perineal care helps to prevent infection and aids in the healing process. Educating the woman to wipe from front to back (urethra to anus) after voiding or defecating is a simple first step. Walking barefoot and getting back into bed can contaminate the linens. Staff members with infections need to stay home until they are no longer contagious. She should also wash her hands before and after these functions.

A 41-week pregnant multigravida arrives at the labor and delivery unit after a NST indicated that her fetus could be experiencing some difficulties in utero. Which diagnostic tool yields more detailed information about the condition of the fetus? a.Ultrasound for fetal anomalies b.Biophysical profile (BPP) c.MSAFP screening d.Percutaneous umbilical blood sampling (PUBS)

ANS: B Real-time ultrasound permits a detailed assessment of the physical and physiologic characteristics of the developing fetus and a cataloging of normal and abnormal biophysical responses to stimuli. The BPP is a noninvasive, dynamic assessment of a fetus that is based on acute and chronic markers of fetal disease. An ultrasound for fetal anomalies would most likely have occurred earlier in the pregnancy. It is too late in the pregnancy to perform an MSAFP. Furthermore, it does not provide information related to fetal well-being. Indications for PUBS include prenatal diagnosis or inherited blood disorders, karyotyping of malformed fetuses, detection of fetal infection, determination of the acid-base status of the fetus with IUGR, and assessment and treatment of isoimmunization and thrombocytopenia in the fetus.

36. The condition during which infants are at an increased risk for subgaleal hemorrhage is called what? a. Infection b. Jaundice c. Caput succedaneum d. Erythema toxicum neonatorum

ANS: B Subgaleal hemorrhage is bleeding into the subgaleal compartment and is the result of the transition from a forceps or vacuum application. Because of the breakdown of the red blood cells within a hematoma, infants are at greater risk for jaundice. Subgaleal hemorrhage does not increase the risk for infections. Caput succedaneum is an edematous area on the head caused by pressure against the cervix. Erythema toxicum neonatorum is a benign rash of unknown cause that consists of blotchy red areas.

10. The parents of a newborn ask the nurse how much the newborn can see. The parents specifically want to know what type of visual stimuli they should provide for their newborn. What information provided by the nurse would be most useful to these new parents? a. Infants can see very little until approximately 3 months of age. b. Infants can track their parents eyes and can distinguish patterns; they prefer complex patterns. c. The infants eyes must be protected. Infants enjoy looking at brightly colored stripes. d. Its important to shield the newborns eyes. Overhead lights help them see better.

ANS: B Telling the parents that infants can track their parents eyes and can distinguish patterns is an accurate statement. Development of the visual system continues for the first 6 months of life. Visual acuity is difficult to determine, but the clearest visual distance for the newborn appears to be 19 cm. Infants prefer to look at complex patterns, regardless of the color. They prefer low illumination and withdraw from bright lights.

9. A primiparous woman is watching her newborn sleep. She wants him to wake up and respond to her. The mother asks the nurse how much he will sleep every day. What is an appropriate response by the nurse? a. He will only wake up to be fed, and you should not bother him between feedings. b. The newborn sleeps approximately 17 hours a day, with periods of wakefulness gradually increasing. c. He will probably follow your same sleep and wake patterns, and you can expect him to be awake soon. d. He is being stubborn by not waking up when you want him to. You should try to keep him awake during the daytime so that he will sleep through the night.

ANS: B Telling the woman that the newborn sleeps approximately 17 hours a day with periods of wakefulness that gradually increase is both accurate and the most appropriate response by the nurse. Periods of wakefulness are dictated by hunger, but the need for socializing also appears. Telling the woman that her infant is stubborn and should be kept awake during the daytime is an inappropriate nursing response.

Which nursing intervention is paramount when providing care to a client with preterm labor who has received terbutaline? a.Assess deep tendon reflexes (DTRs). b.Assess for dyspnea and crackles. c.Assess for bradycardia. d.Assess for hypoglycemia.

ANS: B Terbutaline is a beta2-adrenergic agonist that affects the mother's cardiopulmonary and metabolic systems. Signs of cardiopulmonary decompensation include adventitious breath sounds and dyspnea. An assessment for dyspnea and crackles is important for the nurse to perform if the woman is taking magnesium sulfate. Assessing DTRs does not address the possible respiratory side effects of using terbutaline. Since terbutaline is a beta2-adrenergic agonist, it can lead to hyperglycemia, not hypoglycemia. Beta2-adrenergic agonist drugs cause tachycardia, not bradycardia.

11. The lactational amenorrhea method (LAM) of birth control is popular in developing countries and has had limited use in the United States. As breastfeeding rates increase, more women may rely upon this method for birth control. Which information is most important to provide to the client interested in using the LAM for contraception? a.LAM is effective until the infant is 9 months of age. b.This popular method of birth control works best if the mother is exclusively breastfeeding. c.Its typical failure rate is 5%. d.Feeding intervals should be 6 hours during the day.

ANS: B The LAM works best if the mother is exclusively or almost exclusively breastfeeding. Disruption of the breastfeeding pattern increases the risk of pregnancy. After the infant is 6 months of age or menstrual flow has resumed, effectiveness decreases. The typical failure rate is 1% to 2%. Feeding intervals should be no greater than 4 hours during the day and 6 hours at night. DIF: Cognitive Level: Understand REF: p. 191

16. Which information about variations in the infants blood counts is important for the nurse to explain to the new parents? a. A somewhat lower-than-expected red blood cell count could be the result of a delay in clamping the umbilical cord. b. An early high white blood cell (WBC) count is normal at birth and should rapidly decrease. c. Platelet counts are higher in the newborn than in adults for the first few months. d. Even a modest vitamin K deficiency means a problem with the bloods ability to properly clot.

ANS: B The WBC count is normally high on the first day of birth and then rapidly declines. Delayed cord clamping results in an increase in hemoglobin and the red blood cell count. The platelet count is essentially the same for newborns and adults. Clotting is sufficient to prevent hemorrhage unless the deficiency of vitamin K is significant.

15. Which benefit regarding FAMs makes it an appealing choice for some women? a.Adherence to strict recordkeeping b.Absence of chemicals and hormones c.Decreased involvement and intimacy of partner d.Increased spontaneity of coitus

ANS: B The absence of chemicals or hormones to alter the natural menstrual flow is extremely important to some women. The strict recordkeeping with FAMs may be difficult and creates a potential risk for failure. These methods require increased involvement by the partner; however, they also reduce the spontaneity of coitus.

1. A woman gave birth to a healthy 7-pound, 13-ounce infant girl. The nurse suggests that the client place the infant to her breast within 15 minutes after birth. The nurse is aware that the initiation of breastfeeding is most effective during the first 30 minutes after birth. What is the correct term for this phase of alertness? a. Transition period b. First period of reactivity c. Organizational stage d. Second period of reactivity

ANS: B The first period of reactivity is the first phase of transition and lasts up to 30 minutes after birth. The infant is highly alert during this phase. The transition period is the phase between intrauterine and extrauterine existence. An organizational stage is not a valid stage. The second period of reactivity occurs approximately between 4 and 8 hours after birth, after a period of sleep.

What is the primary nursing responsibility when caring for a client who is experiencing an obstetric hemorrhage associated with uterine atony? a.Establishing venous access b.Performing fundal massage c.Preparing the woman for surgical intervention d.Catheterizing the bladder

ANS: B The initial management of excessive postpartum bleeding is a firm massage of the uterine fundus. Although establishing venous access may be a necessary intervention, fundal massage is the initial intervention. The woman may need surgical intervention to treat her postpartum hemorrhage, but the initial nursing intervention is to assess the uterus. After uterine massage, the nurse may want to catheterize the client to eliminate any bladder distention that may be preventing the uterus from properly contracting.

20. Importantly, the nurse must be aware of which information related to the use of IUDs? a.Return to fertility can take several weeks after the device is removed. b.IUDs containing copper can provide an emergency contraception option if inserted within a few days of unprotected intercourse. c.IUDs offer the same protection against STIs as the diaphragm. d.Consent forms are not needed for IUD insertion.

ANS: B The woman has up to 5 days to insert the IUD after unprotected sex. The return to fertility is immediate after the removal of the IUD. IUDs offer no protection against STIs. A consent form is required for insertion, as is a negative pregnancy test.

A pregnant woman at 29 weeks of gestation has been diagnosed with preterm labor. Her labor is being controlled with tocolytic medications. She asks when she might be able to go home. Which response by the nurse is most accurate? a."After the baby is born." b."When we can stabilize your preterm labor and arrange home health visits." c."Whenever your physician says that it is okay." d."It depends on what kind of insurance coverage you have."

ANS: B This client's preterm labor is being controlled with tocolytics. Once she is stable, home care may be a viable option for this type of client. Care of a client with preterm labor is multidisciplinary and multifactorial; the goal is to prevent delivery. In many cases, this goal may be achieved at home. Managed care may dictate an earlier hospital discharge or a shift from hospital to home care. Insurance coverage may be one factor in client care, but ultimately, client safety remains the most important factor.

Which clinical finding is a major use of ultrasonography in the first trimester? a.Amniotic fluid volume b.Presence of maternal abnormalities c.Placental location and maturity d.Cervical length

ANS: B Ultrasonography can detect certain uterine abnormalities such as bicornuate uterus, fibroids, and ovarian cysts. Amniotic fluid volume, placental location and maturity, and cervical length are not available via ultrasonography until the second or third trimester.

What would a steady trickle of bright red blood from the vagina in the presence of a firm fundus suggest to the nurse? a.Uterine atony b.Lacerations of the genital tract c.Perineal hematoma d.Infection of the uterus

ANS: B Undetected lacerations will bleed slowly and continuously. Bleeding from lacerations is uncontrolled by uterine contraction. The fundus is not firm in the presence of uterine atony. A hematoma would develop internally. Swelling and discoloration would be noticeable; however, bright bleeding would not be. With an infection of the uterus, an odor to the lochia and systemic symptoms such as fever and malaise would be present.

Which nursing intervention is necessary before a first-trimester transabdominal ultrasound? a.Place the woman on nothing by mouth (nil per os [NPO]) for 12 hours. b.Instruct the woman to drink 1 to 2 quarts of water. c.Administer an enema. d.Perform an abdominal preparation

ANS: B When the uterus is still in the pelvis, visualization may be difficult. Performing a first-trimester transabdominal ultrasound requires the woman to have a full bladder, which will elevate the uterus upward and provide a better visualization of the fetus; therefore, being NPO is not appropriate. Neither an enema nor an abdominal preparation is necessary for this procedure.

30. Which cardiovascular changes cause the foramen ovale to close at birth? a. Increased pressure in the right atrium b. Increased pressure in the left atrium c. Decreased blood flow to the left ventricle d. Changes in the hepatic blood flow

ANS: B With the increase in the blood flow to the left atrium from the lungs, the pressure is increased, and the foramen ovale is functionally closed. The pressure in the right atrium decreases at birth and is higher during fetal life. Blood flow increases to the left ventricle after birth. The hepatic blood flow changes but is not the reason for the closure of the foramen ovale.

16. With regard to rubella and Rh issues, nurses should be aware that: a. Breastfeeding mothers cannot be vaccinated with the live attenuated rubella virus b. Women should be warned that the rubella vaccination is teratogenic and they must avoid pregnancy for at least 1 month after vaccination c. Rh immune globulin is safely administered intravenously because it cannot harm a nursing infant d. Rh immune globulin boosts the immune system and thereby enhances the effectiveness of vaccinations

ANS: B Women should understand they must practice contraception for at least 1 month after being vaccinated. Because the live attenuated rubella virus is not communicable in breast milk, breastfeeding mothers can be vaccinated. Rh immune globulin is administered IM; it should never be given to an infant. Rh immune globulin suppresses the immune system and therefore might thwart the rubella vaccination.

1. What are the various modes of heat loss in the newborn? (Select all that apply.) a. Perspiration b. Convection c. Radiation d. Conduction e. Urination

ANS: B, C, D Convection, radiation, evaporation, and conduction are the four modes of heat loss in the newborn.

Indications for a primary cesarean birth are often nonrecurring. Therefore, a woman who has had a cesarean birth with a low transverse scar may be a candidate for vaginal birth after cesarean (VBAC). Which clients would beless likely to have a successful VBAC? (Select all that apply.) aLengthy interpregnancy interval b.African-American race c.Delivery at a rural hospital d.Estimated fetal weight <4000 g e.Maternal obesity (BMI >30)

ANS: B, C, E Indications for a low success rate for a VBAC delivery include a short interpregnancy interval, non-Caucasian race, gestational age longer than 40 weeks, maternal obesity, preeclampsia, fetal weight greater than 4000 g, and delivery at a rural or private hospital.

Postabortion instructions may differ among providers regarding tampon use and the resumption of intercourse. However, education should be provided regarding serious complications. When should the woman who has undergone an induced abortion be instructed to return to the emergency department? (Select all that apply.) a. Fever higher than 39 C b. Chills c. Foul-smelling vaginal discharge d. Bleeding greater than four pads in 2 hours e. Severe abdominal pain

ANS: B, C, E The client should report to a health care facility for any of the following symptoms: fever higher than 38 C, chills, bleeding more than two saturated pads in 2 hours or heavy bleeding lasting for days, foul-smelling discharge, abdominal tenderness or pain, and cramping or backache. DIF: Cognitive Level: Apply REF: p. 193

The client and her partner are considering male sterilization as a form of permanent birth control. While educating the client regarding the risks and benefits of the procedure, which information should the nurse include? (Select all that apply.) a. Sterilization should be performed under general anesthesia. b. Pain, bleeding, and infection are possible complications. c. Pregnancy may still be possible. d. Vasectomy may affect potency. e. Secondary sex characteristics are unaffected.

ANS: B, C, E Vasectomy is the most commonly used procedure for male sterilization and is performed on an outpatient basis under local anesthesia. Pain, bleeding, swelling, and infection are considered complications. Reversal is generally unsuccessful; however, it may take several weeks to months for all sperm to be cleared from the sperm ducts. Another form of contraception is necessary until the sperm counts are zero. Vasectomy has no effect on potency, and secondary sex characteristics are not affected. DIF: Cognitive Level: Understand REF: p. 190

The nurse recognizes that uterine hyperstimulation with oxytocin requires emergency interventions. What clinical cues alert the nurse that the woman is experiencing uterine hyperstimulation? (Select all that apply.) a.Uterine contractions lasting <90 seconds and occurring >2 minutes in frequency b.Uterine contractions lasting >90 seconds and occurring <2 minutes in frequency c.Uterine tone <20 mm Hg d.Uterine tone >20 mm Hg e.Increased uterine activity accompanied by a nonreassuring FHR and pattern

ANS: B, D, E Uterine contractions that occur less frequently than 2 minutes apart and last longer than 90 seconds, a uterine tone over 20 mm Hg, and a nonreassuring FHR and pattern are indications of uterine hyperstimulation with oxytocin administration. Uterine contractions that occur more frequently than 2 minutes apart and last less than 90 seconds are the expected goal of oxytocin induction. A uterine tone less than 20 mm Hg is normal.

In the past, factors to determine whether a woman was likely to develop a high-risk pregnancy were primarily evaluated from a medical point of view. A broader, more comprehensive approach to high-risk pregnancy has been adopted today. Four categories have now been established, based on the threats to the health of the woman and the outcome of pregnancy. Which category should not be included in this group? a.Biophysical b.Psychosocial c.Geographic d.Environmental

ANS: C A geographic category is correctly referred to as sociodemographic risk. These factors stem from the mother and her family. Ethnicity may be one of the risks to pregnancy; however, it is not the only factor in this category. Low income, lack of prenatal care, age, parity, and marital status also are included. Biophysical is one of the broad categories used for determining risk. These include genetic considerations, nutritional status, and medical and obstetric disorders. Psychosocial risks include smoking, caffeine, drugs, alcohol, and psychologic status. All of these adverse lifestyles can have a negative effect on the health of the mother or fetus. Environmental risks are risks that can affect both fertility and fetal development. These include infections, chemicals, radiation, pesticides, illicit drugs, and industrial pollutants.

5. Which client would be an ideal candidate for injectable progestins such as Depo-Provera (DMPA) as a contraceptive choice? a.The ideal candidate for DMPA wants menstrual regularity and predictability. b.The client has a history of thrombotic problems or breast cancer. c.The ideal candidate has difficulty remembering to take oral contraceptives daily. d.The client is homeless or mobile and rarely receives health care.

ANS: C Advantages of DMPA include its contraceptive effectiveness, compared with the effectiveness of combined oral contraceptives, and the requirement of only four injections a year. The disadvantages of injectable progestins are prolonged amenorrhea and uterine bleeding. The use of injectable progestin carries an increased risk of venous thrombosis and thromboembolism. To be effective, DMPA injections must be administered every 11 to 13 weeks. Access to health care is necessary to prevent pregnancy or potential complications.

At 35 weeks of pregnancy, a woman experiences preterm labor. Although tocolytic medications are administered and she is placed on bed rest, she continues to experience regular uterine contractions and her cervix is beginning to dilate and efface. What is an important test for fetal well-being at this time? a.PUBS b.Ultrasound for fetal size c.Amniocentesis for fetal lung maturity d.NST

ANS: C Amniocentesis is performed to assess fetal lung maturity in the event of a preterm birth. The fluid is examined to determine the lecithin to sphingomyelin (L/S) ratio. Indications for PUBS include prenatal diagnosis or inherited blood disorders, karyotyping of malformed fetuses, detection of fetal infection, determination of the acid-base status of the fetus with IUGR, and assessment and treatment of isoimmunization and thrombocytopenia in the fetus. Determination of fetal size by ultrasound is typically performed during the second trimester and is not indicated in this scenario. An NST measures the fetal response to fetal movement in a noncontracting mother.

A woman with worsening preeclampsia is admitted to the hospital's labor and birth unit. The physician explains the plan of care for severe preeclampsia, including the induction of labor, to the woman and her husband. Which statement by the husband leads the nurse to believe that the couple needs further information? a."I will help my wife use the breathing techniques that we learned in our childbirth classes." b."I will give my wife ice chips to eat during labor." c."Since we will be here for a while, I will call my mother so she can bring the two boys—2 years and 4 years of age—to visit their mother." d."I will stay with my wife during her labor, just as we planned."

ANS: C Arranging a visit with their two children indicates that the husband does not understand the importance of the quiet, subdued environment that is needed to prevent his wife's condition from worsening. Implementing breathing techniques is indicative of adequate knowledge related to pain management during labor. Administering ice chips indicates an understanding of nutritional needs during labor. Staying with his wife during labor demonstrates the husband's support for his wife and is appropriate.

A client in the third trimester has just undergone an amniocentesis to determine fetal lung maturity. Which statement regarding this testing is important for the nurse in formulating a care plan? a.Because of new imaging techniques, an amniocentesis should have been performed in the first trimester. b.Despite the use of ultrasonography, complications still occur in the mother or infant in 5% to 10% of cases. c.Administration of Rho(D) immunoglobulin may be necessary. d.The presence of meconium in the amniotic fluid is always a cause for concern.

ANS: C As a result of the possibility of fetomaternal hemorrhage, administration of Rho(D) immunoglobulin is the standard of practice after amniocentesis for women who are Rh negative. Amniocentesis is possible after the 14th week of pregnancy when the uterus becomes an abdominal organ. Complications occur in less than 1% of cases; many have been minimized or eliminated through the use of ultrasonography. Meconium in the amniotic fluid before the beginning of labor is not usually a problem.

25. The nurse should be cognizant of which important information regarding the gastrointestinal (GI) system of the newborn? a. The newborns cheeks are full because of normal fluid retention. b. The nipple of the bottle or breast must be placed well inside the babys mouth because teeth have been developing in utero, and one or more may even be through. c. Regurgitation during the first day or two can be reduced by burping the infant and slightly elevating the babys head. d. Bacteria are already present in the infants GI tract at birth because they traveled through the placenta.

ANS: C Avoiding overfeeding can also reduce regurgitation. The newborns cheeks are full because of well-developed sucking pads. Teeth do develop in utero, but the nipple is placed deep because the baby cannot move food from the lips to the pharynx. Bacteria are not present at birth, but they soon enter through various orifices.

What is the primary purpose for the use of tocolytic therapy to suppress uterine activity? a.Drugs can be efficaciously administered up to the designated beginning of term at 37 weeks gestation. b.Tocolytic therapy has no important maternal (as opposed to fetal) contraindications. c.The most important function of tocolytic therapy is to provide the opportunity to administer antenatal glucocorticoids. d.If the client develops pulmonary edema while receiving tocolytic therapy, then intravenous (IV) fluids should be given.

ANS: C Buying time for antenatal glucocorticoids to accelerate fetal lung development may be the best reason to use tocolytic therapy. Once the pregnancy has reached 34 weeks, however, the risks of tocolytic therapy outweigh the benefits. Important maternal contraindications to tocolytic therapy exist. Tocolytic-induced edema can be caused by IV fluids.

While working with the pregnant client in her first trimester, what information does the nurse provide regarding when CVS can be performed (in weeks of gestation)? a.4 b.8 c.10 d.14

ANS: C CVS can be performed in the first or second trimester, ideally between 10 and 13 weeks of gestation. During this procedure, a small piece of tissue is removed from the fetal portion of the placenta. If performed after 9 completed weeks of gestation, then the risk of limb reduction is no greater than in the general population.

Which is the initial treatment for the client with vWD who experiences a PPH? a.Cryoprecipitate b.Factor VIII and von Willebrand factor (vWf) c.Desmopressin d.Hemabate

ANS: C Desmopressin is the primary treatment of choice for vWD and can be administered orally, nasally, and intravenously. This medication promotes the release of factor VIII and vWf from storage. Cryoprecipitate may be used; however, because of the risk of possible donor viruses, other modalities are considered safer. Treatment with plasma products such as factor VIII and vWf is an acceptable option for this client. Because of the repeated exposure to donor blood products and possible viruses, this modality is not the initial treatment of choice. Although the administration of the prostaglandin, Hemabate, is known to promote contraction of the uterus during PPH, it is not effective for the client who has a bleeding disorder.

11. While assessing the integument of a 24-hour-old newborn, the nurse notes a pink papular rash with vesicles superimposed on the thorax, back, and abdomen. What action is the highest priority for the nurse to take at this time? a. Immediately notify the physician. b. Move the newborn to an isolation nursery. c. Document the finding as erythema toxicum neonatorum. d. Take the newborns temperature, and obtain a culture of one of the vesicles.

ANS: C Erythema toxicum neonatorum (or erythema neonatorum) is a newborn rash that resembles flea bites. Notification of the physician, isolation of the newborn, or additional interventions are not necessary when erythema toxicum neonatorum is present.

. If consistently and correctly used, which of the barrier methods of contraception has the lowest failure rate? a. Spermicides b. Female condoms c. Male condoms d. Diaphragms

ANS: C For typical users, the failure rate for male condoms may approach 18%. Spermicide failure rates are approximately 28%. The failure rate for female condoms is approximately 21%. The failure rate for diaphragms with spermicides is 12%. DIF: Cognitive Level: Remember REF: p. 179 TOP: Nursing Process: Planning

The labor of a pregnant woman with preeclampsia is going to be induced. Before initiating the oxytocin (Pitocin) infusion, the nurse reviews the woman's latest laboratory test findings, which reveal a platelet count of 90,000 mm3, an elevated aspartate aminotransaminase (AST) level, and a falling hematocrit. The laboratory results are indicative of which condition? a.Eclampsia b.Disseminated intravascular coagulation (DIC) syndrome c.Hemolysis, elevated liver enzyme levels, and low platelet levels (HELLP) syndrome d.Idiopathic thrombocytopenia

ANS: C HELLP syndrome is a laboratory diagnosis for a variant of severe preeclampsia that involves hepatic dysfunction characterized by hemolysis (H), elevated liver (EL) enzymes, and low platelets (LP). Eclampsia is determined by the presence of seizures. DIC is a potential complication associated with HELLP syndrome. Idiopathic thrombocytopenia is the presence of low platelets of unknown cause and is not associated with preeclampsia.

4. A woman who has just undergone a first-trimester abortion will be using oral contraceptives. To protect against pregnancy, the client should be advised to do what? a.Avoid sexual contact for at least 10 days after starting the pill. b.Use condoms and foam for the first few weeks as a backup. c.Use another method of contraception for 1 week after starting the pill. d.Begin sexual relations once vaginal bleeding has ended.

ANS: C If oral contraceptives are to be started within 3 weeks after an abortion, additional forms of contraception should be used throughout the first week to avoid the risk of pregnancy. DIF: Cognitive Level: Apply REF: p. 193 TOP: Nursing Process: Planning

The indirect Coombs' test is a screening tool for Rh incompatibility. If the titer is greater than ______, amniocentesis may be a necessary next step. a.1:2 b.1:4 c.1:8 d.1:12

ANS: C If the maternal titer for Rh antibodies is greater 1:8, then an amniocentesis is indicated to determine the level of bilirubin in the amniotic fluid. This testing will determine the severity of fetal hemolytic anemia.

17. A recently delivered mother and her baby are at the clinic for a 6-week postpartum checkup. The nurse should be concerned that psychosocial outcomes are not being met if the woman: a. Discusses her labor and birth experience excessively b. Feels that her baby is more attractive and clever than any others c. Has not given the baby a name d. Has a partner or family members who react very positively about the baby

ANS: C If the mother is having difficulty naming her new infant, it may be a signal that she is not adapting well to parenthood. Other red flags include refusal to hold or feed the baby, lack of interaction with the infant, and becoming upset when the baby vomits or needs a diaper change. A new mother who is having difficulty is unwilling to discuss her labor and birth experience. An appropriate nursing diagnosis might be impaired parenting related to a long, difficult labor or unmet expectations of birth. A mother who is willing to discuss her birth experience is making a healthy personal adjustment. The mother who is not coping well finds her baby unattractive and messy. She may also be overly disappointed in the baby's sex. The client might voice concern that the baby reminds her of a family member whom she does not like. Having a partner and/or other family members react positively is an indication that this new mother has a good support system in place. This support system helps reduce anxiety related to her new role as a mother.

14. In the acronym BRAIDED, which letter is used to identify the key components of informed consent that the nurse must document? a.B stands for birth control. b.R stands for reproduction. c.A stands for alternatives. d.I stands for ineffective.

ANS: C In the acronym BRAIDED, A stands for alternatives and information about other viable methods. B stands for benefits and information about the advantages of a particular birth control method and its success rates. Rstands for risks and information about the disadvantages of a particular method and its failure rates. I stands for inquiries and the opportunity to ask questions.

What is the primary purpose for magnesium sulfate administration for clients with preeclampsia and eclampsia? a.To improve patellar reflexes and increase respiratory efficiency b.To shorten the duration of labor c.To prevent convulsions d.To prevent a boggy uterus and lessen lochial flow

ANS: C Magnesium sulfate is the drug of choice used to prevent convulsions, although it can generate other problems. Loss of patellar reflexes and respiratory depression are signs of magnesium toxicity. Magnesium sulfate can also increase the duration of labor. Women are at risk for a boggy uterus and heavy lochial flow as a result of magnesium sulfate therapy.

33. How would the nurse differentiate a meconium stool from a transitional stool in the healthy newborn? a. Observed at age 3 days b. Is residue of a milk curd c. Passes in the first 12 hours of life d. Is lighter in color and looser in consistency

ANS: C Meconium stool is usually passed in the first 12 hours of life, and 99% of newborns have their first stool within 48 hours. If meconium is not passed by 48 hours, then obstruction is suspected. Meconium stool is the first stool of the newborn and is made up of matter remaining in the intestines during intrauterine life. Meconium is dark and sticky.

16. The nurse is providing contraceptive instruction to a young couple who are eager to learn. The nurse should be cognizant of which information regarding the natural family planning method? a.The natural family planning method is the same as coitus interruptus or pulling out. b.This contraception method uses the calendar method to align the womans cycle with the natural phases of the moon. c.This practice is the only contraceptive method acceptable to the Roman Catholic Church. d.The natural family planning method relies on barrier methods during the fertility phases.

ANS: C Natural family planning is the only contraceptive practice acceptable to the Roman Catholic Church. Pulling out is not the same as periodic abstinence, another name for natural family planning. The phases of the moon are not part of the calendar method or any method. Natural family planning is another name for periodic abstinence, which is the accepted way to pass safely through the fertility phases without relying on chemical or physical barriers. DIF: Cognitive Level: Understand REF: p. 173 TOP: Nursing Process: Planning

Women with mild gestational hypertension and mild preeclampsia can be safely managed at home with frequent maternal and fetal evaluation. Complete or partial bed rest is still frequently ordered by some providers. Which complication is rarely the result of prolonged bed rest? a.Thrombophlebitis b.Psychologic stress c.Fluid retention d.Cardiovascular deconditioning

ANS: C No evidence has been found that supports the practice of bed rest to improve pregnancy outcome. Fluid retention is not an adverse outcome of prolonged bed rest. The woman is more likely to experience diuresis with accompanying fluid and electrolyte imbalance and weight loss. Prolonged bed rest is known to increase the risk for thrombophlebitis. Psychologic stress is known to begin on the first day of bed rest and continue for the duration of the therapy. Therefore, restricted activity, rather than complete bed rest, is recommended. Cardiovascular deconditioning is a known complication of bed rest.

Which information should nurses provide to expectant mothers when teaching them how to evaluate daily fetal movement counts (DFMCs)? a.Alcohol or cigarette smoke can irritate the fetus into greater activity. b.Kick counts should be taken every hour and averaged every 6 hours, with every other 6-hour stretch off. c.The fetal alarm signal should go off when fetal movements stop entirely for 12 hours. d.A count of less than four fetal movements in 1 hour warrants future evaluation.

ANS: C No movement in a 12-hour period is cause for investigation and possibly intervention. Alcohol and cigarette smoke temporarily reduce fetal movement. The mother should count fetal activity (kick counts) two or three times daily for 60 minutes each time. A count of less than 3 in 1 hour warrants further evaluation by a NST.

A woman who has recently given birth complains of pain and tenderness in her leg. On physical examination, the nurse notices warmth and redness over an enlarged, hardened area. Which condition should the nurse suspect, and how will it be confirmed? a.Disseminated intravascular coagulation (DIC); asking for laboratory tests b.von Willebrand disease (vWD); noting whether bleeding times have been extended c.Thrombophlebitis; using real-time and color Doppler ultrasound d.Idiopathic or immune thrombocytopenic purpura (ITP); drawing blood for laboratory analysis

ANS: C Pain and tenderness in the extremities, which show warmth, redness, and hardness, is likely thrombophlebitis. A Doppler ultrasound examination is a common noninvasive way to confirm the diagnosis. A diagnosis of DIC is made according to clinical findings and laboratory markers. With DIC, a physical examination will reveal symptoms that may include unusual bleeding, petechiae around a blood pressure cuff on the woman's arm, and/or excessive bleeding from the site of a slight trauma such as a venipuncture site. Symptoms of vWD, a type of hemophilia, include recurrent bleeding episodes, prolonged bleeding time, and factor VIII deficiency. A risk for PPH exists with vWD but does not exhibit a warm or reddened area in an extremity. ITP is an autoimmune disorder in which the life span of antiplatelet antibodies is decreased. Increased bleeding time is a diagnostic finding, and the risk of postpartum uterine bleeding is increased.

20. What marks on a babys skin may indicate an underlying problem that requires notification of a physician? a. Mongolian spots on the back b. Telangiectatic nevi on the nose or nape of the neck c. Petechiae scattered over the infants body d. Erythema toxicum neonatorum anywhere on the body

ANS: C Petechiae (bruises) scattered over the infants body should be reported to the pediatrician because they may indicate underlying problems. Mongolian spots are bluish-black spots that resemble bruises but gradually fade over months and have no clinical significance. Telangiectatic nevi (stork bites, angel kisses) fade by the second year and have no clinical significance. Erythema toxicum neonatorum is an appalling-looking rash; however, it has no clinical significance and requires no treatment.

29. A first-time dad is concerned that his 3-day-old daughters skin looks yellow. In the nurses explanation of physiologic jaundice, what fact should be included? a. Physiologic jaundice occurs during the first 24 hours of life. b. Physiologic jaundice is caused by blood incompatibilities between the mother and the infant blood types. c. Physiologic jaundice becomes visible when serum bilirubin levels peak between the second and fourth days of life. d. Physiologic jaundice is also known as breast milk jaundice.

ANS: C Physiologic jaundice becomes visible when the serum bilirubin reaches a level of 5 mg/dl or higher when the baby is approximately 3 days old. This finding is within normal limits for the newborn. Pathologic jaundice, not physiologic jaundice, occurs during the first 24 hours of life and is caused by blood incompatibilities that result in excessive destruction of erythrocytes; this condition must be investigated. Breast milk jaundice occurs in one third of breastfed infants at 2 weeks and is caused by an insufficient intake of fluids.

Prostaglandin gel has been ordered for a pregnant woman at 43 weeks of gestation. What is the primary purpose of prostaglandin administration? a.To enhance uteroplacental perfusion in an aging placenta b.To increase amniotic fluid volume c.To ripen the cervix in preparation for labor induction d.To stimulate the amniotic membranes to rupture

ANS: C Preparations of prostaglandin E1 and E2 are effective when used before labor induction to ripen (i.e., soften and thin) the cervix. Uteroplacental perfusion is not altered by the use of prostaglandins. The insertion of prostaglandin gel has no effect on the level of amniotic fluid. In some cases, women will spontaneously begin laboring after the administration of prostaglandins, thereby eliminating the need for oxytocin. It is not common for a woman's membranes to rupture as a result of prostaglandin use.

In planning for home care of a woman with preterm labor, which concern should the nurse need to address? a.Nursing assessments are different from those performed in the hospital setting. b.Restricted activity and medications are necessary to prevent a recurrence of preterm labor. c.Prolonged bed rest may cause negative physiologic effects. d.Home health care providers are necessary

ANS: C Prolonged bed rest may cause adverse effects such as weight loss, loss of appetite, muscle wasting, weakness, bone demineralization, decreased cardiac output, risk for thrombophlebitis, alteration in bowel functions, sleep disturbance, and prolonged postpartum recovery. Nursing assessments differ somewhat from those performed in the acute care setting, but this concern does not need to be addressed. Restricted activity and medications may prevent preterm labor but not in all women. In addition, the plan of care is individualized to meet the needs of each client. Many women receive home health nurse visits, but care is individualized for each woman

A primigravida is being monitored at the prenatal clinic for preeclampsia. Which finding is of greatest concern to the nurse? a.Blood pressure (BP) increase to 138/86 mm Hg b.Weight gain of 0.5 kg during the past 2 weeks c.Dipstick value of 3+ for protein in her urine d.Pitting pedal edema at the end of the day

ANS: C Proteinuria is defined as a concentration of 1+ or greater via dipstick measurement. A dipstick value of 3+ alerts the nurse that additional testing or assessment should be performed. A 24-hour urine collection is preferred over dipstick testing attributable to accuracy. Generally, hypertension is defined as a BP of 140/90 mm Hg or an increase in systolic pressure of 30 mm Hg or diastolic pressure of 15 mm Hg. Preeclampsia may be demonstrated as a rapid weight gain of more than 2 kg in 1 week. Edema occurs in many normal pregnancies, as well as in women with preeclampsia. Therefore, the presence of edema is no longer considered diagnostic of preeclampsia.

A pregnant woman has been receiving a magnesium sulfate infusion for treatment of severe preeclampsia for 24 hours. On assessment, the nurse finds the following vital signs: temperature 37.3° C, pulse rate 88 beats per minute, respiratory rate 10 breaths per minute, BP 148/90 mm Hg, absent deep tendon reflexes (DTRs), and no ankle clonus. The client complains, "I'm so thirsty and warm." What is the nurse's immediate action? a.To call for an immediate magnesium sulfate level b.To administer oxygen c.To discontinue the magnesium sulfate infusion d.To prepare to administer hydralazine

ANS: C Regardless of the magnesium level, the client is displaying the clinical signs and symptoms of magnesium toxicity. The first action by the nurse should be to discontinue the infusion of magnesium sulfate. In addition, calcium gluconate, the antidote for magnesium, may be administered. Hydralazine is an antihypertensive drug commonly used to treat hypertension in severe preeclampsia. Typically, hydralazine is administered for a systolic BP higher than 160 mm Hg or a diastolic BP higher than 110 mm Hg.

The nurse is teaching a client with preterm premature rupture of membranes (PPROM) regarding self-care activities. Which activities should the nurse include in her teaching? a.Report a temperature higher than 40° C. b.Tampons are safe to use to absorb the leaking amniotic fluid. c.Do not engage in sexual activity. d.Taking frequent tub baths is safe.

ANS: C Sexual activity should be avoided because it may induce preterm labor. A temperature higher than 38° C should be reported. To prevent the risk of infection, tub baths should be avoided and nothing should be inserted into the vagina. Further, foul-smelling vaginal fluid, which may be a sign of infection, should be reported.

15. If a woman is at risk for thrombus and is not ready to ambulate, nurses might intervene by doing all of these interventions except: a. Putting her in antiembolic stockings (TED hose) and/or sequential compression device (SCD) boots b. Having her flex, extend, and rotate her feet, ankles, and legs c. Having her sit in a chair d. Notifying the physician immediately if a positive Homans' sign occurs

ANS: C Sitting immobile in a chair does not help. Bed exercise and prophylactic footwear might. Antiembolic stockings (TED hose) and SCD boots are recommended. Just sitting in a chair will not help. Bed exercises such as these are useful. Just sitting in a chair does not help. A positive Homans' sign (calf muscle pain or warmth, redness, or tenderness) requires the physician's immediate attention.

The most effective and least expensive treatment of puerperal infection is prevention. What is the most important strategy for the nurse to adopt? a.Large doses of vitamin C during pregnancy b.Prophylactic antibiotics c.Strict aseptic technique, including hand washing, by all health care personnel d.Limited protein and fat intake

ANS: C Strict adherence by all health care personnel to aseptic techniques during childbirth and the postpartum period is extremely important and the least expensive measure to prevent infection. Good nutrition to control anemia is a preventive measure. Increased iron intake assists in preventing anemia. Antibiotics may be administered to manage infections; they are not a cost-effective measure to prevent postpartum infection. Limiting protein and fat intake does not help prevent anemia or prevent infection.

6. While examining a newborn, the nurse notes uneven skinfolds on the buttocks and a clunk when performing the Ortolani maneuver. These findings are likely indicative of what? a. Polydactyly b. Clubfoot c. Hip dysplasia d. Webbing

ANS: C The Ortolani maneuver is used to detect the presence of hip dysplasia. Polydactyly is the presence of extra digits. Clubfoot (talipes equinovarus) is a deformity in which the foot turns inward and is fixed in a plantar-flexion position. Webbing, or syndactyly, is a fusing of the fingers or toes.

Which statement most accurately describes the HELLP syndrome? a.Mild form of preeclampsia b.Diagnosed by a nurse alert to its symptoms c.Characterized by hemolysis, elevated liver enzymes, and low platelets d.Associated with preterm labor but not perinatal mortality

ANS: C The acronym HELLP stands for hemolysis (H), elevated liver (EL) enzymes, and low platelets (LP). The HELLP syndrome is a variant of severe preeclampsia and is difficult to identify because the symptoms are not often obvious. The HELLP syndrome must be diagnosed in the laboratory. Preterm labor is greatly increased; therefore, so is perinatal mortality.

3. The nurse is assessing a full term, quiet, and alert newborn. What is the average expected apical pulse range (in beats per minute)? a. 80 to 100 b. 100 to 120 c. 120 to 160 d. 150 to 180

ANS: C The average infant heart rate while awake is 120 to 160 beats per minute. The newborns heart rate may be approximately 85 to 100 beats per minute while sleeping and typically a little higher than 100 to 120 beats per minute when alert but quiet. A heart rate of 150 to 180 beats per minute is typical when the infant cries.

A 30-year-old gravida 3, para 2-0-0-2 is at 18 weeks of gestation. Which screening test should the nurse recommend be ordered for this client? a.BPP b.Chorionic villi sampling c.MSAFP screening d.Screening for diabetes mellitus

ANS: C The biochemical assessment MSAFP test is performed from week 15 to week 20 of gestation (weeks 16 to 18 are ideal). A BPP is a method of biophysical assessment of fetal well-being in the third trimester. Chorionic villi sampling is a biochemical assessment of the fetus that should be performed from the 10th to 12th weeks of gestation. Screening for diabetes mellitus begins with the first prenatal visit.

32. The nurse is cognizant of which information related to the administration of vitamin K? a. Vitamin K is important in the production of red blood cells. b. Vitamin K is necessary in the production of platelets. c. Vitamin K is not initially synthesized because of a sterile bowel at birth. d. Vitamin K is responsible for the breakdown of bilirubin and the prevention of jaundice.

ANS: C The bowel is initially sterile in the newborn, and vitamin K cannot be synthesized until food is introduced into the bowel. Vitamin K is necessary to activate blood-clotting factors. The platelet count in term newborns is near adult levels. Vitamin K is necessary to activate prothrombin and other blood-clotting factors.

A primigravida at 40 weeks of gestation is having uterine contractions every to 2 minutes and states that they are very painful. Her cervix is dilated 2 cm and has not changed in 3 hours. The woman is crying and wants an epidural. What is the likely status of this woman's labor? a.She is exhibiting hypotonic uterine dysfunction. b.She is experiencing a normal latent stage. c.She is exhibiting hypertonic uterine dysfunction. d.She is experiencing precipitous labor.

ANS: C The contraction pattern observed in this woman signifies hypertonic uterine activity. Typically, uterine activity in this phase occurs at 4- to 5-minute intervals lasting 30 to 45 seconds. Women who experience hypertonic uterine dysfunction, or primary dysfunctional labor, are often anxious first-time mothers who are having painful and frequent contractions that are ineffective at causing cervical dilation or effacement to progress. With hypotonic uterine dysfunction, the woman initially makes normal progress into the active stage of labor; then the contractions become weak and inefficient or stop altogether. Precipitous labor is one that lasts less than 3 hours from the onset of contractions until time of birth.

4. A newborn is placed under a radiant heat warmer. The nurse understands that thermoregulation presents a problem for the newborn. What is the rationale for this difficulty? a. The renal function of a newborn is not fully developed, and heat is lost in the urine. b. The small body surface area of a newborn favors more rapid heat loss than does an adults body surface area. c. Newborns have a relatively thin layer of subcutaneous fat that provides poor insulation. d. Their normal flexed posture favors heat loss through perspiration.

ANS: C The newborn has little thermal insulation. Furthermore, the blood vessels are closer to the surface of the skin. Changes in environmental temperature alter the temperature of the blood, thereby influencing temperature regulation centers in the hypothalamus. Heat loss does not occur through urination. Newborns have a higher body surface-to-weight ratio than adults. The flexed position of the newborn helps guard against heat loss, because it diminishes the amount of body surface exposed to the environment.

15. Which information related to the newborns developing cardiovascular system should the nurse fully comprehend? a. The heart rate of a crying infant may rise to 120 beats per minute. b. Heart murmurs heard after the first few hours are a cause for concern. c. The point of maximal impulse (PMI) is often visible on the chest wall. d. Persistent bradycardia may indicate respiratory distress syndrome (RDS).

ANS: C The newborns thin chest wall often allows the PMI to be observed. The normal heart rate for infants who are not sleeping is 120 to 160 beats per minute. However, a crying infant could temporarily have a heart rate of 180 beats per minute. Heart murmurs during the first few days of life have no pathologic significance; however, an irregular heart rate beyond the first few hours should be further evaluated. Persistent tachycardia may indicate RDS; bradycardia may be a sign of congenital heart blockage.

9. Under the Newborns' and Mothers' Health Protection Act, all health plans are required to allow new mothers and newborns to remain in the hospital for a minimum of _____ hours after a normal vaginal birth and for _____ hours after a cesarean birth. a. 24; 72 b. 24; 96 c. 48; 96 d. 48; 120

ANS: C The specified stays are 48 hours (2 days) for a vaginal birth and 96 hours (4 days) for a cesarean birth. The attending provider and the mother together can decide on an earlier discharge. The specified stays are 48 hours (2 days) for a vaginal birth and 96 hours (4 days) for a cesarean birth. A client may be discharged either 24 hours after a vaginal birth or 72 hours after a cesarean birth if she is stable and her provider is in agreement. The specified stays are 48 hours (2 days) for a vaginal birth and 96 hours (4 days) for a cesarean birth. A client may be discharged 24 hours after a vaginal birth if she is stable and her provider is in agreement. The specified stays are 48 hours (2 days) for a vaginal birth and 96 hours (4 days) for a cesarean birth. A client is unlikely to remain in the hospital for 120 hours after a cesarean birth unless there are complications.

The American College of Obstetricians and Gynecologists (ACOG) has developed a comprehensive list of risk factors associated with the development of preeclampsia. Which client exhibits the greatest number of these risk factors? a.30-year-old obese Caucasian with her third pregnancy b.41-year-old Caucasian primigravida c.19-year-old African American who is pregnant with twins d.25-year-old Asian American whose pregnancy is the result of donor insemination

ANS: C Three risk factors are present in the 19-year-old African-American client. She has African-American ethnicity, is at the young end of the age distribution, and has a multiple pregnancy. In planning care for this client, the nurse must frequently monitor her BP and teach her to recognize the early warning signs of preeclampsia. The 30-year-old obese Caucasian client has only has one known risk factor: obesity. Age distribution appears to be U-shaped, with women younger than 20 years of age and women older than 40 years of age being at greatest risk. Preeclampsia continues to be more frequently observed in primigravidas; this client is a multigravida woman. Two risk factors are present for the 41-year-old Caucasian primigravida client. Her age and status as a primigravida place her at increased risk for preeclampsia. Caucasian women are at a lower risk than are African-American women. The 25-year-old Asian-American client exhibits only one risk factor. Pregnancies that result from donor insemination, oocyte donation, and embryo donation are at an increased risk of developing preeclampsia.

10. A woman will be taking oral contraceptives using a 28-day pack. What advice should the nurse provide to protect this client from an unintended pregnancy? a.Limit sexual contact for one cycle after starting the pill. b.Use condoms and foam instead of the pill for as long as the client takes an antibiotic. c.Take one pill at the same time every day. d.Throw away the pack and use a backup method if two pills are missed during week 1 of her cycle.

ANS: C To maintain adequate hormone levels for contraception and to enhance compliance, clients should take oral contraceptives at the same time each day. If contraceptives are to be started at any time other than during normal menses or within 3 weeks after birth or an abortion, then another method of contraception should be used through the first week to prevent the risk of pregnancy. Taken exactly as directed, oral contraceptives prevent ovulation, and pregnancy cannot occur. No strong pharmacokinetic evidence indicates a link between the use of broad-spectrum antibiotics and altered hormonal levels in oral contraceptive users. If the client misses two pills during week 1, then she should take two pills a day for 2 days and finish the package and use a backup contraceptive method for the next 7 consecutive days. DIF: Cognitive Level: Apply REF: pp. 181-184 TOP: Nursing Process: Planning

Which statement related to cephalopelvic disproportion (CPD) is the least accurate? a.CPD can be related to either fetal size or fetal position. b.The fetus cannot be born vaginally. c.CPD can be accurately predicted. d.Causes of CPD may have maternal or fetal origins.

ANS: C Unfortunately, accurately predicting CPD is not possible. Although CPD is often related to excessive fetal size (macrosomia), malposition of the fetal presenting part is the problem in many cases, not true CPD. When CPD is present, the fetus cannot fit through the maternal pelvis to be born vaginally. CPD may be related to either fetal origins such as macrosomia or malposition or maternal origins such as a too small or malformed pelvis.

A woman is having her first child. She has been in labor for 15 hours. A vaginal examination performed 2 hours earlier revealed the cervix to be dilated to 5 cm and 100% effaced, and the presenting part of the fetus was at station 0; however, another vaginal examination performed 5 minutes ago indicated no changes. What abnormal labor pattern is associated with this description? a.Prolonged latent phase b.Protracted active phase c.Secondary arrest d.Protracted descent

ANS: C With a secondary arrest of the active phase, the progress of labor has stopped. This client has not had any anticipated cervical change, indicating an arrest of labor. In the nulliparous woman, a prolonged latent phase typically lasts longer than 20 hours. A protracted active phase, the first or second stage of labor, is prolonged (slow dilation). With a protracted descent, the fetus fails to descend at an anticipated rate during the deceleration phase and second stage of labor.

5. An African-American woman noticed some bruises on her newborn daughters buttocks. The client asks the nurse what causes these. How would the nurse best explain this integumentary finding to the client? a. Lanugo b. Vascular nevus c. Nevus flammeus d. Mongolian spot

ANS: D A Mongolian spot is a bluish-black area of pigmentation that may appear over any part of the exterior surface of the infants body and is more commonly noted on the back and buttocks and most frequently observed on infants whose ethnic origins are Mediterranean, Latin American, Asian, or African. Lanugo is the fine, downy hair observed on a term newborn. A vascular nevus, commonly called a strawberry mark, is a type of capillary hemangioma. A nevus flammeus, commonly called a port wine stain, is most frequently found on the face.

7. Which finding could prevent early discharge of a newborn who is now 12 hours old? a. Birth weight of 3000 g b. One meconium stool since birth c. Voided, clear, pale urine three times since birth d. Infant breastfed once with some difficulty with latch and sucking and once with some success for about 5 minutes on each breast

ANS: D An infant needs to complete at least two successful feedings (normal sucking and swallowing) before an early discharge. Birth weight of 3000 g is a normal infant finding that does not prevent early discharge. Passage of one meconium stool is a normal infant finding that does not prevent early discharge. Having voided three times since birth is a normal infant finding that does not prevent early discharge.

What is the importance of obtaining informed consent for a number of contraceptive methods? a. Contraception is an invasive procedure that requires hospitalization. b. The method may require a surgical procedure to insert a device. c. The contraception method chosen may be unreliable. d. The method chosen has potentially dangerous side effects.

ANS: D Being aware of the potential side effects is important for couples who are making an informed decision about the use of contraceptives. The only contraceptive method that is a surgical procedure and requires hospitalization is sterilization. Some methods have greater efficacy than others, and this efficacy should be included in the teaching. DIF: Cognitive Level: Understand REF: p. 172

A woman at 26 weeks of gestation is being assessed to determine whether she is experiencing preterm labor. Which finding indicates that preterm labor is occurring? a.Estriol is not found in maternal saliva. b.Irregular, mild uterine contractions are occurring every 12 to 15 minutes. c.Fetal fibronectin is present in vaginal secretions. d.The cervix is effacing and dilated to 2 cm.

ANS: D Cervical changes such as shortened endocervical length, effacement, and dilation are predictors of imminent preterm labor. Changes in the cervix accompanied by regular contractions indicate labor at any gestation. Estriol is a form of estrogen produced by the fetus that is present in plasma at 9 weeks of gestation. Levels of salivary estriol have been shown to increase before preterm birth. Irregular, mild contractions that do not cause cervical change are not considered a threat. The presence of fetal fibronectin in vaginal secretions between 24 and 36 weeks of gestation could predict preterm labor, but it has only a 20% to 40% positive predictive value. Of more importance are other physiologic clues of preterm labor such as cervical changes.

Which statement regarding the term contraceptive failure rate is the most accurate? a.The contraceptive failure rate refers to the percentage of users expected to have an accidental pregnancy over a 5-year span. b.It refers to the minimum rate that must be achieved to receive a government license. c.The contraceptive failure rate increases over time as couples become more careless. d.It varies from couple to couple, depending on the method and the users.

ANS: D Contraceptive effectiveness varies from couple to couple, depending on how well a contraceptive method is used and how well it suits the couple. The contraceptive failure rate measures the likelihood of accidental pregnancy in the first year only. Failure rates decline over time because users gain experience.

If nonsurgical treatment for late PPH is ineffective, which surgical procedure would be appropriate to correct the cause of this condition? a.Hysterectomy b.Laparoscopy c.Laparotomy d.Dilation and curettage (D&C)

ANS: D D&C allows the examination of the uterine contents and the removal of any retained placental fragments or blood clots. Hysterectomy is the removal of the uterus and is not the appropriate treatment for late PPH. A laparoscopy is the insertion of an endoscope through the abdominal wall to examine the peritoneal cavity, but it, too, is not the appropriate treatment for this condition. A laparotomy is the surgical incision into the peritoneal cavity to explore it but is also not the appropriate treatment for late PPH.

The nurse has evaluated a client with preeclampsia by assessing DTRs. The result is a grade of 3+. Which DTR response most accurately describes this score? a.Sluggish or diminished b.Brisk, hyperactive, with intermittent or transient clonus c.Active or expected response d.More brisk than expected, slightly hyperactive

ANS: D DTRs reflect the balance between the cerebral cortex and the spinal cord. They are evaluated at baseline and to detect changes. A slightly hyperactive and brisk response indicates a grade 3+ response.

Nurses need to understand the basic definitions and incidence data regarding PPH. Which statement regarding this condition is most accurate? a.PPH is easy to recognize early; after all, the woman is bleeding. b.Traditionally, it takes more than 1000 ml of blood after vaginal birth and 2500 ml after cesarean birth to define the condition as PPH. c.If anything, nurses and physicians tend to overestimate the amount of blood loss. d.Traditionally, PPH has been classified as early PPH or late PPH with respect to birth.

ANS: D Early PPH is also known as primary, or acute, PPH; late PPH is known as secondary PPH. Unfortunately, PPH can occur with little warning and is often recognized only after the mother has profound symptoms. Traditionally, a 500-ml blood loss after a vaginal birth and a 1000-ml blood loss after a cesarean birth constitute PPH. Medical personnel tend to underestimate blood loss by as much as 50% in their subjective observations.

The nurse is performing an assessment on a client who thinks she may be experiencing preterm labor. Which information is the most important for the nurse to understand and share with the client? a.Because all women must be considered at risk for preterm labor and prediction is so variable, teaching pregnant women the symptoms of preterm labor probably causes more harm through false alarms. b.Braxton Hicks contractions often signal the onset of preterm labor. c.Because preterm labor is likely to be the start of an extended labor, a woman with symptoms can wait several hours before contacting the primary caregiver. d.Diagnosis of preterm labor is based on gestational age, uterine activity, and progressive cervical change.

ANS: D Gestational age of 20 to 37 weeks, uterine contractions, and a cervix that is 80% effaced or dilated 2 cm indicates preterm labor. It is essential that nurses teach women how to detect the early symptoms of preterm labor. Braxton Hicks contractions resemble preterm labor contractions, but they are not true labor. Waiting too long to see a health care provider could result in essential medications failing to be administered. Preterm labor is not necessarily long-term labor.

The client has been on magnesium sulfate for 20 hours for the treatment of preeclampsia. She just delivered a viable infant girl 30 minutes ago. What uterine findings does the nurse expect to observe or assess in this client? a.Absence of uterine bleeding in the postpartum period b.Fundus firm below the level of the umbilicus c.Scant lochia flow d.Boggy uterus with heavy lochia flow

ANS: D High serum levels of magnesium can cause a relaxation of smooth muscle such as the uterus. Because of this tocolytic effect, the client will most likely have a boggy uterus with increased amounts of bleeding. All women experience uterine bleeding in the postpartum period, especially those who have received magnesium therapy. Rather than scant lochial flow, however, this client will most likely have a heavy flow attributable to the relaxation of the uterine wall caused by magnesium administration.

A woman with preeclampsia has a seizure. What is the nurse's highest priority during a seizure? a.To insert an oral airway b.To suction the mouth to prevent aspiration c.To administer oxygen by mask d.To stay with the client and call for help

ANS: D If a client becomes eclamptic, then the nurse should stay with the client and call for help. Nursing actions during a convulsion are directed toward ensuring a patent airway and client safety. Insertion of an oral airway during seizure activity is no longer the standard of care. The nurse should attempt to keep the airway patent by turning the client's head to the side to prevent aspiration. Once the seizure has ended, it may be necessary to suction the client's mouth. Oxygen is administered after the convulsion has ended.

Which statement related to the induction of labor is most accurate? a.Can be achieved by external and internal version techniques b.Is also known as a trial of labor (TOL) c.Is almost always performed for medical reasons d.Is rated for viability by a Bishop score

ANS: D Induction of labor is likely to be more successful with a Bishop score of 9 or higher for first-time mothers or 5 or higher for veterans. Version is the turning of the fetus to a better position by a physician for an easier or safer birth. A TOL is the observance of a woman and her fetus for several hours of active labor to assess the safety of vaginal birth. Two thirds of cases of induced labor are elective and not done for medical reasons

When would an internal version be indicated to manipulate the fetus into a vertex position? a.Fetus from a breech to a cephalic presentation before labor begins b.Fetus from a transverse lie to a longitudinal lie before a cesarean birth c.Second twin from an oblique lie to a transverse lie before labor begins d.Second twin from a transverse lie to a breech presentation during a vaginal birth

ANS: D Internal version is used only during a vaginal birth to manipulate the second twin into a presentation that allows it to be vaginally born. For internal version to occur, the cervix needs to be completely dilated.

14. Because a full bladder prevents the uterus from contracting normally, nurses intervene to help the woman empty her bladder spontaneously as soon as possible. If all else fails, the last thing the nurse might try is: a. Pouring water from a squeeze bottle over the woman's perineum b. Placing oil of peppermint in a bedpan under the woman c. Asking the physician to prescribe analgesics d. Inserting a sterile catheter

ANS: D Invasive procedures usually are the last to be tried, especially with so many other simple and easy methods available (e.g., water, peppermint vapors, pain pills). Pouring water over the perineum may stimulate voiding. It is easy, noninvasive, and should be tried early on. The oil of peppermint releases vapors that may relax the necessary muscles. It is easy, noninvasive, and should be tried early on. If the woman is anticipating pain from voiding, pain medications may be helpful. Other nonmedical means could be tried first, but medications still come before insertion of a catheter.

Nurses should be aware of the strengths and limitations of various biochemical assessments during pregnancy. Which statement regarding monitoring techniques is the most accurate? a.Chorionic villus sampling (CVS) is becoming more popular because it provides early diagnosis. b.MSAFP screening is recommended only for women at risk for NTDs. c.PUBS is one of the triple-marker tests for Down syndrome. d.MSAFP is a screening tool only; it identifies candidates for more definitive diagnostic procedures.

ANS: D MSAFP is a screening tool, not a diagnostic tool. CVS provides a rapid result, but it is declining in popularity because of advances in noninvasive screening techniques. An MSAFP screening is recommended for all pregnant women. MSAFP screening, not PUBS, is part of the triple-marker tests for Down syndrome.

10. Nursing care in the fourth trimester includes an important intervention sometimes referred to as taking the time to mother the mother. Specifically, this expression refers to: a. Formally initializing individualized care by confirming the woman's and infant's ID numbers on their respective wrist bands ("This is your baby") b. Teaching the mother to check the identity of any person who comes to remove the baby from the room ("It's a dangerous world out there") c. Including other family members in the teaching of self-care and child care ("We're all in this together") d. Nurturing the woman by providing encouragement and support as she takes on the many tasks of motherhood

ANS: D Many professionals believe that the nurse's nurturing and support function is more important than providing physical care and teaching. Matching ID wrist bands is more of a formality but also a get-acquainted procedure. Mothering the mother is more a process of encouraging and supporting the woman in her new role.

Which statement is the most complete and accurate description of medical abortions? a.Medical abortions are performed only for maternal health. b.They can be achieved through surgical procedures or with drugs. c.Medical abortions are mostly performed in the second trimester. d.They can be either elective or therapeutic.

ANS: D Medical abortions can be either elective (the womans choice) or therapeutic (for reasons of maternal or fetal health) and are performed through the use of medications rather than surgical procedures. Medical abortions are usually performed in the first trimester.

27. A nursing student is helping the nursery nurses with morning vital signs. A baby born 10 hours ago by cesarean section is found to have moist lung sounds. What is the best interpretation of these data? a. The nurse should immediately notify the pediatrician for this emergency situation. b. The neonate must have aspirated surfactant. c. If this baby was born vaginally, then a pneumothorax could be indicated. d. The lungs of a baby delivered by cesarean section may sound moist during the first 24 hours after childbirth.

ANS: D Moist lung sounds will resolve within a few hours. A surfactant acts to keep the expanded alveoli partially open between respirations for this common condition of newborns. In a vaginal birth, absorption of the remaining lung fluid is accelerated by the process of labor and delivery. The remaining lung fluid will move into interstitial spaces and be absorbed by the circulatory and lymphatic systems. Moist lung sounds are particularly common in infants delivered by cesarean section. The surfactant is produced by the lungs; therefore, aspiration is not a concern.

8. The trend in the United States is for women to remain hospitalized no longer than 1 or 2 days after giving birth for all of the following reasons except: a. A wellness orientation rather than a sick-care model b. A desire to reduce health care costs c. Consumer demand for fewer medical interventions and more family-focused experiences d. Less need for nursing time as a result of more medical and technologic advances and devices available at home that can provide information

ANS: D Nursing time and care are in demand as much as ever; the nurse just has to do things more quickly. A wellness orientation seems to focus on getting clients out the door sooner. In most cases less hospitalization results in lower costs. People believe the family gives more nurturing care than the institution.

4. A 25-year-old multiparous woman gave birth to an infant boy 1 day ago. Today her husband brings a large container of brown seaweed soup to the hospital. When the nurse enters the room, the husband asks for help with warming the soup so that his wife can eat it. The nurse's most appropriate response is to ask the woman: a. "Didn't you like your lunch?" b. "Does your doctor know that you are planning to eat that?" c. "What is that anyway?" d. "I'll warm the soup in the microwave for you."

ANS: D Offering to warm the food shows cultural sensitivity to the dietary preferences of the woman and is the most appropriate response. Cultural dietary preferences must be respected. Women may request that family members bring favorite or culturally appropriate foods to the hospital. Asking the woman to identify her food does not show cultural sensitivity.

Which classification of placental separation is not recognized as an abnormal adherence pattern? a.Placenta accreta b.Placenta increta c.Placenta percreta d.Placenta abruptio

ANS: D Placenta abruptio is premature separation of the placenta as opposed to partial or complete adherence. This classification occurs between the 20th week of gestation and delivery in the area of the decidua basalis. Symptoms include localized pain and bleeding. Placenta accreta is a recognized degree of attachment. With placenta accreta, the trophoblast slightly penetrates into the myometrium. Placenta increta is a recognized degree of attachment that results in deep penetration of the myometrium. Placenta percreta is the most severe degree of placental penetration that results in deep penetration of the myometrium. Bleeding with complete placental attachment occurs only when separation of the placenta is attempted after delivery. Treatment includes blood component therapy and, in extreme cases, hysterectomy may be necessary.

What is one of the initial signs and symptoms of puerperal infection in the postpartum client? a.Fatigue continuing for longer than 1 week b.Pain with voiding c.Profuse vaginal lochia with ambulation d.Temperature of 38° C (100.4° F) or higher on 2 successive days

ANS: D Postpartum or puerperal infection is any clinical infection of the genital canal that occurs within 28 days after miscarriage, induced abortion, or childbirth. The definition used in the United States continues to be the presence of a fever of 38° C (100.4° F) or higher on 2 successive days of the first 10 postpartum days, starting 24 hours after birth. Fatigue is a late finding associated with infection. Pain with voiding may indicate a urinary tract infection (UTI), but it is not typically one of the earlier symptoms of infection. Profuse lochia may be associated with endometritis, but it is not the first symptom associated with infection.

Which assessment is least likely to be associated with a breech presentation? a.Meconium-stained amniotic fluid b.Fetal heart tones heard at or above the maternal umbilicus c.Preterm labor and birth d.Postterm gestation

ANS: D Postterm gestation is not likely to occur with a breech presentation. The presence of meconium in a breech presentation may be a result of pressure on the fetal wall as it traverses the birth canal. Fetal heart tones heard at the level of the umbilical level of the mother are a typical finding in a breech presentation because the fetal back would be located in the upper abdominal area. Breech presentations often occur in preterm births.

The obstetric provider has informed the nurse that she will be performing an amniotomy on the client to induce labor. What is the nurse's highest priority intervention after the amniotomy is performed? a.Applying clean linens under the woman b.Taking the client's vital signs c.Performing a vaginal examination d.Assessing the fetal heart rate (FHR)

ANS: D The FHR is assessed before and immediately after the amniotomy to detect any changes that might indicate cord compression or prolapse. Providing comfort measures, such as clean linens, for the client is important but not the priority immediately after an amniotomy. The woman's temperature should be checked every 2 hours after the rupture of membranes but not the priority immediately after an amniotomy. The woman would have had a vaginal examination during the procedure. Unless cord prolapse is suspected, another vaginal examination is not warranted. Additionally, FHR assessment provides clinical cues to a prolapsed cord.

How does the nurse document a NST during which two or more FHR accelerations of 15 beats per minute or more occur with fetal movement in a 20-minute period? a.Nonreactive b.Positive c.Negative d.Reactive

ANS: D The NST is reactive (normal) when two or more FHR accelerations of at least 15 beats per minute (each with a duration of at least 15 seconds) occur in a 20-minute period. A nonreactive result means that the heart rate did not accelerate during fetal movement. A positive result is not used with NST. CST uses positive as a result term. A negative result is not used with NST. CST uses negative as a result term.

A woman is using the basal body temperature (BBT) method of contraception. She calls the clinic and tells the nurse, My period is due in a few days, and my temperature has not gone up. What is the nurses mostappropriate response? a.This probably means that youre pregnant. b.Dont worry; its probably nothing. c.Have you been sick this month? d.You probably didnt ovulate during this cycle.

ANS: D The absence of a temperature decrease most likely is the result of a lack of ovulation. Pregnancy cannot occur without ovulation, which is being measured using the BBT method. A comment such as, Dont worry; its probably nothing, discredits the clients concerns. Illness is most likely the cause of an increase in BBT. DIF: Cognitive Level: Apply REF: pp. 175-176

6. A client currently uses a diaphragm and spermicide for contraception. She asks the nurse to explain the major differences between the cervical cap and the diaphragm. What is the most appropriate response by the nurse? a.No spermicide is used with the cervical cap, so its less messy. b.The diaphragm can be left in place longer after intercourse. c.Repeated intercourse with the diaphragm is more convenient. d.The cervical cap can be safely used for repeated acts of intercourse without adding more spermicide later.

ANS: D The cervical cap can be inserted hours before sexual intercourse without the need for additional spermicide later. Spermicide should be used inside the cap as an additional chemical barrier. The cervical cap should remain in place for 6 hours after the last act of intercourse. Repeated intercourse with the cervical cap is more convenient because no additional spermicide is needed. DIF: Cognitive Level: Apply REF: p. 181 TOP: Nursing Process: Planning

24. While evaluating the reflexes of a newborn, the nurse notes that with a loud noise the newborn symmetrically abducts and extends his arms, his fingers fan out and form a C with the thumb and forefinger, and he has a slight tremor. The nurse would document this finding as a positive _____ reflex. a. tonic neck b. glabellar (Myerson) c. Babinski d. Moro

ANS: D The characteristics displayed by the infant are associated with a positive Moro reflex. The tonic neck reflex occurs when the infant extends the leg on the side to which the infants head simultaneously turns. The glabellar (Myerson) reflex is elicited by tapping on the infants head while the eyes are open. A characteristic response is blinking for the first few taps. The Babinski reflex occurs when the sole of the foot is stroked upward along the lateral aspect of the sole and then across the ball of the foot. A positive response occurs when all the toes hyperextend, with dorsiflexion of the big toe.

19. Nurses should be cognizant of what information with regard to the noncontraceptive medical effects of combination oral contraceptives (COCs)? a.COCs can cause TSS if the prescription is wrong. b.Hormonal withdrawal bleeding is usually a little more profuse than in normal menstruation and lasts a week for those who use COCs. c.COCs increase the risk of endometrial and ovarian cancers. d.Effectiveness of COCs can be altered by some over-the-counter medications and herbal supplements.

ANS: D The effectiveness of COCs can be altered by some over-the-counter medications and herbal supplements. TSS can occur in some who use the diaphragm, but it is not a consequence of taking oral contraceptive pills. Hormonal withdrawal bleeding usually is lighter than in normal menstruation and lasts a couple of days. Oral contraceptive pills offer protection against the risk of endometrial and ovarian cancers. DIF: Cognitive Level: Understand REF: p. 184 TOP: Nursing Process: Planning

8. What is the most critical physiologic change required of the newborn after birth? a. Closure of fetal shunts in the circulatory system b. Full function of the immune defense system c. Maintenance of a stable temperature d. Initiation and maintenance of respirations

ANS: D The most critical adjustment of a newborn at birth is the establishment of respirations. The cardiovascular system changes significantly after birth as a result of fetal respirations, which reduce pulmonary vascular resistance to the pulmonary blood flow and initiate a chain of cardiac changes that support the cardiovascular system. After the establishment of respirations, heat regulation is critical to newborn survival. The infant relies on passive immunity received from the mother for the first 3 months of life.

17. Which infant response to cool environmental conditions is either not effective or not available to them? a. Constriction of peripheral blood vessels b. Metabolism of brown fat c. Increased respiratory rates d. Unflexing from the normal position

ANS: D The newborns flexed position guards against heat loss, because it reduces the amount of body surface exposed to the environment. The newborns body is able to constrict the peripheral blood vessels to reduce heat loss. Burning brown fat generates heat. The respiratory rate may rise to stimulate muscular activity, which generates heat.

In evaluating the effectiveness of magnesium sulfate for the treatment of preterm labor, which finding alerts the nurse to possible side effects? a.Urine output of 160 ml in 4 hours b.DTRs 2+ and no clonus c.Respiratory rate (RR) of 16 breaths per minute d.Serum magnesium level of 10 mg/dl

ANS: D The therapeutic range for magnesium sulfate management is 4 to 7.5 mg/dl. A serum magnesium level of 10 mg/dl could lead to signs and symptoms of magnesium toxicity, including oliguria and respiratory distress. Urine output of 160 ml in 4 hours, DTRs of 2+, and a RR of 16 breaths per minute are all normal findings.

In comparing the abdominal and transvaginal methods of ultrasound examination, which information should the nurse provide to the client? a.Both require the woman to have a full bladder. b.The abdominal examination is more useful in the first trimester. c.Initially, the transvaginal examination can be painful. d.The transvaginal examination allows pelvic anatomy to be evaluated in greater detail.

ANS: D The transvaginal examination allows pelvic anatomy to be evaluated in greater detail than the abdominal method and also allows intrauterine pregnancies to be diagnosed earlier. The abdominal examination requires a full bladder; the transvaginal examination requires an empty one. The transvaginal examination is more useful in the first trimester; the abdominal examination works better after the first trimester. Neither the abdominal nor the transvaginal method of ultrasound examination should be painful, although the woman will feel pressure as the probe is moved during the transvaginal examination.

21. The brain is vulnerable to nutritional deficiencies and trauma in early infancy. What is the rationale for this physiologic adaptation in the newborn? a. Incompletely developed neuromuscular system b. Primitive reflex system c. Presence of various sleep-wake states d. Cerebellum growth spurt

ANS: D The vulnerability of the brain is likely due to the cerebellum growth spurt. By the end of the first year, the cerebellum ends its growth spurt that began at approximately 30 weeks of gestation. The neuromuscular system is almost completely developed at birth. The reflex system is not relevant to the cerebellum growth spurt. The various sleep-wake states are not relevant to the cerebellum growth spurt.

A woman arrives at the clinic seeking confirmation that she is pregnant. The following information is obtained: She is 24 years old with a body mass index (BMI) of 17.5. She admits to having used cocaine "several times" during the past year and occasionally drinks alcohol. Her blood pressure is 108/70 mm Hg. The family history is positive for diabetes mellitus and cancer. Her sister recently gave birth to an infant with a neural tube defect (NTD). Which characteristics places this client in a high-risk category? a.Blood pressure, age, BMI b.Drug and alcohol use, age, family history c.Family history, blood pressure (BP), BMI d.Family history, BMI, drug and alcohol abuse

ANS: D The woman's family history of an NTD, her low BMI, and her drug and alcohol use abuse are high risk factors of pregnancy. The woman's BP is normal, and her age does not put her at risk. Her BMI is low and may indicate poor nutritional status, which is a high risk.

A woman at 39 weeks of gestation with a history of preeclampsia is admitted to the labor and birth unit. She suddenly experiences increased contraction frequency of every 1 to 2 minutes, dark red vaginal bleeding, and a tense, painful abdomen. Which clinical change does the nurse anticipate? a.Eclamptic seizure b.Rupture of the uterus c.Placenta previa d.Abruptio placentae

ANS: D Uterine tenderness in the presence of increasing tone may be the earliest sign of abruptio placentae. Women with preeclampsia are at increased risk for an abruption attributable to decreased placental perfusion. Eclamptic seizures are evidenced by the presence of generalized tonic-clonic convulsions. Uterine rupture exhibits hypotonic uterine activity, signs of hypovolemia, and, in many cases, the absence of pain. Placenta previa exhibits bright red, painless vaginal bleeding.

Which statement best describes chronic hypertension? a.Chronic hypertension is defined as hypertension that begins during pregnancy and lasts for the duration of the pregnancy. b.Chronic hypertension is considered severe when the systolic BP is higher than 140 mm Hg or the diastolic BP is higher than 90 mm Hg. c.Chronic hypertension is general hypertension plus proteinuria. d.Chronic hypertension can occur independently of or simultaneously with preeclampsia.

ANS: D Women with chronic hypertension may develop superimposed preeclampsia, which increases the morbidity for both the mother and the fetus. Chronic hypertension is present before pregnancy or diagnosed before the 20 weeks of gestation and persists longer than 6 weeks postpartum. Chronic hypertension becomes severe with a diastolic BP of 110 mm Hg or higher. Proteinuria is an excessive concentration of protein in the urine and is a complication of hypertension, not a defining characteristic.

2. If a mother and her family have freely chosen early discharge from the hospital, the nurse and the health care provider are not legally responsible if complications occur and her condition had not been stabilized within normal limits. Is this statement true or false?

ANS: F Even if the mother chose to leave, the medical and nursing staffs still could be sued for abandonment.

TRUE/FALSE 1. Regardless of her obstetric status, no woman should be discharged from the recovery area until she has completely recovered from the effects of anesthesia and has been cleared by a member of the anesthesia care team. Is this statement true or false?

ANS: T It takes several hours to recover from anesthesia. Obstetric recovery areas are held to the same standard of care expected for any postanesthesia recovery.

A nurse is providing breast care education to a client after mammography. Which information regarding fibrocystic changes in the breast is important for the nurse to share? a. Fibrocystic breast disease is a disease of the milk ducts and glands in the breasts b. Healthy women with fibrocystic breast disease find lumpiness with pain and tenderness in varying degrees in the breast tissue during menstrual cycles. c. Lumpiness is accompanied by tenderness after menses. d. It is a premalignant disorder characterized by lumps found in the breast tissue.

B

A primigravida at 40 weeks of gestation is having uterine contractions every 1½ to 2 minutes and states that they are very painful. Her cervix is dilated 2 cm and has not changed in 3 hours. The woman is crying and wants an epidural. What is the likelystatus of this woman's labor? a. She is experiencing precipitous labor. b. She is exhibiting hypotonic uterine dysfunction. c. She is exhibiting hypertonic uterine dysfunction d. She is experiencing a normal latent stage

B

A woman who has recently given birth complains of pain and tenderness in her leg. On physical examination, the nurse notices warmth and redness over an enlarged, hardened area. Which condition should the nurse suspect, and how will it be confirmed? a. von Willebrand disease (vWD); noting whether bleeding times have been extended b. Thrombophlebitis; using real-time and color Doppler ultrasound c. Idiopathic or immune thrombocytopenic purpura (ITP); drawing blood for laboratory analysis d. Disseminated intravascular coagulation (DIC); asking for laboratory tests

B

In assisting the breastfeeding mother to position the baby, which information regarding positioning is important for the nurse to keep in mind? a. While supporting the head, the mother should push gently on the occiput. b. Whatever the position used, the infant is "belly to belly" with the mother. c. Women with perineal pain and swelling prefer the modified cradle position. d. The cradle position is usually preferred by mothers who had a cesarean birth.

B

New screening mammogram recommendations by ACOG include which of the following? a. All women begin screening mammogram at 40 years old b. Women should talk to their health care provider to decide the best age to begin screening mammograms c. All women after 55 y/o should have annual screening mammograms d. Only women with increased risk for breast begin screening mammogram at age 40

B

Sally is 34 weeks pregnant and has a biophysical profile score of 8. She asks the nurse what does that mean? The nurse's best response is: a. "Since you are more than 36 weeks, it is best to deliver your baby as she may have asphyxia. I will call your doctor." b. "Your baby has low risk for having a problem having enough oxygen in her body and your doctor will probably want to do the test again soon. I will call your doctor. " c. " Your baby may be having some problems and your doctor may want to check to see if it is safe to deliver her soon. I will call your doctor." d. "Your baby is in danger of dying and you will probably need to deliver ASAP. I will call your doctor."

B

The nurse is completing the lab order for a pap smear performed on a 22 year old female. Which of the following options will the nurse select? a. Pap smear - cytology only b. Pap smear- cytology with high risk HPV test c. Pap smear - no cytology, high risk HPV testing only d. Pap smear- cytology with reflux to high risk HPV test for ASCUS result

B

The nurse is teaching new parents about metabolic screening for the newborn. Which statement is most helpful to these clients? a. All states test for phenylketonuria (PKU), hypothyroidism, cystic fibrosis, and sickle cell diseases. b. If genetic screening is performed before the infant is 24 hours old, then it should be repeated at age 1 to 2 weeks. c. Hearing screening is now mandated by federal law. d. Federal law prohibits newborn genetic testing without parental consent.

B

What is the most critical physiologic change required of the newborn after birth? a. Full function of the immune defense system b. Initiation and maintenance of respirations c. Maintenance of a stable temperature d. Closure of fetal shunts in the circulatory system

B

Which important component of nutritional counseling should the nurse include in health teaching for a pregnant woman who is experiencing cholecystitis? a. Teach the woman that the bulk of calories should come from proteins. b. Instruct the woman to eat a low-fat diet and to avoid fried foods. c. Assess the woman's dietary history for adequate calories and proteins. d. Instruct the woman to eat a low-cholesterol, low-salt diet.

B

Which infant is most likely to express Rh incompatibility? a. Infant of an Rh-negative mother and a father who is Rh positive and heterozygous for the Rh factor b. Infant of an Rh-negative mother and a father who is Rh positive and homozygous for the Rh factor c. Infant who is Rh positive and a mother who is Rh positive d. Infant who is Rh negative and a mother who is Rh negative

B

Which nursing intervention is paramount when providing care to a client with preterm labor who has received terbutaline? a. Assess for hypoglycemia. b. Assess for dyspnea and crackles. c. Assess deep tendon reflexes (DTRs). d. Assess for bradycardia.

B

While providing care to the maternity client, the nurse should be aware that one of these anxiety disorders is likely to be triggered by the process of labor and birth. Which disorder fits this criterion? a. Phobias b. Posttraumatic stress disorder (PTSD) c. Obsessive-compulsive disorder (OCD) d. Panic disorder

B

The musculoskeletal system adapts to the changes that occur during pregnancy. A woman can expect to experience what change? A. Her center of gravity will shift backward. B. She will have increased lordosis. C. She will have increased abdominal muscle tone. D..She will notice decreased mobility of her pelvic joints.

B (An increase in the normal lumbosacral curve (lordosis) develops, and a compensatory curvature in the cervicodorsal region develops to help the woman maintain her balance. The center of gravity shifts forward. She will have decreased muscle tone. She will notice increased mobility of her pelvic joints.)

The nurse is assessing a pregnant female who has signs of ballottement and increased pulse rate. The nurse is able to visualize the fetus by radiography images, but the laboratory reports show a negative urine pregnancy report. What is the most probable age of the fetus? A. 6 weeks B. 16 weeks C. 26 weeks D. 36 weeks

B (An increase in the pulse rate is seen in between 14 and 20 weeks of gestation in a pregnant female. Ballottement is a sign of passive movements in the fetus, which is generally observed between weeks 16 and 18 of pregnancy. The fetus can be visualized by radiographic images during week 16 of pregnancy. Human chorionic gonadotropin (hCG) levels in the urine decline after 60 days of pregnancy (week 12), which results in a negative urine pregnancy test. Therefore the probable age of the fetus is 16 weeks. In week 6 of pregnancy, the fetus is not visualized by radiography. In weeks 26 and 36, signs of ballottement and increased pulse are not seen, but fetal movements are observed.)

A 31-year-old woman believes that she may be pregnant. She took an OTC pregnancy test 1 week ago after missing her period; the test was positive. During her assessment interview, the nurse inquires about the woman's last menstrual period and asks whether she is taking any medications. The woman states that she takes medicine for epilepsy. She has been under considerable stress lately at work and has not been sleeping well. She also has a history of irregular periods. Her physical examination does not indicate that she is pregnant. She has an ultrasound scan, which reveals that she is not pregnant. What is the most likely cause of the false-positive pregnancy test result? A. She took the pregnancy test too early. B. She takes anticonvulsants. C. She has a fibroid tumor. D. She has been under considerable stress and has a hormone imbalance.

B (Anticonvulsants may cause false-positive pregnancy test results. OTC pregnancy tests use enzyme-linked immunosorbent assay technology, which can yield positive results 4 days after implantation. Implantation occurs 6 to 10 days after conception. If the woman were pregnant, she would be into her third week at this point (having missed her period 1 week ago). Fibroid tumors do not produce hormones and have no bearing on hCG pregnancy tests. Although stress may interrupt normal hormone cycles (menstrual cycles), it does not affect human chorionic gonadotropin levels or produce positive pregnancy test results.)

The nurse is teaching a group of nursing students about the changes in shape, size, and position of the uterus during pregnancy. Arrange the shapes and sizes of the uterus during pregnancy in an ascending order. 1. Grapefruit-shaped uterus 2. Orange fruit-shaped uterus 3. Large hen's egg-shaped uterus 4. Upside-down pear-shaped uterus A. 3, 4, 2, 1 B. 4, 3, 2, 1 C. 1, 3, 4, 2 D. 3, 4, 1, 2

B (At conception the uterus has a small size and is in the shape of an upside-down pear. At week 7 of gestation the size of the uterus increases and the uterus takes the shape of a large hen's egg. At week 10 of gestation the uterus turns into a size of an orange. The uterus takes the shape of a grapefruit by week 12 of gestation.)

To reassure and educate pregnant clients about changes in their cardiovascular system, maternity nurses should be aware that: A. A pregnant woman experiencing disturbed cardiac rhythm, such as sinus arrhythmia requires close medical and obstetric observation, no matter how healthy she otherwise may appear. B. Changes in heart size and position and increases in blood volume create auditory changes from 20 weeks to term. C. Palpitations are twice as likely to occur in twin gestations. D. All of the above changes likely will occur.

B (Auscultatory changes should be discernible after 20 weeks of gestation. A healthy woman with no underlying heart disease does not need any therapy. The maternal heart rate increases in the third trimester, but palpitations may not occur. Auditory changes are discernible at 20 weeks.)

Cardiovascular system changes occur during pregnancy. Which finding would be considered normal for a woman in her second trimester? A. Less audible heart sounds (S1, S2) B. Increased pulse rate C. Increased blood pressure D. Decreased red blood cell (RBC) production

B (Between 14 and 20 weeks of gestation, the pulse increases about 10 to 15 beats/min, which persists to term. Splitting of S1 and S2 is more audible. In the first trimester, blood pressure usually remains the same as at the prepregnancy level, but it gradually decreases up to about 20 weeks of gestation. During the second trimester, both the systolic and the diastolic pressures decrease by about 5 to 10 mm Hg. Production of RBCs accelerates during pregnancy.)

Cardiovascular system changes occur during pregnancy. Which finding is considered normal for a woman in her second trimester? A. Less audible heart sounds (S1, S2) B. Increased pulse rate C. Increased blood pressure D. Decreased red blood cell (RBC) production

B (Between 14 and 20 weeks of gestation, the pulse increases about 10 to 15 beats/min, which persists to term. Splitting of S1 and S2 is more audible. In the first trimester, blood pressure usually remains the same as the prepregnancy level, but it gradually decreases up to about 20 weeks of gestation. During the second trimester, both the systolic and diastolic pressures decrease by about 5 to 10 mm Hg. Production of RBCs accelerates during pregnancy.)

To reassure and educate pregnant clients about the functioning of their kidneys in eliminating waste products, maternity nurses should be aware that: A. Increased urinary output makes pregnant women less susceptible to urinary infection. B. Increased bladder sensitivity and then compression of the bladder by the enlarging uterus results in the urge to urinate even if the bladder is almost empty. C. Renal (kidney) function is more efficient when the woman assumes a supine position. D. Using diuretics during pregnancy can help keep kidney function regular.

B (First bladder sensitivity and then compression of the bladder by the uterus result in the urge to urinate more often. Numerous anatomic changes make a pregnant woman more susceptible to urinary tract infection. Renal function is more efficient when the woman lies in the lateral recumbent position and less efficient when she is supine. Diuretic use during pregnancy can overstress the system and cause problems.)

A woman is at 14 weeks of gestation. The nurse would expect to palpate the fundus at which level? A. Not palpable above the symphysis at this time B. Slightly above the symphysis pubis C. At the level of the umbilicus D. Slightly above the umbilicus

B (In normal pregnancies, the uterus grows at a predictable rate. It may be palpated above the symphysis pubis sometime between the twelfth and fourteenth weeks of pregnancy. As the uterus grows, it may be palpated above the symphysis pubis sometime between the twelfth and fourteenth weeks of pregnancy. The uterus rises gradually to the level of the umbilicus at 22 to 24 weeks of gestation.)

A patient in her first trimester complains of nausea and vomiting. She asks, "Why does this happen?" The nurse's best response is: A. "It is due to an increase in gastric motility." B. "It may be due to changes in hormones." C. "It is related to an increase in glucose levels." D. "It is caused by a decrease in gastric secretions."

B (Nausea and vomiting are believed to be caused by increased levels of hormones, decreased gastric motility, and hypoglycemia. Gastric motility decreases during pregnancy. Glucose levels decrease in the first trimester. Although gastric secretions decrease, this is not the main cause of nausea and vomiting.)

The nurse teaches a pregnant woman about the presumptive, probable, and positive signs of pregnancy. The woman demonstrates understanding of the nurse's instructions if she states that a positive sign of pregnancy is: A. A positive pregnancy test. B. Fetal movement palpated by the nurse-midwife. C. Braxton Hicks contractions. D. Quickening.

B (Positive signs of pregnancy are attributed to the presence of a fetus, such as hearing the fetal heartbeat or palpating fetal movement. A positive pregnancy test and Braxton Hicks contractions are probable signs of pregnancy. Quickening is a presumptive sign of pregnancy.)

The nurse observes that a patient has severe itching during pregnancy. Which function in the patient is affected? A. Renal function B. Hepatic function C. Respiratory function D. Gastrointestinal function

B (Severe itching in the pregnant woman is a condition called pruritus gravidarum. It is the result of intrahepatic cholestasis (accumulation of bile in the liver) caused by placental steroids. Hence, the hepatic function was affected in the patient. Itching is not indicative of renal, pulmonary, or gastrointestinal function. Glucosuria and proteinuria indicate that the renal function is affected. Nasal stuffiness, sinus stuffiness, and epistaxis indicate that the upper respiratory function is impaired. Pyrosis or heartburn indicates that the gastrointestinal function is impaired.)

Which finding in the urine analysis of a pregnant woman is considered a variation of normal? A. Proteinuria B. Glycosuria C. Bacteria in the urine. D. Ketones in the urine.

B (Small amounts of glucose may indicate "physiologic spilling." The presence of protein could indicate kidney disease or preeclampsia. Urinary tract infections are associated with bacteria in the urine. An increase in ketones indicates that the patient is exercising too strenuously or has an inadequate fluid and food intake)

Which time-based description of a stage of development in pregnancy is accurate? A. Viability—22 to 37 weeks since the last menstrual period (LMP) (assuming a fetal weight >500 g) B. Term—pregnancy from the beginning of week 38 of gestation to the end of week 42 C. Preterm—pregnancy from 20 to 28 weeks D. Postdate—pregnancy that extends beyond 38 weeks

B (Term is 38 to 42 weeks of gestation. Viability is the ability of the fetus to live outside the uterus before coming to term, or 22 to 24 weeks since LMP. Preterm is 20 to 37 weeks of gestation. Postdate or postterm is a pregnancy that extends beyond 42 weeks or what is considered the limit of full term.)

A woman's obstetric history indicates that she is pregnant for the fourth time and all of her children from previous pregnancies are living. One was born at 39 weeks of gestation, twins were born at 34 weeks of gestation, and another child was born at 35 weeks of gestation. What is her gravidity and parity using the GTPAL system? A. 3-1-1-1-3 B. 4-1-2-0-4 C. 3-0-3-0-3 D. 4-2-1-0-3

B (The correct calculation of this woman's gravidity and parity is 4-1-2-0-4. The numbers reflect the woman's gravidity and parity information. Using the GPTAL system, her information is calculated as: G: The first number reflects the total number of times the woman has been pregnant; she is pregnant for the fourth time. T: This number indicates the number of pregnancies carried to term, not the number of deliveries at term; only one of her pregnancies has resulted in a fetus at term. P: This is the number of pregnancies that resulted in a preterm birth; the woman has had two pregnancies in which she delivered preterm. A: This number signifies whether the woman has had any abortions or miscarriages before the period of viability; she has not. L: This number signifies the number of children born that currently are living; the woman has four children.)

The nurse reviews the medical records of a patient and suggests the patient avoid becoming pregnant. Why does the nurse suggest so? The patient: A. Has excess proteins in the urine. B. Is using isotretinoin (Accutane). C. Has increased blood sugar levels. D. Is taking promethazine (Phenergan).

B (The nurse suggests the patient avoid pregnancy because the patient is using isotretinoin (Accutane) for the treatment of acne. This medication is teratogenic and is associated with fetal malformations. Proteinuria and increased blood glucose levels are the common conditions during pregnancy, although they disappear after childbirth. Promethazine (Phenergan) therapy can cause only false-negative results for pregnancy tests. This drug is not a contraindication for pregnancy.)

The maternity nurse understands that vascular volume increases 40% to 60% during pregnancy to: A. Compensate for decreased renal plasma flow. B. Provide adequate perfusion of the placenta. C Eliminate metabolic wastes of the mother. D. Prevent maternal and fetal dehydration.

B (The primary function of increased vascular volume is to transport oxygen and nutrients to the fetus via the placenta. Renal plasma flow increases during pregnancy. Assisting with pulling metabolic wastes from the fetus for maternal excretion is one purpose of the increased vascular volume.)

To reassure and educate pregnant clients about changes in the uterus, nurses should be aware that: A. Lightening occurs near the end of the second trimester as the uterus rises into a different position. B. The woman's increased urinary frequency in the first trimester is the result of exaggerated uterine antireflexion caused by softening. C. Braxton Hicks contractions become more painful in the third trimester, particularly if the woman tries to exercise. D. The uterine souffle is the movement of the fetus.

B (The softening of the lower uterine segment is called Hegar's sign. Lightening occurs in the last 2 weeks of pregnancy, when the fetus descends. Braxton Hicks contractions become more defined in the final trimester but are not painful. Walking or exercise usually causes them to stop. The uterine souffle is the sound made by blood in the uterine arteries; it can be heard with a fetal stethoscope.)

The nurse is assessing a 3-month pregnant patient who is given folic acid supplement. The patient is worried because of the appearance of reddish spider-like rashes on the face and neck. What does the nurse tell the patient about these rashes? A. "This is a side effect of folic acid." B. "This disappears after pregnancy." C. "This is caused by a food allergy." D. "This is caused by decreased estrogen."

B (Vascular spider-like rashes are tiny, star-shaped or branched, slightly raised, and pulsating end-arterioles usually found on the neck, thorax, face, and arms during pregnancy. These spider-like rashes usually disappear after pregnancy. The appearance of vascular spider-like rashes is common during the 2 to 5 months of pregnancy and is not a result of a food allergy. Folic acid supplementation is given in pregnancy to reduce birth defects. Folic acid does not cause vascular or skin changes. Vascular spider-like rashes are not caused by elevated estrogen levels.)

Physiologic anemia often occurs during pregnancy as a result of: A. Inadequate intake of iron. B. Dilution of hemoglobin concentration. C. The fetus establishing iron stores. D. Decreased production of erythrocytes.

B (When blood volume expansion is more pronounced and occurs earlier than the increase in red blood cells, the woman has physiologic anemia, which is the result of dilution of hemoglobin concentration rather than inadequate hemoglobin. Inadequate intake of iron may lead to true anemia. There is an increased production of erythrocytes during pregnancy.)

The nurse providing care for the laboring woman should understand that variable FHR decelerations are caused by: A. Altered fetal cerebral blood flow B. Umbilical cord compression C. Uteroplacental insufficiency D. Fetal hypoxemia

B A. Incorrect: Altered fetal cerebral blood flow would result in early decelerations in the FHR. B. Correct: Variable decelerations can occur any time during the uterine contracting phase and are caused by compression of the umbilical cord. C. Incorrect: Uteroplacental insufficiency would result in late decelerations in the FHR. D. Incorrect: Fetal hypoxemia would result in tachycardia initially, then bradycardia if hypoxia continues. p. 507

During labor a fetus with an average heart rate of 135 beats/min over a 10-minute period would be considered to have: A. Bradycardia B. A normal baseline heart rate C. Tachycardia D. Hypoxia

B A. Incorrect: Bradycardia is an FHR below 110 beats/min for 10 minutes or longer. B. Correct: The baseline heart rate is measured over 10 minutes; a normal range is 110 to 160 beats/min. C. Incorrect: Tachycardia is an FHR over 160 beats/min for 10 minutes or longer. D. Incorrect: Hypoxia is an inadequate supply of oxygen; no indication of this condition exists with a baseline heart rate in the normal range. p. 502

What correctly matches the type of deceleration with its likely cause? A. Early deceleration—umbilical cord compression B. Late deceleration—uteroplacental inefficiency C. Variable deceleration—head compression D. Prolonged deceleration—cause unknown

B A. Incorrect: Early deceleration is caused by head compression. B. Correct: Late deceleration is caused by uteroplacental inefficiency. C. Incorrect: Variable deceleration is caused by umbilical cord compression. D. Incorrect: Prolonged deceleration has a variety of either benign or critical causes. p. 507

What three measures should the nurse implement to provide intrauterine resuscitation? Select the best response that indicates the priority of actions that should be taken, starting with the most important. A. Call the provider, reposition the mother, and perform a vaginal exam B. Reposition the mother, increase IV fluid, and provide oxygen via face mask C. Administer oxygen to the mother, increase IV fluid, and notify the care provider D. Perform a vaginal exam, reposition the mother, and provide oxygen via face mask

B A. Incorrect: The nurse should initiate intrauterine resuscitation in an ABC manner, similar to basic life support. The first priority is to open the maternal and fetal vascular systems by repositioning the mother for improved perfusion. The second priority is to increase blood volume by increasing the IV fluid. The third priority is to optimize oxygenation of the circulatory volume by providing oxygen via face mask. If these interventions do not resolve the fetal heart rate issue quickly, the primary provider should be notified immediately. B. Correct: These are the correct nursing actions for intrauterine resuscitation. C. Incorrect: The nurse should initiate intrauterine resuscitation in an ABC manner, similar to basic life support. The first priority is to open the maternal and fetal vascular systems by repositioning the mother for improved perfusion. The second priority is to increase blood volume by increasing the IV fluid. The third priority is to optimize oxygenation of the circulatory volume by providing oxygen via face mask. If these interventions do not resolve the fetal heart rate issue quickly, the primary provider should be notified immediately. D. Incorrect: The nurse should initiate intrauterine resuscitation in an ABC manner, similar to basic life support. The first priority is to open the maternal and fetal vascular systems by repositioning the mother for improved perfusion. The second priority is to increase blood volume by increasing the IV fluid. The third priority is to optimize oxygenation of the circulatory volume by providing oxygen via face mask. If these interventions do not resolve the fetal heart rate issue quickly, the primary provider should be notified immediately. p. 513

A new client and her partner arrive on the labor, delivery, recovery, and postpartum (LDRP) unit for the birth of their first child. You apply the EFM to the woman. Her partner asks you to explain what is printing on the graph, referring to the EFM strip. He wants to know what the baby's heart rate should be. Your best response is: A. "Don't worry about that machine; that's my job." B. "The top line graphs the baby's heart rate. Generally, the heart rate is between 110 and 160. The heart rate will fluctuate in response to what is happening during labor." C. "The top line graphs the baby's heart rate, and the bottom line lets me know how strong the contractions are." D. "Your doctor will explain all of that later."

B A. Incorrect: This discredits the partner's feelings and does not provide the teaching he is requesting. B. Correct: This statement educates the partner about fetal monitoring and provides support and information to alleviate his fears. C. Incorrect: This statement provides inaccurate information and does not address the partner's concerns about the fetal heart rate. The EFM graphs the frequency and duration of the contractions, not the intensity. D. Incorrect: Nurses should take every opportunity to provide client and family teaching, especially when information is requested. pp. 501-502

The perinatal nurse observes the new mother watching her baby daughter closely, touching her face, and asking many questions about infant feeding. This stage of mothering is best described as: Select one: a. Taking in b. Taking hold c. Taking charge d. Taking time

B As the mother's physical condition improves, she begins to take charge and enters the taking-hold phase where she assumes care for herself and her infant. At this time, the mother eagerly wants information about infant care and shows signs of bonding with her infant. During this phase, the nurse should closely observe mother-infant interactions for signs of poor bonding, and if present, implement actions to facilitate attachment. see p 508 and Table 22-4

Pelvic floor exercises, also known as Kegel exercises, will help to strengthen the perineal muscles and encourage healing after childbirth. The nurse requests the client to repeat back instructions for this exercise. Which response by the client indicates successful learning? a. "I contract my thighs, buttocks, and abdomen." b. "I pretend that I am trying to stop the flow of urine in midstream." c. "I stand while practicing this new exercise routine." d. "I perform 10 of these exercises every day."

B Kegel exercises is a technique used to strengthen the muscles that support the pelvic floor. have the woman pretend she is trying to prevent the passage of gas, pretend she is trying to stop the flow of urine or think about how the vagina can contract around the penis during intercourse. Avoid straining or bearing down motions.

A woman with worsening preeclampsia is admitted to the hospital's labor and birth unit. The physician explains the plan of care for severe preeclampsia, including the induction of labor, to the woman and her husband. Which statement by the husband leads the nurse to believe that the couple needs further information? a. "I will give my wife ice chips to eat during labor." b. "Since we will be here for a while, I will call my mother so she can bring the two boys—2 years and 4 years of age—to visit their mother." c. "I will stay with my wife during her labor, just as we planned." d. "I will help my wife use the breathing techniques that we learned in our childbirth classes."

B Need to maintain quiet, low stimulant environment.

With regard to skeletal injuries sustained by a neonate during labor or birth, nurses should be aware that: A. A newborn's skull is still forming and fractures fairly easily. B. Unless a blood vessel is involved, linear skull fractures heal without special treatment. C. Clavicle fractures often need to be set with an inserted pin for stability. D. Other than the skull, the most common skeletal injuries are to leg bones.

B A. Incorrect: Because the newborn skull is flexible, considerable force is required to fracture it. B. Correct: About 70% of neonatal skull fractures are linear. C. Incorrect: Clavicle fractures need no special treatment. D. Incorrect: The clavicle is the bone most often fractured during birth. p. 993

With regard to the classification of neonatal bacterial infection, nurses should be aware that: A. Congenital infection progresses slower than nosocomial infection. B. Nosocomial infection can be prevented by effective handwashing; early onset cannot. C. Infections occur with about the same frequency in boy and girl infants, although female mortality is higher. D. The clinical sign of a rapid, high fever makes infection easier to diagnose.

B A. Incorrect: Congenital (early onset) infections progress more rapidly than nosocomial (late onset) infections. B. Correct: Handwashing is an effective preventative measure for late onset (nosocomial) infections, because these infections come from the environment around the infant. Early onset, or congenital, infections are caused by the normal flora at the maternal vaginal tract. Congenital (early onset) infections progress more rapidly than nosocomial (late onset) infections. C. Incorrect: Infection occurs about twice as often in boys and results in higher mortality. Congenital (early onset) infections progress more rapidly than nosocomial (late onset) infections. D. Incorrect: Clinical signs of neonatal infection are nonspecific and similar to noninfectious problems, making diagnosis difficult. Congenital (early onset) infections progress more rapidly than nosocomial (late onset) infections. p. 1002

An infant diagnosed with erythroblastosis fetalis would characteristically exhibit: A. Edema B. Immature red blood cells C. Enlargement of the heart D. Ascites

B A. Incorrect: Edema would occur with hydrops fetalis, a more severe form of erythroblastosis fetalis. B. Correct: Erythroblastosis fetalis occurs when the fetus compensates for the anemia associated with Rh incompatibility by producing large numbers of immature erythrocytes to replace those hemolyzed. C. Incorrect: The fetus with hydrops fetalis may exhibit effusions into the peritoneal, pericardial, and pleural spaces. D. Incorrect: The infant with hydrops fetalis displays signs of ascites. p. 1026

When planning care for an infant with a fractured clavicle, the nurse should recognize that in addition to gentle handling: A. Prone positioning will facilitate bone alignment. B. No special treatment is necessary. C. Parents should be taught range of motion exercises. D. The shoulder should be immobilized with a splint.

B A. Incorrect: Fractures in newborns generally heal rapidly. Except for gentle handling, no accepted treatment for a fractured clavicle exists. B. Correct: Fractures in newborns generally heal rapidly. Except for gentle handling, no accepted treatment for a fractured clavicle exists. C. Incorrect: Movement should be limited, and the infant should be gently handled. It is not necessary to perform range of motion exercises on the infant. D. Incorrect: A fractured clavicle does not require immobilization with a splint. p. 994

The most common cause of pathologic hyperbilirubinemia is: A. Hepatic disease B. Hemolytic disorders in the newborn C. Postmaturity D. Congenital heart defect

B A. Incorrect: Hepatic damage may be a cause of pathologic hyperbilirubinemia, but it is not the most common cause. B. Correct: Hemolytic disorders in the newborn are the most common cause of pathologic jaundice. C. Incorrect: Prematurity would be a potential cause of pathologic hyperbilirubinemia in neonates, but it is not the most common cause. D. Incorrect: Congenital heart defect is not a common cause of pathologic hyperbilirubinemia in neonates. p. 1025

With regard to the understanding and treatment of infants born to mothers who are substance abusers, nurses should be aware that: A. Infants born to addicted mothers are also addicted. B. Mothers who abuse one substance likely will use or abuse another, compounding the infant's difficulties. C. The NICU Network Neurobehavioral Scale (NNNS) is designed to assess the damage the mother has done to herself. D. No laboratory procedures are available that can identify the intrauterine drug exposure of the infant.

B A. Incorrect: Infants of substance-abusing mothers may have some of the physiologic signs but are not addicted in the behavioral sense. "Drug-exposed newborn" is a more accurate description than "addict." B. Correct: Multiple substance use (even just alcohol and tobacco) makes it difficult to assess the problems of the exposed infant, particularly with regard to withdrawal manifestations. C. Incorrect: The NNNS is designed to assess the neurologic, behavioral, and stress/abstinence function of the neonate. D. Incorrect: Newborn urine, hair, or meconium sampling may be used to identify an infant's intrauterine drug exposure. p. 1017

27. Which intervention by the nurse would reduce the risk of abduction of the newborn from the hospital? a. Instructing the mother not to give her infant to anyone except the one nurse assigned to her that day b. Applying an electronic and identification bracelet to the mother and the infant c. Carrying the infant when transporting him or her in the halls d. Restricting the amount of time infants are out of the nursery

B (A measure taken by many facilities is to band both the mother and the baby with matching identification bracelets and band the infant with an electronic device that will sound an alarm if the infant is removed from the maternity unit. It is impossible for one nurse to be on call for one mother and baby for the entire shift; therefore, parents need to be able to identify the nurses who are working on the unit. Infants should always be transported in their bassinette for both safety and security reasons. All maternity unit nursing staff should have unique identification bracelets in comparison with the rest of the hospital. Infants should remain with their parents and spend as little time in the nursery as possible.)

12. At 1 minute after birth a nurse assesses an infant and notes a heart rate of 80 beats per minute, some flexion of extremities, a weak cry, grimacing, and a pink body but blue extremities. Which Apgar score does the nurse calculate based upon these observations and signs? a. 4 b. 5 c. 6 d. 7

B (Each of the five signs the nurse notes scores a 1 on the Apgar scale, for a total of 5. A score of 4 is too low for this infant. A score of 6 is too high for this infant. A score of 7 is too high for an infant with this presentation.)

23. What is the nurses initial action while caring for an infant with a slightly decreased temperature? a. Immediately notify the physician. b. Place a cap on the infants head, and have the mother perform kangaroo care. c. Tell the mother that the infant must be kept in the nursery and observed for the next 4 hours. d. Change the formula; a decreased body temperature is a sign of formula intolerance.

B (Keeping the head well covered with a cap prevents further heat loss from the head, and placing the infant skin-to-skin against the mother should increase the infants temperature. Nursing actions are needed first to correct the problem. If the problem persists after the interventions, physician notification may then be necessary. A slightly decreased temperature can be treated in the mothers room, offering an excellent time for parent teaching on the prevention of cold stress. Mild temperature instability is an expected deviation from normal during the first days after childbirth as the infant adapts to external life.)

2. A new father wants to know what medication was put into his infants eyes and why it is needed. How does the nurse explain the purpose of the erythromycin (Ilotycin) ophthalmic ointment? a. Erythromycin (Ilotycin) ophthalmic ointment destroys an infectious exudate caused by Staphylococcus that could make the infant blind. b. This ophthalmic ointment prevents gonorrheal and chlamydial infection of the infants eyes, potentially acquired from the birth canal. c. Erythromycin (Ilotycin) prevents potentially harmful exudate from invading the tear ducts of the infants eyes, leading to dry eyes. d. This ointment prevents the infants eyelids from sticking together and helps the infant see.

B (The nurse should explain that prophylactic erythromycin ophthalmic ointment is instilled in the eyes of all neonates to prevent gonorrheal and chlamydial infection that potentially could have been acquired from the birth canal. This prophylactic ophthalmic ointment is not instilled to prevent dry eyes and has no bearing on vision other than to protect against infection that may lead to vision problems.)

A new client and her partner arrive on the labor, delivery, recovery, and postpartum unit for the birth of their first child. You apply the electronic fetal monitor (EFM) to the woman. Her partner asks you to explain what is printing on the graph, referring to the EFM strip. He wants to know what the baby's heart rate should be. Your best response is: a. "Don't worry about that machine; that's my job." b. "The top line graphs the baby's heart rate. Generally the heart rate is between 110 and 160. The heart rate will fluctuate in response to what is happening during labor." c. "The top line graphs the baby's heart rate, and the bottom line lets me know how strong the contractions are." d. "Your doctor will explain all of that later."

B "The top line graphs the baby's heart rate. Generally the heart rate is between 110 and 160. The heart rate will fluctuate in response to what is happening during labor" educates the partner about fetal monitoring and provides support and information to alleviate his fears. "Don't worry about that machine; that's my job" discredits the partner's feelings and does not provide the teaching he is requesting. "The top line graphs the baby's heart rate, and the bottom line lets me know how strong the contractions are" provides inaccurate information and does not address the partner's concerns about the fetal heart rate. The EFM graphs the frequency and duration of the contractions, not the intensity. Nurses should take every opportunity to provide client and family teaching, especially when information is requested.

What correctly matches the type of deceleration with its likely cause? a. Early deceleration—umbilical cord compression b. Late deceleration—uteroplacental inefficiency c. Variable deceleration—head compression d. Prolonged deceleration—cause unknown

B Late deceleration is caused by uteroplacental inefficiency. Early deceleration is caused by head compression. Variable deceleration is caused by umbilical cord compression. Prolonged deceleration has a variety of either benign or critical causes.

What three measures should the nurse implement to provide intrauterine resuscitation? Select the response that best indicates the priority of actions that should be taken. a. Call the provider, reposition the mother, and perform a vaginal examination. b. Reposition the mother, increase intravenous (IV) fluid, and provide oxygen via face mask. c. Administer oxygen to the mother, increase IV fluid, and notify the care provider. d. Perform a vaginal examination, reposition the mother, and provide oxygen via face mask.

B Repositioning the mother, increasing intravenous (IV) fluid, and providing oxygen via face mask are correct nursing actions for intrauterine resuscitation. The nurse should initiate intrauterine resuscitation in an ABC manner, similar to basic life support. The first priority is to open the maternal and fetal vascular systems by repositioning the mother for improved perfusion. The second priority is to increase blood volume by increasing the IV fluid. The third priority is to optimize oxygenation of the circulatory volume by providing oxygen via face mask. If these interventions do not resolve the fetal heart rate issue quickly, the primary provider should be notified immediately.

During labor a fetus with an average heart rate of 135 beats/min over a 10-minute period would be considered to have: a. Bradycardia. c. Tachycardia. b. A normal baseline heart rate. d. Hypoxia.

B The baseline heart rate is measured over 10 minutes; a normal range is 110 to 160 beats/min. Bradycardia is a fetal heart rate (FHR) below 110 beats/min for 10 minutes or longer. Tachycardia is an FHR over 160 beats/min for 10 minutes or longer. Hypoxia is an inadequate supply of oxygen; no indication of this condition exists with a baseline heart rate in the normal range.

Why is continuous electronic fetal monitoring usually used when oxytocin is administered? a. The mother may become hypotensive. b. Uteroplacental exchange may be compromised. c. Maternal fluid volume deficit may occur. d. Fetal chemoreceptors are stimulated.

B The uterus may contract more firmly, and the resting tone may be increased with oxytocin use. This response reduces entrance of freshly oxygenated maternal blood into the intervillous spaces, thus depleting fetal oxygen reserves. Hypotension is not a common side effect of oxytocin. All laboring women are at risk for fluid volume deficit; oxytocin administration does not increase the risk. Oxytocin affects the uterine muscles.

The nurse providing care for the laboring woman realizes that variable fetal heart rate (FHR) decelerations are caused by: a. Altered fetal cerebral blood flow. c. Uteroplacental insufficiency. b. Umbilical cord compression. d. Fetal hypoxemia.

B Variable decelerations can occur any time during the uterine contracting phase and are caused by compression of the umbilical cord. Altered fetal cerebral blood flow would result in early decelerations in the FHR. Uteroplacental insufficiency would result in late decelerations in the FHR. Fetal hypoxemia would result in tachycardia initially and then bradycardia if hypoxia continues.

14. The nurse is completing a physical examination of the newborn 24 hours after birth. Which component of the evaluation is correct? a. The parents are excused to reduce their normal anxiety. b. The nurse can gauge the neonates maturity level by assessing his or her general appearance. c. Once often neglected, blood pressure is now routinely checked. d. When the nurse listens to the neonates heart, the S1 and S2 sounds can be heard; the S1sound is somewhat higher in pitch and sharper than the S2 sound.

B (The nurse is looking at skin color, alertness, cry, head size, and other features. The parents presence actively involves them in child care and gives the nurse the chance to observe their interactions. Blood pressure is not usually taken unless cardiac problems are suspected. The S2 sound is higher and sharper than the S1 sound.)

3. Which client is most likely to experience strong and uncomfortable afterpains? a. A woman who experienced oligohydramnios b. A woman who is a gravida 4, para 4-0-0-4 c. A woman who is bottle-feeding her infant d. A woman whose infant weighed 5 pounds, 3 ounces

B (Afterpains are more common in multiparous women. In a woman who experienced polyhydramnios, afterpains are more noticeable because the uterus was greatly distended. Breastfeeding may cause the afterpains to intensify. In a woman who delivered a large infant, afterpains are more noticeable because the uterus was greatly distended.)

19. The nurse is explaining the benefits associated with breastfeeding to a new mother. Which statement by the nurse would provide conflicting information to the client? a.Women who breastfeed have a decreased risk of breast cancer. b.Breastfeeding is an effective method of birth control. c.Breastfeeding increases bone density. d.Breastfeeding may enhance postpartum weight loss.

B (Although breastfeeding delays the return of fertility, it is not an effective birth control method. Women who breastfeed have a decreased risk of breast cancer, an increase in bone density, and a possibility of faster postpartum weight loss.)

28. Which instruction should the nurse provide to reduce the risk of nipple trauma? a.Limit the feeding time to less than 5 minutes. b.Position the infant so the nipple is far back in the mouth. c.Assess the nipples before each feeding. d.Wash the nipples daily with mild soap and water.

B (If the infants mouth does not cover as much of the areola as possible, the pressure during sucking will be applied to the nipple, thus causing trauma to the area. Stimulating the breast for less than 5 minutes will not produce the extra milk the infant may need and will also limit access to the higher-fat hindmilk. Assessing the nipples for trauma is important; however, this action alone will not prevent sore nipples. Soap can be drying to the nipples and should be avoided during breastfeeding.)

23. The breastfeeding mother should be taught a safe method to remove the breast from the babys mouth. Which suggestion by the nurse is most appropriate? a.Slowly remove the breast from the babys mouth when the infant has fallen asleep and the jaws are relaxed. b.Break the suction by inserting your finger into the corner of the infants mouth. c.A popping sound occurs when the breast is correctly removed from the infants mouth. d.Elicit the Moro reflex to wake the baby and remove the breast when the baby cries.

B (Inserting a finger into the corner of the babys mouth between the gums to break the suction avoids trauma to the breast. The infant who is sleeping may lose grasp on the nipple and areola, resulting in chewing on the nipple that makes it sore. A popping sound indicates improper removal of the breast from the babys mouth and may cause cracks or fissures in the breast. Most mothers prefer the infant to continue to sleep after the feeding. Gentle wake-up techniques are recommended.)

17. Which statement by a newly delivered woman indicates that she knows what to expect regarding her menstrual activity after childbirth? a. My first menstrual cycle will be lighter than normal and then will get heavier every month thereafter. b. My first menstrual cycle will be heavier than normal and will return to my prepregnant volume within three or four cycles. c. I will not have a menstrual cycle for 6 months after childbirth. d. My first menstrual cycle will be heavier than normal and then will be light for several months after.

B (My first menstrual cycle will be heavier than normal and will return to my prepregnant volume within three or four cycles is an accurate statement and indicates her understanding of her expected menstrual activity. She can expect her first menstrual cycle to be heavier than normal, which occurs by 3 months after childbirth, and the volume of her subsequent cycles will return to prepregnant levels within three to four cycles.)

13. The nurse should be cognizant of which postpartum physiologic alteration? a. Cardiac output, pulse rate, and stroke volume all return to prepregnancy normal values within a few hours of childbirth. b. Respiratory function returns to nonpregnant levels by 6 to 8 weeks after childbirth. c. Lowered white blood cell count after pregnancy can lead to false-positive results on tests for infections. d. Hypercoagulable state protects the new mother from thromboembolism, especially after a cesarean birth.

B (Respirations should decrease to within the womans normal prepregnancy range by 6 to 8 weeks after childbirth. Stroke volume increases and cardiac output remains high for a couple of days. However, the heart rate and blood pressure quickly return to normal. Leukocytosis increases 10 to 12 days after childbirth, which can obscure the diagnosis of acute infections, producing false-negative test results. The hypercoagulable state increases the risk of thromboembolism, especially after a cesarean birth.)

1. A new mother recalls from prenatal class that she should try to feed her newborn daughter when she exhibits feeding readiness cues rather than waiting until the baby is frantically crying. Which feeding cue would indicate that the baby is ready to eat? a.Waves her arms in the air b.Makes sucking motions c.Has the hiccups d.Stretches out her legs straight

B (Sucking motions, rooting, mouthing, and hand-to-mouth motions are examples of feeding readiness cues. Waving her arms in the air, having the hiccups, and stretching out her extremities are not typical feeding readiness cues.)

17. A new father is ready to take his wife and newborn son home. He proudly tells the nurse who is discharging them that within the next week he plans to start feeding the infant cereal between breastfeeding sessions. Which information should the nurse provide regarding this feeding plan? a.Feeding solid foods before your son is 4 to 6 months old may decrease your sons intake of sufficient calories. b.Feeding solid foods between breastfeeding sessions before your son is 4 to 6 months old will lead to an early cessation of breastfeeding. c.Your feeding plan will help your son sleep through the night. d.Feeding solid foods before your son is 4 to 6 months old will limit his growth.

B (The introduction of solid foods before the infant is 4 to 6 months of age may result in overfeeding and decreased intake of breast milk. The belief that feeding solid foods helps infants sleep through the night is untrue. The proper balance of carbohydrate, protein, and fat for an infant to grow properly is in the breast milk or formula.)

11. Which statement regarding the postpartum uterus is correct? a. At the end of the third stage of labor, the postpartum uterus weighs approximately 500 g. b. After 2 weeks postpartum, it should be abdominally nonpalpable. c. After 2 weeks postpartum, it weighs 100 g. d. Postpartum uterus returns to its original (prepregnancy) size by 6 weeks postpartum.

B (The uterus does not return to its original size. At the end of the third stage of labor, the uterus weighs approximately 1000 g. After 2 weeks postpartum, the uterus weighs approximately 350 g. The normal self-destruction of excess hypertrophied tissue accounts for the slight increase in uterine size after each pregnancy.)

19. Which description of postpartum restoration or healing times is accurate? a. The cervix shortens, becomes firm, and returns to form within a month postpartum. b. Vaginal rugae reappear by 3 weeks postpartum. c. Most episiotomies heal within a week. d. Hemorrhoids usually decrease in size within 2 weeks of childbirth.

B (Vaginal rugae reappear by 3 weeks postpartum; however, they are never as prominent as in nulliparous women. The cervix regains its form within days; the cervical os may take longer. Most episiotomies take 2 to 3 weeks to heal. Hemorrhoids can take 6 weeks to decrease in size.)

For which of the following conditions is Daily Fetal Movement Count indicated? (Select all that apply). a. Pregnant woman who is older than 35 years with no complications b. Mother with low amniotic fluid volume or oligohydraminos c. Mother with diabetes during pregnancy d. Mother who complains of headaches during pregnancy that are relieved with Tylenol

B, C

1. The breast-feeding mother should be taught to expect which changes to the condition of the breasts? (Select all that apply.) a. Breast tenderness is likely to persist for approximately 1 week after the start of lactation. b. As lactation is established, a mass may form that can be distinguished from cancer by its positional shift from day to day. c. In nonlactating mothers, colostrum is present for the first few days after childbirth. d. If suckling is never begun or is discontinued, then lactation ceases within a few days to a week. e. Little change occurs to the breasts in the first 48 hours.

B, C, D (Breasts become fuller and heavier as colostrum transitions to milk; this fullness should last 72 to 96 hours. The movable, noncancerous mass is a filled milk sac. Colostrum is present for a few days whether or not the mother breastfeeds. A mother who does not want to breastfeed should also avoid stimulating her nipples. Little change to the breasts occurs in the first 24 hours of childbirth.)

A tiered system of categorizing FHR has been recommended by regulatory agencies. Nurses, midwives, and physicians who care for women in labor must have a working knowledge of fetal monitoring standards and understand the significance of each category. These categories include (Select all that apply): a. Reassuring. b. Category I. c. Category II. d. Nonreassuring. e. Category III.

B, C, E The three tiered system of FHR tracings include Category I, II, and III. Category I is a normal tracing requiring no action. Category II FHR tracings are indeterminate. This category includes tracings that do not meet Category I or III criteria. Category III tracings are abnormal and require immediate intervention.

A pregnant woman was admitted for induction of labor at 43 weeks of gestation with sure dates. A nonstress test (NST) in the obstetrician's office revealed a nonreactive tracing. On artificial rupture of membranes, thick meconium-stained fluid was noted. What should the nurse caring for the infant after birth anticipate? (Select all that apply). a. absence of scalp hair, b. hypoglycemia c. meconium aspiration d. excessive vernix caseosa covering the skin e. increased amount of subcutaneous fat f. dry, cracked skin

B, C, F

The postpartum mother asks the nurse why is it so important to prevent cold stress in her baby- can't she shiver to stay warm? What should the nurse include in their response? (Select all that apply) a. Yes, your baby can shiver; but it important that we prevent her from needing to so. b. If your baby gets too cold and we don't help her, her body will use a lot of oxygen to try and get warm, which can ultimately lead to decrease oxygen to her lungs and body causing respiratory problems. c. Your baby will eventually warm up on her own, but it is best if we keep the hat on her head. d. No, your baby cannot shiver; therefore, it is important to prevent her from losing body heat.

B, D

Which of the following can provide contraception and protection against sexually transmitted infections? (Select all that apply). a. natural family planning b. condoms c. diaphragm d. abstinence e. intrauterine devices f. birth control pills

B, D

According to standard professional thinking, nurses should auscultate the fetal heart rate (FHR):

Before and after ambulation and rupture of membranes.

18 year old Ellen has a positive pregnancy test and cries when she sees the result. Per her LMP, the nurse determines she is approx 7 weeks pregnant. She asks the nurse what should she do. Which of the following is the nurse's best initial response? a. You need to decide what obstetrician you want to see for prenatal care. b. I think adoption would be your best choice at this time. c. Let's discuss your three options. d. Here is a list of clinics that provide abortions. My sister liked this one best.

C

A NICU nurse is caring for a full-term neonate being treated for group B streptococcus (GBS). The mother of the neonate is crying and shares that she cannot understand how her baby became infected. The best response by the nurse is: a. "Newborns are more susceptible to infections due to an immature immune system. Would you like additional information on the newborn immune system?" b. "The infection was transmitted to your baby during the birthing process. Do you have a history of sexual transmitted infections?" c. "Approximately 15% to 40% of women have no symptoms but are carriers of group B streptococcus which is found in the vaginal and lower intestinal areas. What other questions do you have regarding your baby's health?" d. "I see that this is very upsetting for you. I will come back later and answer your questions."

C

A client is to take Clomiphene Citrate for infertility. Which of the following is the expected action of this medication? a. Decrease the symptoms of endometriosis b. Increase serum progesterone levels c. Stimulate release of FSH and LH d. Reduce the acidity of vaginal secretions

C

A multipara, 26 weeks' gestation and accompanied by her husband, has just delivered a fetal demise. Which of the following nursing actions is appropriate at this time? a. Encourage the parents to pray for the baby's soul. b. Advise the parents that it is better for the baby to have died than to have had to live with a defect. c. Provide opportunities for grieving parents and family members to spend time with the baby d. Advise the parents to refrain from discussing the baby's death with their other children.

C

A newborn in the neonatal intensive care unit (NICU) is dying as a result of a massive infection. The parents speak to the neonatologist, who informs them of their son's prognosis. When the father sees his son, he says, "He looks just fine to me. I can't understand what all this is about." What is the most appropriate response or reaction by the nurse at this time? a. "You'll have to face up to the fact that he is going to die sooner or later." b. Quietly stand beside the infant's father. c. "This must be a difficult time for you. Tell me how you're doing." d. "Didn't the physician tell you about your son's problems?"

C

A pregnant woman at term is transported to the emergency department (ED) after a severe vehicular accident. The obstetric nurse responds and rushes to the ED with a fetal monitor. Cardiopulmonary arrest occurs as the obstetric nurse arrives. What is the highest priority for the trauma team? a. Quickly applying the fetal monitor to determine whether the fetus viability b. Transferring the woman to the surgical unit for an emergency cesarean delivery in case the fetus is still alive c. Starting cardiopulmonary resuscitation (CPR) d. Obtaining IV access, and starting aggressive fluid resuscitation

C

A pregnant woman has been receiving a magnesium sulfate infusion for treatment of severe preeclampsia for 24 hours. On assessment, the nurse finds the following vital signs: temperature 37.3° C, pulse rate 88 beats per minute, respiratory rate 10 breaths per minute, BP 148/90 mm Hg, absent deep tendon reflexes (DTRs), and no ankle clonus. The client complains, "I'm so thirsty and warm." What is the nurse's immediate action? a. To call for an immediate magnesium sulfate level b. Call the physician c. To discontinue the magnesium sulfate infusion d. To administer oxygen

C

Depo provera, or The Shot, contains which of the following hormones? a. neither estrogen or progesterone b. estrogen c. progesterone d. estrogen and progesterone

C

During a prenatal visit, the nurse is explaining dietary management to a woman with pregestational diabetes. Which statement by the client reassures the nurse that teaching has been effective? a. "I will plan my diet based on the results of urine glucose testing." b. "I will need to eat 600 more calories per day because I am pregnant." c. "Diet and insulin needs change during pregnancy." d. "I can continue with the same diet as before pregnancy as long as it is well balanced."

C

In follow-up appointments or visits with parents and their new baby, it is useful if the nurse can identify infant behaviors that can either facilitate or inhibit attachment. What is an inhibiting behavior? a. The infant cries only when hungry or wet. b. The infant clings to the parents. c. The infant seeks attention from any adult in the room. d. The infant's activity is somewhat predictable

C

Near the end of the first week of life, an infant who has not been treated for any infection develops a copper-colored maculopapular rash on the palms and around the mouth and anus. The newborn is displaying signs and symptoms of which condition? a. HIV b. Herpes simplex virus (HSV) infection c. Congenital syphilis d. Gonorrhea

C

The nurse suspects that her postpartum client is experiencing hemorrhagic shock. Which observation indicates or would confirm this diagnosis? a. Calm mental status b. Cool, dry skin c. Urinary output of at least 30 ml/hr d. Absence of cyanosis in the buccal mucosa

C

To manage her diabetes appropriately and to ensure a good fetal outcome, how would the pregnant woman with diabetes alter her diet? a. Increase her consumption of protein. b. Reduce the carbohydrates in her diet. c. Eat her meals and snacks on a fixed schedule. d. Eat six small equal meals per day.

C

What form of heart disease in women of childbearing years generally has a benign effect on pregnancy? a. Rheumatic heart disease b. Congenital heart disease c. Mitral valve prolapse d. Cardiomyopathy

C

What important, immediate postoperative care practice should the nurse remember when caring for a woman who has had a mastectomy? a. The affected arm should be held down close to the woman's side. b. The affected arm should be used for intravenous (IV) therapy. c. The blood pressure (BP) cuff should not be applied to the affected arm. d. Venipuncture for blood work should be performed on the affected arm

C

When assessing the apical pulse (point of maximal impulse: PMI) of the neonate, the stethoscope should be placed at the: a. First or second intercostal space b. Third intercostal space c. Fourth intercostal space d. Fifth intercostal space

C

When assisting the mother, father, and other family members to actualize the loss of an infant, which action is most helpful? a. Using the words lost or gone rather than dead or died b. Making sure the family understands that naming the baby is important c. Ensuring the baby is clothed or wrapped if the parents choose to visit with the baby d. Setting a firm time for ending the visit with the baby so that the parents know when to let go

C

Which instruction should the nurse provide to reduce the risk of nipple trauma? a. Wash the nipples daily with mild soap and water. b. Limit the feeding time to less than 5 minutes. c. Position the infant so the nipple is far back in the mouth. d. Assess the nipples before each feeding.

C

Which woman has the highest risk for endometrial cancer? a. Woman who has been on birth control pills for 15 years b. Perimenopausal woman who has a cystocele c. Postmenopausal woman with hypertension d. Woman who has an intrauterine device (IUD)

C

A client with maternal phenylketonuria (PKU) has come to the obstetrical clinic to begin prenatal care. Why would this preexisting condition result in the need for closer monitoring during pregnancy? a. A pregnant woman is more likely to die without strict dietary control. b. PKU is a recognized cause of preterm labor. c. The fetus may develop neurologic problems. d. Women with PKU are usually mentally handicapped and should not reproduce.

C pp 707-708

Which hematocrit (Hct) and hemoglobin (Hgb) results represent the lowest acceptable values for a woman in the third trimester of pregnancy? A. 38% Hct; 14 g/dL Hgb B. 35% Hct; 13 g/dL Hgb C. 33% Hct; 11 g/dL Hgb D. 32% Hct; 10.5 g/dL Hgb

C (33% Hct; 11 g/dL Hgb represents the lowest acceptable value during the first and the third trimesters. 38% Hct; 14 g/dL Hgb is within normal limits in the nonpregnant woman. 35% Hct; 13 g/dL Hgb is within normal limits for a nonpregnant woman. 32% Hct; 10.5 g/dL Hgb represents the lowest acceptable value for the second trimester when the hemodilution effect of blood volume expansion is at its peak.)

The nurse is assessing a patient who is pregnant and has diabetes. The Doppler ultrasound examination shows that there is a decrease in the uterine blood flow velocity. Which is the reason for reduced uterine blood flow in the patient? A. Reduced estrogen levels B. Lying in the lateral position C. Low arterial blood pressure D. Relaxation of the uterine muscles

C (An increase in the arterial pressure increases the velocity of blood flow to the uterus. Therefore low arterial pressure decreases the uterine blood flow velocity and thereby decreases the blood supply to the fetus. Supine position of the mother decreases the intervillous blood flow. Therefore lateral position is preferred for sleeping. The blood flow would be the highest in this position, compared with the supine and prone positions. Estrogen has a vasodilator effect. Therefore reduced estrogen levels would decrease the velocity of the uterine blood flow. Contraction of the uterine muscles reduces the blood flow, whereas relaxation of the uterine muscles increases the blood flow.)

Some pregnant clients may complain of changes in their voice and impaired hearing. The nurse can tell these clients that these are common reactions to: A. A decreased estrogen level. B. Displacement of the diaphragm, resulting in thoracic breathing. C. Congestion and swelling, which occur because the upper respiratory tract has become more vascular. D. Increased blood volume.

C (Estrogen levels increase, causing the upper respiratory tract to become more vascular producing swelling and congestion in the nose and ears leading to voice changes and impaired hearing. The diaphragm is displaced, and the volume of blood is increased. However, the main concern is increased estrogen levels.)

The laboratory reports of a pregnant female reveal severe hyponatremia. Which hormone supplementation helps in normalizing sodium levels in the patient? A. Insulin B. Oxytocin C. Aldosterone D. Serum prolactin

C (Hyponatremia is a condition in which the body has low levels of sodium because of excess excretion of sodium. Aldosterone is a hormone that stimulates excess sodium reabsorption from the renal tubules of the kidneys. Therefore the administration of aldosterone is useful for treating hyponatremia. Insulin is the hormone that is used to control blood sugar levels in the body. The hormone oxytocin stimulates uterine contractions and milk ejection from the breasts. Oxytocin is also used to induce labor pain in pregnant women. Serum prolactin prepares the pregnant woman for lactation.)

To reassure and educate pregnant clients about changes in their breasts, nurses should be aware that: A. The visibility of blood vessels that form an intertwining blue network indicates full function of Montgomery's tubercles and possibly infection of the tubercles. B. The mammary glands do not develop until 2 weeks before labor. C. Lactation is inhibited until the estrogen level declines after birth. D. Colostrum is the yellowish oily substance used to lubricate the nipples for breastfeeding

C (Lactation is inhibited until after birth. The visible blue network of blood vessels is a normal outgrowth of a richer blood supply. The mammary glands are functionally complete by midpregnancy. Colostrum is a creamy, white-to-yellow premilk fluid that can be expressed from the nipples before birth.)

A patient at 24 weeks of gestation contacts the nurse at her obstetric provider's office to complain that she has cravings for dirt and gravel. The nurse is aware that this condition is known as ________ and may indicate anemia. A. Ptyalism B. Pyrosis C. Pica D. Decreased peristalsis

C (Pica (a desire to eat nonfood substances) is an indication of iron deficiency and should be evaluated. Ptyalism (excessive salivation), pyrosis (heartburn), and decreased peristalsis are normal findings of gastrointestinal change during pregnancy. Food cravings during pregnancy are normal.)

Following an assessment, the nurse finds that a pregnant female is alcoholic and a smoker. What advice does the nurse give the patient? "Avoid these behaviors because they can: A. Elevate stress during the pregnancy." B. Lead to hemolytic anemia in pregnancy." C. Elevate blood pressure in pregnancy." D. Increase the risk for bleeding during delivery."

C (Smoking and alcohol stimulates the sympathetic nervous system. Thus the heart rate and blood pressure gets increased, which may also affect the fetus. Alcohol and smoking are usually consumed to relieve stress and are not known to induce stress in pregnancy. Hemolytic anemia is a form of anemia that occurs because of hemolysis of red blood cell (RBC). Smoking and alcohol does not cause hemolysis. Bleeding may be caused as a side effect of anticoagulants; it may not be a harmful effect of smoking and alcohol.)

The nurse is assessing a pregnant female in the second trimester. The patient informs the nurse that she feels like eating clay. The nurse refers the patient for a blood test. What could be the reason for this referral? To check for: A. Sodium levels in the blood B. The white blood cell (WBC) count C. Hemoglobin levels in the blood D. Human chorionic gonadotropin (hCG) levels

C (Some pregnant women tend to have nonfood cravings, such as for clay and ice. This condition is referred as pica. This condition is a manifestation of iron deficiency. Iron deficiency can be determined by reduced hemoglobin levels in the blood. Therefore the most likely reason for referring the patient for a blood test is to check for the patient's hemoglobin levels. Alterations in sodium, WBC, and human chorionic gonadotropin levels are not known to be associated with nonfood cravings. Sodium levels are usually checked to assess the filtration ability of the kidney. WBC count is generally increased during pregnancy to around 15,000 cells/mm3 of blood. An increase of WBC count of more than 15,000 cells/mm3 would be suggestive of infection. Increasing levels of hCG in the blood early in pregnancy causes nausea and vomiting.)

The nurse providing care for the laboring woman should understand that late FHR decelerations are caused by: A. Altered cerebral blood flow B. Umbilical cord compression C. Uteroplacental insufficiency D. Meconium fluid

C A. Incorrect: Altered fetal cerebral blood flow would result in early decelerations in the FHR. B. Incorrect: Umbilical cord compression would result in variable decelerations in the FHR. C. Correct: Uteroplacental insufficiency would result in late decelerations in the FHR. D. Incorrect: Meconium-stained fluid may or may not produce changes in the fetal heart rate, depending on the gestational age of the fetus and whether other causative factors associated with fetal distress are present. p. 507

According to standard professional thinking, nurses should auscultate the FHR: A. Every 15 minutes in the active phase of the first stage of labor in the absence of risk factors B. Every 20 minutes in the second stage regardless of whether risk factors are present C. Before and after ambulation and rupture of membranes D. More often in a woman's first pregnancy

C A. Incorrect: In the active phase of the first stage of labor, the FHR should be auscultated every 30 minutes if no risk factors are involved; with risk factors, it should be auscultated every 15 minutes. B. Incorrect: In the second stage of labor, the FHR should be auscultated every 15 minutes if no risk factors are involved; with risk factors, it should be auscultated every 5 minutes. C. Correct: The FHR should be auscultated before and after administration of medications and induction of anesthesia. D. Incorrect: The fetus of a first-time mother is automatically at greater risk. p. 499

The nurse caring for the woman in labor should understand that maternal hypotension can result in: A. Early decelerations B. Fetal dysrhythmias C. Uteroplacental insufficiency D. Spontaneous rupture of membranes

C A. Incorrect: Maternal hypotension is not associated with this condition. B. Incorrect: Maternal hypotension is not associated with this condition. C. Correct: Low maternal blood pressure reduces placental blood flow during uterine contractions, resulting in fetal hypoxemia. D. Incorrect: Maternal hypotension is not associated with this condition. p. 503

The nurse caring for the woman in labor should understand that increased variability of the fetal heart rate might be caused by: A. Narcotics B. Barbiturates C. Methamphetamines D. Tranquilizers

C A. Incorrect: Narcotics, barbiturates, and tranquilizers might be causes of decreased variability; methamphetamines might cause increased variability. B. Incorrect: Narcotics, barbiturates, and tranquilizers might be causes of decreased variability; methamphetamines might cause increased variability. C. Correct: Narcotics, barbiturates, and tranquilizers might be causes of decreased variability; methamphetamines might cause increased variability. D. Incorrect: Narcotics, barbiturates, and tranquilizers might be causes of decreased variability; methamphetamines might cause increased variability. p. 503-504

The nurse caring for a woman in labor understands that prolonged decelerations: A. Are a continuing pattern of benign decelerations that do not require intervention B. Constitute a baseline change when they last longer than 5 minutes C. Usually are isolated events that end spontaneously D. Require the usual fetal monitoring by the nurse

C A. Incorrect: Prolonged decelerations usually are isolated events that end spontaneously. However, in certain combinations with late and/or variable decelerations, they are a danger sign that requires the nurse to notify the physician or midwife immediately. B. Incorrect: A deceleration that lasts longer than 10 minutes constitutes a baseline change. Prolonged decelerations usually are isolated events that end spontaneously. However, in certain combinations with late and/or variable decelerations, they are a danger sign that requires the nurse to notify the physician or midwife immediately. C. Correct: Prolonged decelerations usually are isolated events that end spontaneously. However, in certain combinations with late and/or variable decelerations, they are a danger sign that requires the nurse to notify the physician or midwife immediately. D. Incorrect: Prolonged decelerations usually are isolated events that end spontaneously. However, in certain combinations with late and/or variable decelerations, they are a danger sign that requires the nurse to notify the physician or midwife immediately. p. 509

A number of methods to assist in the assessment of fetal well-being have been developed for use in conjunction with electronic fetal monitoring (EFM). These various technologies assist in supporting interventions for a nonreassuring fetal heart rate pattern when necessary. The labor and delivery nurse should be aware that one of these modalities, fetal oxygen saturation monitoring, includes the use of: A. A fetal acoustic stimulator B. Fetal blood sampling C. Fetal pulse oximetry D. Umbilical cord acid-base determination

C A. Incorrect: Stimulation of the fetus is done in an effort to elicit a fetal heart rate response. The two acceptable methods of achieving this result are the use of fetal scalp stimulation or vibroacoustic stimulation. Vibroacoustic stimulation is performed by using an artificial larynx or fetal acoustic stimulation device over the fetal head for 1 or 2 seconds. B. Incorrect: Sampling of the fetal scalp blood was designed to assess fetal pH, O2, and CO2. The sample is obtained from the fetal scalp through a dilated cervix. This test is usually done in tertiary care centers, where results can be immediately available. It has recently fallen out of favor because test results vary widely. C. Correct: Continuous monitoring of the fetal O2 saturation by fetal pulse oximetry is a method that was approved for clinical use in 2000 by the FDA. This process works in a similar method to obtaining a pulse oximetry in a child or adult. A specially designed sensor is inserted into the uterus and lies against the fetus's temple or cheek. A normal result is 30% to 70%, with 30% being the cutoff for further intervention. D. Incorrect: This test is not completed until after birth. Cord blood is drawn from the umbilical artery and tested for pH, O2, and CO2. pp. 513-516

What is an advantage of external electronic fetal monitoring? A. The ultrasound transducer can accurately measure short-term variability and beat-to-beat changes in the FHR. B. The tocotransducer can measure and record the frequency, regularity, intensity, and approximate duration of UCs. C. The tocotransducer is especially valuable for measuring UA during the first stage of labor. D. Once correctly applied by the nurse, the transducer need not be repositioned even when the woman changes positions.

C A. Incorrect: These short-term changes cannot be measured with this technology. B. Incorrect: The tocotransducer cannot measure and record the intensity of UCs. C. Correct: This is especially true when the membranes are intact. D. Incorrect: The transducer must be repositioned when the woman or the fetus changes position. p. 500

Fetal bradycardia is most common during: A. Intraamniotic infection B. Fetal anemia C. Prolonged umbilical cord compression D. Tocolytic treatment using ritodrine

C A. Incorrect: This circumstance most likely would result in fetal tachycardia. B. Incorrect: This circumstance most likely would result in fetal tachycardia. C. Correct: Fetal bradycardia can be considered a later sign of fetal hypoxia and is known to occur before fetal death. Bradycardia can result from placental transfer of drugs, prolonged compression of the umbilical cord, maternal hypothermia, and maternal hypotension. D. Incorrect: This circumstance most likely would result in fetal tachycardia. p. 503

A primiparous woman is in the taking-in stage of psychosocial recovery and adjustment after childbirth. Recognizing the needs of women during this stage, how should the nurse respond? a. Recognize the woman's limited attention span by giving her written materials to read when she gets home rather than doing a teaching session while she is in the hospital. b. Foster an active role in the baby's care. c. Provide time for the mother to reflect on the events of her labor and delivery. d. Promote maternal independence by encouraging her to meet her own hygiene and comfort needs.

C For the first 24 hours and up to 48 hours the focus is on the self and meeting of basic needs. The mother tends to be excited, talkative, and have a desire to review the birth experience.

A woman with worsening preeclampsia is admitted to the hospital's labor and birth unit. The physician explains the plan of care for severe preeclampsia, including the induction of labor, to the woman and her husband. Which statement by the husband leads the nurse to believe that the couple needs further information? a. "I will give my wife ice chips to eat during labor." b. "Since we will be here for a while, I will call my mother so she can bring the two boys—2 years and 4 years of age—to visit their mother." c. "I will stay with my wife during her labor, just as we planned." d. "I will help my wife use the breathing techniques that we learned in our childbirth classes."

C Need to maintain quiet, low stimulant environment.

A woman who is 12 weeks postpartum presents with the following behavior: she reports severe mood swings and hearing voices, believes her infant is going to die, she has to be reminded to shower and put on clean clothes, and she feels she is unable to care for her baby. These behaviors are associated with which of the following? a. Postpartum blues b. Postpartum depression c. Postpartum psychosis d. Maladaptive mother-infant attachment

C Postpartum psychosis (PPP) is a variant of bipolar disorder and is the most serious form of postpartum mood disorders. Onset of symptoms can be as early as the 3rd postpartum day. Assessment findings include paranoia, delusions associated with the baby, mood swings, extreme agitation, confused thinking, inability to care for self or infant, and strange beliefs.

A woman who is 30 weeks of gestation arrives at the hospital with bleeding. Which differential diagnosis would not be applicable for this client? a. Placenta previa b. Cord insertion c. Spontaneous abortion d. Abruptio placentae

C Spontaneous abortion occurs prior to 20 weeks

A 26-year-old pregnant woman, gravida 2, para 1-0-0-1, is 28 weeks pregnant when she experiences bright red, painless vaginal bleeding. On her arrival at the hospital, which diagnostic procedure will the client most likely have performed? a. Contraction stress test (CST) b. Internal fetal monitoring c. Transvaginal ultrasound for placental location d. Amniocentesis for fetal lung maturity

C The symptoms, bright red and painlessvaginal bleeding, indicate placental previa. An ultrasound can confirm this diagnosis.

A nurse notes that an Eskimo woman does not cuddle or interact with her newborn other than to feed him, change his diapers or soiled clothes, and put him to bed. While evaluating this client's behavior with her infant, what realization does the nurse make? Select one: a. The woman needs a referral to a social worker for further evaluation of her parenting behaviors once she goes home with the newborn. b. The woman is inexperienced in caring for a newborn. c. What appears to be a lack of interest in the newborn is, in fact, the cultural way of demonstrating intense love by attempting to ward off evil spirits. d. Extra time needs to be planned for assisting the woman in bonding with her newborn

C What appears to be a lack of interest in the newborn is, in fact, the cultural way of demonstrating intense love by attempting to ward off evil spirits.

A primigravida has just delivered a healthy infant girl. The nurse is about to administer erythromycin ointment in the infant's eyes when the mother asks, "What is that medicine for?" The nurse responds: A. "It is an eye ointment to help your baby see you better." B. "It is to protect your baby from contracting herpes from your vaginal tract." C. "Erythromycin is given prophylactically to prevent a gonorrheal infection." D. "This medicine will protect your baby's eyes from drying out over the next few days."

C A. Incorrect: Erythromycin has no bearing on enhancing vision. B. Incorrect: Erythromycin is used to prevent an infection caused by gonorrhea, not herpes. C. Correct: With the prophylactic use of erythromycin, the incidence of gonococcal conjunctivitis has declined to less than 0.5%. Eye prophylaxis is administered at or shortly after birth to prevent ophthalmia neonatorum. D. Incorrect: Erythromycin is given to prevent infection, not for lubrication. p. 1004

With regard to central nervous system injuries to the infant during labor and birth, nurses should be aware that: A. Intracranial hemorrhage (ICH) as a result of birth trauma is more likely to occur in the preterm, low-birth-weight infant. B. Subarachnoid hemorrhage (the most common form of ICH) occurs in term infants as a result of hypoxia. C. In many infants, signs of hemorrhage in a full-term infant are absent and diagnosed only through laboratory tests. D. Spinal cord injuries almost always result from forceps-assisted deliveries.

C A. Incorrect: ICH as a result of birth trauma is more likely to occur in the full-term, large infant. B. Incorrect: Subarachnoid hemorrhage in term infants is a result of trauma; in preterm infants, it is a result of hypoxia. C. Correct: Abnormalities in lumbar punctures or red blood cell counts, for instance, or in visuals on CT scan might reveal a hemorrhage. D. Incorrect: Spinal cord injuries are almost always from breech births; they are rare today because cesarean birth often is used for breech presentation. p. 995

Infants of mothers with diabetes are at higher risk for developing: A. Anemia B. Hyponatremia C. Respiratory distress syndrome D. Sepsis

C A. Incorrect: Infants of diabetic mothers (IDMs) are not at risk for anemia. They are at risk for polycythemia. B. Incorrect: IDMs are not at risk for hyponatremia. They are at risk for hypocalcemia and hypomagnesemia. C. Correct: IDMs are at risk for macrosomia, birth trauma, perinatal asphyxia, respiratory distress syndrome, hypoglycemia, hypocalcemia, hypomagnesemia, cardiomyopathy, hyperbilirubinemia, and polycythemia. D. Incorrect: IDMs are not at risk for sepsis. p. 996

With regard to congenital abnormalities involving the central nervous system, nurses should be aware that: A. Although the death rate from most congenital anomalies has decreased over the past several decades, neural tube defects (NTDs) have gone up in the last few years. B. Spina bifida cystica usually is asymptomatic and may not be diagnosed unless associated problems are present. C. A major preoperative nursing intervention for a neonate with myelomeningocele is to protect the protruding sac from injury. D. Microcephaly can be corrected with timely surgery.

C A. Incorrect: Most congenital anomalies have had a stable neonatal death rate since the 1930s; NTDs are declining because of mandatory food fortification with folic acid. B. Incorrect: Spina bifida occulta often is asymptomatic; spina bifida cystica has a visible sac. C. Correct: The nurse protects the infant by laying the baby on his or her side. D. Incorrect: Microcephaly is a tiny head; there is no treatment. p. 1036

Providing care for the neonate born to a mother who abuses substances can present a challenge for the health care team. Nursing care for this infant requires a multisystem approach. The first step in the provision of this care is: A. Pharmacologic treatment B. Reduction of environmental stimuli C. Neonatal abstinence syndrome scoring D. Adequate nutrition and maintenance of fluid and electrolyte balance

C A. Incorrect: Pharmacologic treatment is based on the severity of withdrawal symptoms. Symptoms are determined by using a standard assessment tool. Medications of choice are morphine, phenobarbital, diazepam, or diluted tincture of opium. B. Incorrect: Swaddling, holding, and reducing environmental stimuli are essential in providing care to the infant who is experiencing withdrawal. These nursing interventions are appropriate for the infant who displays central nervous system disturbances. C. Correct: Neonatal abstinence syndrome (NAS) is the term used to describe the cohort of symptoms associated with drug withdrawal in the neonate. The Neonatal Abstinence Scoring System evaluates CNS, metabolic, vasomotor, respiratory, and gastrointestinal disturbances. This evaluation tool enables the care team to develop an appropriate plan of care. The infant is scored throughout the length of stay and the treatment plan is adjusted accordingly. D. Incorrect: Poor feeding is one of the GI symptoms common to this client population. Fluid and electrolyte balance must be maintained and adequate nutrition provided. These infants often have a poor suck reflex and may need to be fed via gavage. pp. 1017-1019

A plan of care for an infant experiencing symptoms of drug withdrawal should include: A. Administering chloral hydrate for sedation B. Feeding every 4 to 6 hours to allow extra rest C. Swaddling the infant snugly and holding the baby tightly D. Playing soft music during feeding

C A. Incorrect: Phenobarbital or diazepam may be administered to decrease CNS irritability. B. Incorrect: The infant should be fed in small, frequent amounts and burped well to diminish aspiration and maintain hydration. C. Correct: The infant should be wrapped snugly to reduce self-stimulation behaviors and protect the skin from abrasions. D. Incorrect: The infant should not be stimulated (such as with music), because this will increase activity and potentially increase CNS irritability. p. 1017

In caring for the mother who has abused (or is abusing) alcohol and for her infant, nurses should be aware that: A. The pattern of growth restriction of the fetus begun in prenatal life is halted after birth, and normal growth takes over. B. Two-thirds of newborns with fetal alcohol syndrome (FAS) are boys. C. Alcohol-related neurodevelopmental disorders (ARND) not sufficient to meet FAS criteria (learning disabilities, speech and language problems) are often not detected until the child goes to school. D. Both the distinctive facial features of the FAS infant and the diminished mental capacities tend toward normal over time.

C A. Incorrect: The pattern of growth restriction persists after birth. B. Incorrect: Two-thirds of newborns with FAS are girls. C. Correct: Some learning problems do not become evident until the child is at school. D. Incorrect: Although the distinctive facial features of the FAS infant tend to become less evident, the mental capacities never become normal. p. 1013

Near the end of the first week of life, an infant who has not been treated for any infection develops a copper-colored, maculopapular rash on the palms and around the mouth and anus. The newborn is showing signs of: A. Gonorrhea B. Herpes simplex virus infection C. Congenital syphilis D. HIV

C A. Incorrect: The rash is indicative of congenital syphilis. The lesions may extend over the trunk and extremities. B. Incorrect: The rash is indicative of congenital syphilis. The lesions may extend over the trunk and extremities. C. Correct: The rash is indicative of congenital syphilis. The lesions may extend over the trunk and extremities. D. Incorrect: The rash is indicative of congenital syphilis. The lesions may extend over the trunk and extremities. p. 1005

An infant was born 2 hours ago at 37 weeks of gestation, weighing 4.1 kg. The infant appears chubby with a flushed complexion and is very tremulous. The tremors are most likely the result of: A. Birth injury B. Hypocalcemia C. Hypoglycemia D. Seizures

C A. Incorrect: This infant is macrosomic and at risk for hypoglycemia. The description is indicative of a macrocosmic infant. The tremors are jitteriness that is associated with hypoglycemia. B. Incorrect: This infant is macrosomic and at risk for hypoglycemia. The description is indicative of a macrocosmic infant. The tremors are jitteriness that is associated with hypoglycemia. C. Correct: Hypoglycemia is common in the macrosomic infant. Signs of hypoglycemia include jitteriness, apnea, tachypnea, and cyanosis. D. Incorrect: This infant is macrosomic and at risk for hypoglycemia. The description is indicative of a macrocosmic infant. The tremors are jitteriness that is associated with hypoglycemia. p. 998

15. The nurse is teaching new parents about metabolic screening for the newborn. Which statement is most helpful to these clients? a. All states test for phenylketonuria (PKU), hypothyroidism, cystic fibrosis, and sickle cell diseases. b. Federal law prohibits newborn genetic testing without parental consent. c. If genetic screening is performed before the infant is 24 hours old, then it should be repeated at age 1 to 2 weeks. d. Hearing screening is now mandated by federal law.

C (If testing is performed before the infant is 24 hours old, then genetic screening should be repeated when the infant is 1 to 2 weeks old. All states test for PKU and hypothyroidism but not for other genetic defects. Federal law mandates newborn genetic screening; however, parents can decline the testing. A waiver should be signed, and a notation made in the infants medical record. Federal law does not mandate screening for hearing problems; however, the majority of states have enacted legislation mandating newborn hearing screening. In the United States, the majority (95%) of infants are screened for hearing loss before discharge from the hospital.)

18. If the newborn has excess secretions, the mouth and nasal passages can be easily cleared with a bulb syringe. How should the nurse instruct the parents on the use of this instrument? a. Avoid suctioning the nares. b. Insert the compressed bulb into the center of the mouth. c. Suction the mouth first. d. Remove the bulb syringe from the crib when finished.

C (The mouth should always be suctioned first to prevent the infant from inhaling pharyngeal secretions by gasping as the nares are suctioned. After compressing the bulb, the syringe should be inserted into one side of the mouth. If it is inserted into the center of the mouth, then the gag reflex is likely to be initiated. When the infants cry no longer sounds as though it is through mucus or a bubble, suctioning can be stopped. The nasal passages should be suctioned one nostril at a time. The bulb syringe should remain in the crib so that it is easily accessible if needed again.)

4. What is the rationale for the administration of vitamin K to the healthy full-term newborn? a. Most mothers have a diet deficient in vitamin K, which results in the infant being deficient. b. Vitamin K prevents the synthesis of prothrombin in the liver and must be administered by injection. c. Bacteria that synthesize vitamin K are not present in the newborns intestinal tract. d. The supply of vitamin K in the healthy full-term newborn is inadequate for at least 3 to 4 months and must be supplemented.

C (Vitamin K is provided because the newborn does not have the intestinal flora to produce this vitamin for the first week. The maternal diet has no bearing on the amount of vitamin K found in the newborn. Vitamin K promotes the formation of clotting factors in the liver and is used for the prevention and treatment of hemorrhagic disease in the newborn. Vitamin K is not produced in the intestinal tract of the newborn until after microorganisms are introduced. By day 8, normal newborns are able to produce their own vitamin K.)

9. The nurse is performing a gestational age and physical assessment on the newborn. The infant appears to have an excessive amount of saliva. This clinical finding may be indicative of what? a. Excessive saliva is a normal finding in the newborn. b. Excessive saliva in a neonate indicates that the infant is hungry. c. It may indicate that the infant has a tracheoesophageal fistula or esophageal atresia. d. Excessive saliva may indicate that the infant has a diaphragmatic hernia.

C (The presence of excessive saliva in a neonate should alert the nurse to the possibility of a tracheoesophageal fistula or esophageal atresia. Excessive salivation may not be a normal finding and should be further assessed for the possibility that the infant has an esophageal abnormality. The hungry infant reacts by making sucking motions, rooting, or making hand-to-mouth movements. The infant with a diaphragmatic hernia exhibits severe respiratory distress.)

Fetal bradycardia is most common during: a. Intraamniotic infection. b. Fetal anemia. c. Prolonged umbilical cord compression. d. Tocolytic treatment using terbutaline.

C Fetal bradycardia can be considered a later sign of fetal hypoxia and is known to occur before fetal death. Bradycardia can result from placental transfer of drugs, prolonged compression of the umbilical cord, maternal hypothermia, and maternal hypotension. Intraamniotic infection, fetal anemia, and tocolytic treatment using terbutaline would most likely result in fetal tachycardia.

The nurse caring for the woman in labor should understand that maternal hypotension can result in: a. Early decelerations. c. Uteroplacental insufficiency. b. Fetal dysrhythmias. d. Spontaneous rupture of membranes.

C Low maternal blood pressure reduces placental blood flow during uterine contractions and results in fetal hypoxemia. Maternal hypotension is not associated with early decelerations, fetal dysrhythmias, or spontaneous rupture of membranes.

The nurse caring for a woman in labor understands that prolonged decelerations: a. Are a continuing pattern of benign decelerations that do not require intervention. b. Constitute a baseline change when they last longer than 5 minutes. c. Usually are isolated events that end spontaneously. d. Require the usual fetal monitoring by the nurse.

C Prolonged decelerations usually are isolated events that end spontaneously. However, in certain combinations with late and/or variable decelerations, they are a danger sign that requires the nurse to notify the physician or midwife immediately. A deceleration that lasts longer than 10 minutes constitutes a baseline change.

According to standard professional thinking, nurses should auscultate the fetal heart rate (FHR): a. Every 15 minutes in the active phase of the first stage of labor in the absence of risk factors. b. Every 20 minutes in the second stage, regardless of whether risk factors are present. c. Before and after ambulation and rupture of membranes. d. More often in a woman's first pregnancy.

C The FHR should be auscultated before and after administration of medications and induction of anesthesia. In the active phase of the first stage of labor, the FHR should be auscultated every 30 minutes if no risk factors are involved; with risk factors it should be auscultated every 15 minutes. In the second stage of labor the FHR should be auscultated every 15 minutes if no risk factors are involved; with risk factors it should be auscultated every 5 minutes. The fetus of a first-time mother is automatically at greater risk.

The uterine contractions of a woman early in the active phase of labor are assessed by an internal uterine pressure catheter (IUPC). The nurse notes that the intrauterine pressure at the peak of the contraction ranges from 65 to 70 mm Hg and the resting tone range is 6 to 10 mm Hg. The uterine contractions occur every 3 to 4 minutes and last an average of 55 to 60 seconds. On the basis of this information, the nurse should: a. Notify the woman's primary health care provider immediately. b. Prepare to administer an oxytocic to stimulate uterine activity. c. Document the findings because they reflect the expected contraction pattern for the active phase of labor. d. Prepare the woman for the onset of the second stage of labor.

C The nurse is responsible for monitoring the uterine contractions to ascertain whether they are powerful and frequent enough to accomplish the work of expelling the fetus and the placenta. In addition, the nurse would document these findings in the client's medical record. This labor pattern indicates that the client is in the active phase of the first stage of labor. Nothing indicates a need to notify the primary care provider at this time. Oxytocin augmentation is not needed for this labor pattern; this contraction pattern indicates adequate active labor. Her contractions eventually will become stronger, last longer, and come closer together during the transition phase of the first stage of labor. The transition phase precedes the second stage of labor, or delivery of the fetus.

What is an advantage of external electronic fetal monitoring? a. The ultrasound transducer can accurately measure short-term variability and beat-to-beat changes in the fetal heart rate. b. The tocotransducer can measure and record the frequency, regularity, intensity, and approximate duration of uterine contractions (UCs). c. The tocotransducer is especially valuable for measuring uterine activity during the first stage of labor. d. Once correctly applied by the nurse, the transducer need not be repositioned even when the woman changes positions.

C The tocotransducer is especially valuable for measuring uterine activity during the first stage of labor, particularly when the membranes are intact. Short-term changes cannot be measured with this technology. The tocotransducer cannot measure and record the intensity of UCs. The transducer must be repositioned when the woman or fetus changes position.

The nurse providing care for the laboring woman should understand that late fetal heart rate (FHR) decelerations are the result of: a. Altered cerebral blood flow. c. Uteroplacental insufficiency. b. Umbilical cord compression. d. Meconium fluid.

C Uteroplacental insufficiency would result in late decelerations in the FHR. Altered fetal cerebral blood flow would result in early decelerations in the FHR. Umbilical cord compression would result in variable decelerations in the FHR. Meconium-stained fluid may or may not produce changes in the fetal heart rate, depending on the gestational age of the fetus and whether other causative factors associated with fetal distress are present.

3. A nurse is assessing a newborn girl who is 2 hours old. Which finding warrants a call to the health care provider? a. Blood glucose of 45 mg/dl using a Dextrostix screening method b. Heart rate of 160 beats per minute after vigorously crying c. Laceration of the cheek d. Passage of a dark black-green substance from the rectum

C (Accidental lacerations can be inflicted by a scalpel during a cesarean birth. They are most often found on the scalp or buttocks and may require an adhesive strip for closure. Parents would be overly concerned about a laceration on the cheek. A blood glucose level of 45 mg/dl and a heart rate of 160 beats per minute after crying are both normal findings that do not warrant a call to the physician. The passage of meconium from the rectum is an expected finding in the newborn.)

13. Which statement accurately describes an appropriate-for-gestational age (AGA) weight assessment? a. AGA weight assessment falls between the 25th and 75th percentiles for the infants age. b. AGA weight assessment depends on the infants length and the size of the newborns head. c. AGA weight assessment falls between the 10th and 90th percentiles for the infants age. d. AGA weight assessment is modified to consider intrauterine growth restriction (IUGR).

C (An AGA weight falls between the 10th and 90th percentiles for the infants age. The AGA range is larger than the 25th and 75th percentiles. The infants length and head size are measured, but these measurements do not affect the normal weight designation. IUGR applies to the fetus, not to the newborns weight.)

1. An infant boy was delivered minutes ago. The nurse is conducting the initial assessment. Part of the assessment includes the Apgar score. When should the Apgar assessment be performed? a. Only if the newborn is in obvious distress b. Once by the obstetrician, just after the birth c. At least twice, 1 minute and 5 minutes after birth d. Every 15 minutes during the newborns first hour after birth

C (Apgar scoring is performed at 1 minute and at 5 minutes after birth. Scoring may continue at 5-minute intervals if the infant is in distress and requires resuscitation efforts. The Apgar score is performed on all newborns. Apgar score can be completed by the nurse or the birth attendant. The Apgar score permits a rapid assessment of the newborns transition to extrauterine life. An interval of every 15 minutes is too long to wait to complete this assessment.)

22. A mother expresses fear about changing her infants diaper after he is circumcised. What does the client need to be taught to care for her newborn son? a.Cleanse the penis with prepackaged diaper wipes every 3 to 4 hours. b.Apply constant, firm pressure by squeezing the penis with the fingers for at least 5 minutes if bleeding occurs. c.Gently cleanse the penis with water and apply petroleum jelly around the glans after each diaper change. d.Wash off the yellow exudate that forms on the glans at least once every day to prevent infection.

C (Gently cleansing the penis with water and applying petroleum jelly around the glans after each diaper change are appropriate techniques when caring for an infant who has had a circumcision. With each diaper change, the penis should be washed with warm water to remove any urine or feces. If bleeding occurs, then the mother should apply gentle pressure to the site of the bleeding with a sterile gauze square. Yellow exudates are part of normal healing and cover the glans penis 24 hours after the circumcision; yellow exudates are not an infective process and should not be removed.)

11. What is the primary rationale for nurses wearing gloves when handling the newborn? a. To protect the baby from infection b. As part of the Apgar protocol c. To protect the nurse from contamination by the newborn d. Because the nurse has the primary responsibility for the baby during the first 2 hours

C (With the possibility of transmission of viruses such as HBV and the human immunodeficiency virus (HIV) through maternal blood and amniotic fluid, the newborn must be considered a potential contamination source until proven otherwise. As part of Standard Precautions, nurses should wear gloves when handling the newborn until blood and amniotic fluid are removed by bathing. Proper hand hygiene is all that is necessary to protect the infant from infection. Wearing gloves is not necessary to complete the Apgar score assessment. The nurse assigned to the mother-baby couplet has primary responsibility for the newborn, regardless of whether or not she wears gloves.)

20. While discussing the societal impacts of breastfeeding, the nurse should be cognizant of the benefits and educate the client accordingly. Which statement as part of this discussion would be incorrect? a.Breastfeeding requires fewer supplies and less cumbersome equipment. b.Breastfeeding saves families money. c.Breastfeeding costs employers in terms of time lost from work. d.Breastfeeding benefits the environment.

C (Actually, less time is lost to work by breastfeeding mothers, in part because infants are healthier. Breastfeeding is convenient because it does not require cleaning or transporting bottles and other equipment. It saves families money because the cost of formula far exceeds the cost of extra food for the lactating mother. Breastfeeding uses a renewable resource; it does not need fossil fuels, advertising, shipping, or disposal.)

27. As the nurse assists a new mother with breastfeeding, the client asks, If formula is prepared to meet the nutritional needs of the newborn, what is in breast milk that makes it better? What is the nurses best response? a.More calories b.Essential amino acids c.Important immunoglobulins d.More calcium

C (Breast milk contains immunoglobulins that protect the newborn against infection. The calorie count of formula and breast milk is approximately the same. All the essential amino acids are in both formula and breast milk; however, the concentrations may differ. Calcium levels are higher in formula than in breast milk, which can cause an excessively high renal solute load if the formula is not properly diluted.)

21. What should the nurses next action be if the clients white blood cell (WBC) count is 25,000/mm3 on her second postpartum day? a. Immediately inform the physician. b. Have the laboratory draw blood for reanalysis. c. Recognize that this count is an acceptable range at this point postpartum. d. Immediately begin antibiotic therapy.

C (During the first 10 to 12 days after childbirth, WBC values between 20,000 and 25,000/mm3 are common. Because a WBC count of 25,000/mm3 on her second postpartum day is normal, alerting the physician is not warranted nor is reassessment or antibiotics needed; the WBC count is not elevated.)

20. Which statement, related to the reconditioning of the urinary system after childbirth, should the nurse understand? a. Kidney function returns to normal a few days after birth. b. Diastasis recti abdominis is a common condition that alters the voiding reflex. c. Fluid loss through perspiration and increased urinary output accounts for a weight loss of more than 2 kg during the puerperium. d. With adequate emptying of the bladder, bladder tone is usually restored 2 to 3 weeks after childbirth.

C (Excess fluid loss through other means besides perspiration and increased urinary output occurs as well. Kidney function usually returns to normal in approximately 1 month. Diastasis recti abdominis is the separation of muscles in the abdominal wall and has no effect on the voiding reflex. Bladder tone is usually restored 5 to 7 days after childbirth.)

12. Which statement regarding the nutrient needs of breastfed infants is correct? a.Breastfed infants need extra water in hot climates. b.During the first 3 months, breastfed infants consume more energy than formula-fed infants. c.Breastfeeding infants should receive oral vitamin D drops daily during at least the first 2 months. d.Vitamin K injections at birth are not necessary for breastfed infants.

C (Human milk contains only small amounts of vitamin D. All infants who are breastfed should receive 400 International Units of vitamin D each day. Neither breastfed nor formula-fed infants need to be fed water, not even in very hot climates. During the first 3 months, formula-fed infants consume more energy than breastfed infants and therefore tend to grow more rapidly. Vitamin K shots are required for all infants because the bacteria that produce it are absent from the babys stomach at birth.)

15. Which information should the nurse provide to a breastfeeding mother regarding optimal self-care? a.She will need an extra 1000 calories a day to maintain energy and produce milk. b.She can return to prepregnancy consumption patterns of any drinks as long as she gets enough calcium. c.She should avoid trying to lose large amounts of weight. d.She must avoid exercising because it is too fatiguing.

C (Large weight loss releases fat-stored contaminants into her breast milk, and it also involves eating too little and/or exercising too much. A breastfeeding mother needs to add only 200 to 500 extra calories to her diet to provide the extra nutrients for her infant. However, this is true only if she does not drink alcohol, limits coffee to no more than two cups (including caffeine in chocolate, tea, and some sodas, too), and carefully reads the herbal tea ingredients. Although she needs her rest, moderate exercise is healthy.)

8. A new mother wants to be sure that she is meeting her daughters needs while feeding the baby commercially prepared infant formula. The nurse should evaluate the mothers knowledge about appropriate infant feeding techniques. Which statement by the client reassures the nurse that correct learning has taken place? a.Since reaching 2 weeks of age, I add rice cereal to my daughters formula to ensure adequate nutrition. b.I warm the bottle in my microwave oven. c.I burp my daughter during and after the feeding as needed. d.I refrigerate any leftover formula for the next feeding.

C (Most infants swallow air when fed from a bottle and should be given a chance to burp several times during and after the feeding. Solid food should not be introduced to the infant for at least 4 to 6 months after birth. A microwave should never be used to warm any food to be given to an infant. The heat is not distributed evenly, which may pose a risk of burning the infant. Any formula left in the bottle after the feeding should be discarded because the infants saliva has mixed with it.)

26. Which action by the mother will initiate the milk ejection reflex (MER)? a.Wearing a firm-fitting bra b.Drinking plenty of fluids c.Placing the infant to the breast d.Applying cool packs to her breast

C (Oxytocin, which causes the MER reflex, increases in response to nipple stimulation. A firm bra is important to support the breast; however, it will not initiate the MER reflex. Drinking plenty of fluids is necessary for adequate milk production, but adequate intake of water alone will not initiate the MER reflex. Cool packs to the breast will decrease the MER reflex.)

5. Which hormone remains elevated in the immediate postpartum period of the breastfeeding woman? a. Estrogen b. Progesterone c. Prolactin d. Human placental lactogen

C (Prolactin levels in the blood progressively increase throughout pregnancy. In women who breastfeed, prolactin levels remain elevated into the sixth week after birth. Estrogen levels decrease significantly after expulsion of the placenta, reaching their lowest levels 1 week into the postpartum period. Progesterone levels decrease significantly after expulsion of the placenta, reaching their lowest levels 1 week into the postpartum period. Human placental lactogen levels dramatically decrease after expulsion of the placenta.)

24. Which type of formula is not diluted with water, before being administered to an infant? a.Powdered b.Concentrated c.Ready-to-use d.Modified cows milk

C (Ready-to-use formula can be poured directly from the can into the babys bottle and is good (but expensive) when a proper water supply is not available. Formula should be well mixed to dissolve the powder and make it uniform in consistency. Improper dilution of concentrated formula may cause malnutrition or sodium imbalances. Cows milk is more difficult for the infant to digest and is not recommended, even if it is diluted.)

22. Which documentation on a womans chart on postpartum day 14 indicates a normal involution process? a. Moderate bright red lochial flow b. Breasts firm and tender c. Fundus below the symphysis and nonpalpable d. Episiotomy slightly red and puffy

C (The fundus descends 1 cm per day; consequently, it is no longer palpable by postpartum day 14. The lochia should be changed by this day to serosa. Breasts are not part of the involution process. The episiotomy should not be red or puffy at this stage.)

21. In assisting the breastfeeding mother to position the baby, which information regarding positioning is important for the nurse to keep in mind? a.The cradle position is usually preferred by mothers who had a cesarean birth. b.Women with perineal pain and swelling prefer the modified cradle position. c.Whatever the position used, the infant is belly to belly with the mother. d.While supporting the head, the mother should push gently on the occiput.

C (The infant naturally faces the mother, belly to belly. The football position is usually preferred after a cesarean birth. Women with perineal pain and swelling prefer the side-lying position because they can rest while breastfeeding. The mother should never push on the back of the head. It may cause the baby to bite, hyperextend the neck, or develop an aversion to being brought near the breast.)

6. A newborn is jaundiced and is receiving phototherapy via ultraviolet bank lights. What is the most appropriate nursing intervention when caring for an infant with hyperbilirubinemia and receiving phototherapy? a. Applying an oil-based lotion to the newborns skin to prevent dying and cracking b. Limiting the newborns intake of milk to prevent nausea, vomiting, and diarrhea c. Placing eye shields over the newborns closed eyes d. Changing the newborns position every 4 hours

C (The infants eyes must be protected by an opaque mask to prevent overexposure to the light. Eye shields should completely cover the eyes but not occlude the nares. Lotions and ointments should not be applied to the infant because they absorb heat and can cause burns. The lights increase insensible water loss, placing the infant at risk for fluid loss and dehydration. Therefore, adequate hydration is important for the infant. The infant should be turned every 2 hours to expose all body surfaces to the light.)

5. A breastfeeding woman develops engorged breasts at 3 days postpartum. What action will help this client achieve her goal of reducing the engorgement? a.Skip feedings to enable her sore breasts to rest. b.Avoid using a breast pump. c.Breastfeed her infant every 2 hours. d.Reduce her fluid intake for 24 hours.

C (The mother should be instructed to attempt feeding her infant every 2 hours while massaging the breasts as the infant is feeding. Skipping feedings may cause further swelling and discomfort. If the infant does not adequately feed and empty the breast, then the mother may pump to extract the milk and relieve some of the discomfort. Dehydration further irritates swollen breast tissue.)

6. Two days ago a woman gave birth to a full-term infant. Last night she awakened several times to urinate and noted that her gown and bedding were wet from profuse diaphoresis. Which physiologic alteration is the cause for the diaphoresis and diuresis that this client is experiencing? a. Elevated temperature caused by postpartum infection b. Increased basal metabolic rate after giving birth c. Loss of increased blood volume associated with pregnancy d. Increased venous pressure in the lower extremities

C (Within 12 hours of birth, women begin to lose the excess tissue fluid that has accumulated during pregnancy. One mechanism for reducing these retained fluids is the profuse diaphoresis that often occurs, especially at night, for the first 2 or 3 days after childbirth. Postpartal diuresis is another mechanism by which the body rids itself of excess fluid. An elevated temperature causes chills and possibly dehydration, not diaphoresis and diuresis. Diaphoresis and diuresis are sometimes referred to as reversal of the water metabolism of pregnancy, not as the basal metabolic rate. Postpartal diuresis may be caused by the removal of increased venous pressure in the lower extremities.)

18. According to demographic research, which woman is least likely to breastfeed and therefore most likely to need education regarding the benefits and proper techniques of breastfeeding? a.Between 30 and 35 years of age, Caucasian, and employed part time outside the home b.Younger than 25 years of age, Hispanic, and unemployed c.Younger than 25 years of age, African-American, and employed full time outside the home d.35 years of age or older, Caucasian, and employed full time at home

C (Women least likely to breastfeed are typically younger than 25 years of age, have a lower income, are less educated, are employed full time outside the home, and are African-American.)

16 year old Emily lives in Blackfoot, ID. She has a boyfriend but does not want to have a baby for at least 10 years. She wants an effective, easy birth control method and denies medical problems. She has periods every 4-6 weeks. The nurse would recommend which of the following methods to Emily? (Select all that apply). a. bilateral tubal ligation b. natural family planning c. hormonal IUD d. the implant

C, D

2. Changes in blood volume after childbirth depend on several factors such as blood loss during childbirth and the amount of extravascular water (physiologic edema) mobilized and excreted. What amount of blood loss does the postpartum nurse anticipate? (Select all that apply.) a. 100 ml b. 250 ml or less c. 300 to 500 ml d. 500 to 1000 ml e. 1500 ml or greater

C, D (The average blood loss for a vaginal birth of a single fetus ranges from 300 to 500 ml (10% of blood volume). The typical blood loss for women who gave birth by cesarean is 500 to 1000 ml (15% to 30% of blood volume). During the first few days after childbirth, the plasma volume further decreases as a result of diuresis. Pregnancy-induced hypervolemia (i.e., an increase in blood volume of at least 35%) allows most women to tolerate considerable blood loss during childbirth.)

During pregnancy, many changes occur as a direct result of the presence of the fetus. Which of these adaptations meet this criteria? A. Leukorrhea B. Development of the operculum C. Quickening D. Ballottement E. Lightening

C, D, E (Leukorrhea is a white or slightly gray vaginal discharge that develops in response to cervical stimulation by estrogen and progesterone. Quickening is the first recognition of fetal movements or "feeling life." Quickening is often described as a flutter and is felt earlier in multiparous women than in primiparas. Lightening occurs when the fetus begins to descend into the pelvis. This occurs 2 weeks before labor in the nullipara and at the start of labor in the multipara. Mucus fills the cervical canal creating a plug otherwise known as the operculum. The operculum acts as a barrier against bacterial invasion during the pregnancy. Passive movement of the unengaged fetus is referred to as ballottement. Select all that apply)

If a woman is at risk for thrombus and is not ready to ambulate, which nursing intervention would the nurse use? (Select all that apply.) a. Having her sit in a chair b. Promoting bed rest c. Having her flex, extend, and rotate her feet, ankles, and legs d. Immediately notifying the physician if a positive Homans sign occurs e. Putting her in antiembolic stockings (thromboembolic deterrent [TED] hose) and/or sequential compression device (SCD) boots

C, D, E Prophylactics for thrombus include ambulation or movement, antiembolic stockings, and/or sequential compression device. A positive Homans sign is when passive dorsiflextion of the foot results in calf pain that is often associated with warmth and redness of the calf.

Which of the following is true regarding physiologic jaundice? (Select all that apply). a. Jaundice is noted within 24 hours after birth b. It is recommended the mother stop breastfeeding until jaundice resolves c. Jaundice is noted 24 hours after birth d. Jaundice persists longer than 14 days of life in a term infant e. Jaundice is more common in preterm infants

C, E

The baseline fetal heart rate (FHR) is the average rate during a 10-minute segment. Changes in FHR are categorized as periodic or episodic. These patterns include both accelerations and decelerations. The labor nurse is evaluating the patient's most recent 10-minute segment on the monitor strip and notes a late deceleration. This is likely to be caused by which physiologic alteration (Select all that apply)? a. Spontaneous fetal movement b. Compression of the fetal head c. Placental abruption d. Cord around the baby's neck e. Maternal supine hypotension

C, E Late decelerations are almost always caused by uteroplacental insufficiency. Insufficiency is caused by uterine tachysystole, maternal hypotension, epidural or spinal anesthesia, IUGR, intraamniotic infection, or placental abruption. Spontaneous fetal movement, vaginal examination, fetal scalp stimulation, fetal reaction to external sounds, uterine contractions, fundal pressure and abdominal palpation are all likely to cause accelerations of the FHR. Early decelerations are most often the result of fetal head compression and may be caused by uterine contractions, fundal pressure, vaginal examination, and placement of an internal electrode. A variable deceleration is likely caused by umbilical cord compression. This may happen when the umbilical cord is around the baby's neck, arm, leg, or other body part or when there is a short cord, a knot in the cord, or a prolapsed cord.

When assessing the relative advantages and disadvantages of internal and external electronic fetal monitoring, nurses comprehend that both:

Can be used during the antepartum and intrapartum periods.

A tiered system of categorizing FHR has been recommended by regulatory agencies. Nurses, midwives, and physicians who care for women in labor must have a working knowledge of fetal monitoring standards and understand the significance of each category. These categories include: (**Not sure what they're asking or what the options are! I guess just know what the categories are?!)

Category I: normal tracing requiring no action Category II: FHR tracings are indeterminate Category III: abnormal; require immediate intervention

While evaluating an external monitor tracing of a woman in active labor, the nurse notes that the fetal heart rate for five sequential contractions begins to decelerate late in the contraction, with the nadir of the decelerations occurring after the peak of the contraction. The nurse's first priority is to:

Change the woman's position. [Late decelerations may be caused by maternal supine hypotension syndrome.]

Perinatal nurses are legally responsible for:

Correctly interpreting fetal heart rate (FHR) patterns, initiating appropriate nursing interventions, and documenting the outcomes

A client in late middle age who is certain she is not pregnant tells the nurse during an office visit that she has urinary problems, as well as sensations of bearing down and of something in her vagina. What condition would the nurse suspect based upon this report? a. Pelvic relaxation b. Genital fistulas c. Uterine prolapse d. Cystocele and/or rectocele

D

A client is diagnosed with having a stillborn infant. At first, she appears stunned by the news, cries a little, and then asks the nurse to call her mother. What is the proper term for the phase of bereavement that this client is experiencing? a. Anticipatory grief b. Reorganization c. Intense grief d. Acute distress

D

A nurse is providing education to a support group of women newly diagnosed with breast cancer. It is important for the nurse to discuss which factor related to breast cancer with the group? a. Genetic mutations account for 50% of women who will develop breast cancer. b. In the United States, 1 in 10 women will develop breast cancer in her lifetime. c. Breast cancer is the leading cause of cancer death in women. d. The exact cause of breast cancer remains unknown.

D

A premature infant never seems to sleep longer than an hour at a time. Each time a light is turned on, an incubator closes, or people talk near her crib, she wakes up and inconsolably cries until held. What is the correct nursing diagnosis beginning with "ineffective coping, related to"? a. Severe immaturity b. Behavioral responses c. Physiologic distress d. Environmental stress

D

The client is being induced in response to worsening preeclampsia. She is also receiving magnesium sulfate. It appears that her labor has not become active, despite several hours of oxytocin administration. She asks the nurse, "Why is this taking so long?" What is the nurse's most appropriate response? a. "I don't know why it is taking so long." b. "Your baby is just being stubborn." c. "The length of labor varies for different women." d. "The magnesium is relaxing your uterus and competing with the oxytocin. It may increase the duration of your labor."

D

The nurse is performing an assessment on a client who thinks she may be experiencing preterm labor. Which information is the most important for the nurse to understand and share with the client? a. Because preterm labor is likely to be the start of an extended labor, a woman with symptoms can wait several hours before contacting the primary caregiver. b. Braxton Hicks contractions often signal the onset of preterm labor. c. Because all women must be considered at risk for preterm labor and prediction is so variable, teaching pregnant women the symptoms of preterm labor probably causes more harm through false alarms. d. Diagnosis of preterm labor is based on gestational age, uterine activity, and progressive cervical change.

D

The nurse is preparing to administer methotrexate to the client. This drug is most often used for which obstetric complication? a. Abruptio placentae b. Complete hydatidiform mole c. Missed abortion d. Unruptured ectopic pregnancy

D

While taking a family history, 37 y/o Bernice tells the nurse her paternal grandmother had breast and colon cancer in her 60's, two paternal aunts had breast cancer in their 40's, one of those aunts also had ovarian cancer in her 40's, one paternal uncle had colon cancer in his 40's, her paternal cousin has breast cancer in her 40's, and her father had colon cancer in his 40's. She denies cancer on her mother's side of the family. Bernice states she has 2 younger sisters. Which of the following responses would be best for the nurse to say to Bernice about her reported family history? a. "Since all of these cancers are on your father's side of the family, you have no increased risk for cancer." b. "Have you had a screening mammogram?" c. "Do you have cancer on your mother's side of the family?" d. "The types of cancers reported on your father's side of the family may be due to a gene mutation that can be passed onto to each generation. Have any of your relatives or you considered genetic testing?" e. "Have you had your pap smear recently

D

A patient reports to the nurse that she had missed her period this month and suspects that she is a pregnant. What would be the most suitable nursing action for this patient? A. Assess for Hegar sign. B. Assess for Chadwick sign. C. Obtain an order for a urine pregnancy test. D. Obtain an order for a serum pregnancy test.

D (Because the woman has missed her period, it is likely that the woman is 4 to 6 weeks pregnant. A serum pregnancy test helps in the earliest detection of pregnancy. This test can be used to detect pregnancy in women who are 4 weeks pregnant. Therefore the nurse should ask the patient to take the serum pregnancy test. It is performed during weeks 4 to 12 of pregnancy. Hegar sign and Chadwick signs will be observed during weeks 6 to 12 of pregnancy, and pelvic congestion may be the other cause for such signs. Urine pregnancy test gives positive results during weeks 6 to 12 of pregnancy.)

The nurse is assessing a 5-month pregnant female and learns that the patient smokes. The nurse instructs the patient to quit smoking. What could be the possible reason for giving this instruction? The patient: A. Has supine hypotensive syndrome. B. Is at risk for developing osteoporosis. C. Is found to have gestational diabetes. D. Has carpal tunnel syndrome in the right hand.

D (Carpal tunnel syndrome is characterized by paresthesia and pain in the hand radiating to the elbow. Smoking and alcohol consumption impairs the microcirculation and worsens the symptoms of the syndrome. Smoking does not worsen the signs of supine hypotensive syndrome, osteoporosis, and gestational diabetes. If the patient had supine hypotensive syndrome, then the nurse would have suggested the patient to lie on the lateral position. If the patient had osteoporosis, then the nurse would have suggested the intake of calcium supplements. Gestational diabetes is a common condition in pregnant women and it disappears after childbirth.)

To reassure and educate their pregnant clients about changes in their blood pressure, maternity nurses should be aware that: A. A blood pressure cuff that is too small produces a reading that is too low; a cuff that is too large produces a reading that is too high. B. Shifting the client's position and changing from arm to arm for different measurements produces the most accurate composite blood pressure reading at each visit. C. The systolic blood pressure increases slightly as pregnancy advances; the diastolic pressure remains constant. D. Compression of the iliac veins and inferior vena cava by the uterus contributes to hemorrhoids in the later stage of term pregnancy.

D (Compression of the iliac veins and inferior vena cava also leads to varicose veins in the legs and vulva. The tightness of a cuff that is too small produces a reading that is too high; similarly the looseness of a cuff that is too large results in a reading that is too low. Because maternal positioning affects readings, blood pressure measurements should be obtained in the same arm and with the woman in the same position. The systolic blood pressure generally remains constant but may decline slightly as pregnancy advances. The diastolic blood pressure first decreases and then gradually increases.)

If exhibited by a pregnant woman, what represents a positive sign of pregnancy? A. Morning sickness B. Quickening C. Positive pregnancy test D. Fetal heartbeat auscultated with Doppler/fetoscope

D (Detection of a fetal heartbeat, palpation of fetal movements and parts by an examiner, and detection of an embryo/fetus with sonographic examination are positive signs diagnostic of pregnancy . Morning sickness and quickening, along with amenorrhea and breast tenderness, are presumptive signs of pregnancy; subjective findings are suggestive but not diagnostic of pregnancy. Other probable signs include changes in integument, enlargement of the uterus, and Chadwick sign. A positive pregnancy test is considered to be a probable sign of pregnancy (objective findings are more suggestive but not yet diagnostic of pregnancy) because error can occur in performing the test or, in rare cases, human chorionic gonadotropin (hCG) may be detected in the urine of nonpregnant women. Chances of error are less likely to occur today because pregnancy tests used are easy to perform and are very sensitive to the presence of the hCG associated with pregnancy.)

Human chorionic gonadotropin (hCG) is an important biochemical marker for pregnancy and the basis for many tests. A maternity nurse should be aware that: A. hCG can be detected 2.5 weeks after conception. B. The hCG level increases gradually and uniformly throughout pregnancy. C. Much lower than normal increases in the level of hCG may indicate a postdate pregnancy. D. A higher than normal level of hCG may indicate an ectopic pregnancy or Down syndrome.

D (Higher levels also could be a sign of multiple gestation. hCG can be detected 7 to 8 days after conception. The hCG level fluctuates during pregnancy: peaking, declining, stabilizing, and increasing again. Abnormally slow increases may indicate impending miscarriage.)

The biochemical reports of a pregnant woman show an increase in the metabolism of glucose and increased fatty acid deposition of the body. Which hormone is responsible for these changes in the patient? A. Insulin B. Estrogen C. Parathyroid D. Human chorionic somatotropin

D (Human chorionic somatotropin decreases the maternal metabolism of glucose and increases the production of fatty acids for metabolic needs. A decrease in the metabolism of glucose and increased fatty acid deposition is caused by the decrease in human chorionic somatotropin. The metabolism of glucose and fatty acid deposition is not affected by the defect in insulin, estrogen, and parathyroid. In pregnant females, insulin is produced to repress the effect of insulin antagonism by placental hormones. A defect in insulin does not lead to the increase of metabolism in glucose. Estrogen is responsible for fatty acid deposition but is not involved in glucose metabolism. Parathyroid hormone controls the metabolism of calcium and magnesium.)

The nurse is caring for a 3-month pregnant woman who reports, "I always feel very thirsty." What does the nurse infer from the patient's statement? The patient: A. Consumes less fiber in the diet. B. Takes high amounts of fat in the diet. C. Has high sodium content in the blood. D. Has increased loss of water from the body.

D (In early pregnancy, the kidneys have increased capacity to excrete water. Therefore the patient may feel thirsty because of increased loss of water. A low-fiber diet may cause constipation in pregnant females. Fiber does not interfere with the water levels in the body. Consumption of fatty foods in proper amounts is necessary in pregnancy, and fatty foods usually do not cause excess thirst. Sodium ions trigger fluid retention in the body and do not cause thirst.)

To reassure and educate pregnant clients about changes in the cervix, vagina, and position of the fetus, nurses should be aware that: A. Because of a number of changes in the cervix, abnormal Papanicolaou (Pap) tests are much easier to evaluate. B. Quickening is a technique of palpating the fetus to engage it in passive movement. C. The deepening color of the vaginal mucosa and cervix (Chadwick's sign) usually appears in the second trimester or later as the vagina prepares to stretch during labor. D. Increased vascularity of the vagina increases sensitivity and may lead to a high degree of arousal, especially in the second trimester.

D (Increased sensitivity and an increased interest in sex sometimes go together. This frequently occurs during the second trimester. Cervical changes make evaluation of abnormal Pap tests more difficult. Quickening is the first recognition of fetal movements by the mother. Ballottement is a technique used to palpate the fetus. Chadwick's sign appears from the sixth to eighth weeks.)

Which statement about a condition of pregnancy is accurate? A. Insufficient salivation (ptyalism) is caused by increases in estrogen. B. Acid indigestion (pyrosis) begins early but declines throughout pregnancy. C. Hyperthyroidism often develops (temporarily) because hormone production increases. D. Nausea and vomiting rarely have harmful effects on the fetus and may be beneficial.

D (Normal nausea and vomiting rarely produce harmful effects, and nausea and vomiting periods may be less likely to result in miscarriage or preterm labor. Ptyalism is excessive salivation, which may be caused by a decrease in unconscious swallowing or stimulation of the salivary glands. Pyrosis begins in the first trimester and intensifies through the third trimester. Increased hormone production does not lead to hyperthyroidism in pregnant women.)

The nurse is caring for a pregnant patient who is in the third trimester. The patient reports a burning sensation starting from the hands to the elbow. On further evaluation, the nurse finds compression in the carpal ligament of the wrist. What finding does the nurse infer from examining the patient? A. Sciatica B. Neuralgia C. Acroesthesia D. Paresthesia

D (Paresthesia is an abnormal sensation that is perceived as a burning and tingling in the skin. This is caused by edema that compresses the nerves. Edema in carpal ligament of the wrist causes carpal tunnel syndrome, which is characterized by paresthesia. Sciatica is a burning pain that is felt in the back, buttocks, and leg when the sciatic nerve is irritated. Neuralgia is a stabbing, burning pain that occurs along a damaged nerve. Acroesthesia is the numbness and tingling of the hands caused by stoop-shouldered stance.)

The nurse caring for the pregnant client must understand that the hormone essential for maintaining pregnancy is: A. Estrogen. B. Human chorionic gonadotropin (hCG). C. Oxytocin. D. Progesterone.

D (Progesterone is essential for maintaining pregnancy; it does so by relaxing smooth muscles. This reduces uterine activity and prevents miscarriage. Estrogen plays a vital role in pregnancy, but it is not the primary hormone for maintaining pregnancy. hCG levels increase at implantation but decline after 60 to 70 days. Oxytocin stimulates uterine contractions.)

Numerous changes in the integumentary system occur during pregnancy. Which change persists after birth? A. Epulis B. Telangiectasia C. Chloasma D. Striae gravidarum

D (Striae gravidarum, or stretch marks, reflect separation within the underlying connective tissue of the skin. They usually fade after birth, although they never disappear completely. An epulis is a red, raised nodule on the gums that bleeds easily. Chloasma, or mask of pregnancy, is a blotchy, brown hyperpigmentation of the skin over the cheeks, nose, and forehead, especially in dark-complexioned pregnant women. Chloasma usually fades after the birth. Telangiectasia, or vascular spiders, are tiny, star-shaped or branchlike, slightly raised, pulsating end-arterioles usually found on the neck, thorax, face, and arms. They occur as a result of elevated levels of circulating estrogen. These usually disappear after birth.)

The nurse is using the New Ballard Scale to determine the gestational age of a newborn. Which assessment finding is consistent with a gestational age of 40 weeks? a. Smooth, pink skin with visible veins b. Faint red marks on the soles of the feet c. Abundant lanugo d. Flexed posture

D 553

If the newborn has excess secretions, the mouth and nasal passages can be easily cleared with a bulb syringe. How should the nurse instruct the parents on the use of this instrument? a. Insert the compressed bulb into the center of the mouth. b. Remove the bulb syringe from the crib when finished. c. Avoid suctioning the nares. d. Suction the mouth first

D 567

Which FHR finding would concern the nurse during labor? A. Accelerations with fetal movement B. Early decelerations C. An average FHR of 126 beats/min D. Late decelerations

D A. Incorrect: Accelerations in the FHR are an indication of fetal well-being. B. Incorrect: Early decelerations in the FHR are associated with head compression as the fetus descends into the maternal pelvic outlet; they generally are not a concern during normal labor. C. Incorrect: This FHR finding is normal and not a concern. D. Correct: Late decelerations are caused by uteroplacental insufficiency and are associated with fetal hypoxemia. They are considered ominous if persistent and uncorrected. p. 507

The most common cause of decreased variability in the FHR that lasts 30 minutes or less is: A. Altered cerebral blood flow B. Fetal hypoxemia C. Umbilical cord compression D. Fetal sleep cycles

D A. Incorrect: Altered fetal cerebral blood flow would result in early decelerations in the FHR. B. Incorrect: Fetal hypoxemia would be evidenced by tachycardia initially and then bradycardia. A persistent decrease or loss of FHR variability may be seen. C. Incorrect: Umbilical cord compression would result in variable decelerations in the FHR. D. Correct: A temporary decrease in variability can occur when the fetus is in a sleep state. These sleep states do not usually last longer than 30 minutes. p. 502

As a perinatal nurse, you realize that an FHR that is tachycardic, bradycardic, has late decelerations, or loss of variability is nonreassuring and is associated with: A. Hypotension B. Cord compression C. Maternal drug use D. Hypoxemia

D A. Incorrect: Fetal bradycardia may be associated with maternal hypotension. B. Incorrect: Fetal variable decelerations are associated with cord compression. C. Incorrect: Maternal drug use is associated with fetal tachycardia. D. Correct: Nonreassuring heart rate patterns are associated with fetal hypoxemia. pp. 502-503

A nurse might be called on to stimulate the fetal scalp: A. As part of fetal scalp blood sampling B. In response to tocolysis C. In preparation for fetal oxygen saturation monitoring D. To elicit an acceleration in the FHR

D A. Incorrect: Fetal scalp blood sampling involves swabbing the scalp with disinfectant before a sample is collected. The nurse would stimulate the fetal scalp to elicit an acceleration of the FHR. B. Incorrect: Tocolysis is relaxation of the uterus. The nurse would stimulate the fetal scalp to elicit an acceleration of the FHR. C. Incorrect: Fetal oxygen saturation monitoring involves the insertion of a sensor. The nurse would stimulate the fetal scalp to elicit an acceleration of the FHR. D. Correct: The scalp can be stimulated using digital pressure during a vaginal examination. p. 513

When assessing the relative advantages and disadvantages of internal and external electronic fetal monitoring, nurses should be aware that both: A. Can be used when membranes are intact B. Measure the frequency, duration, and intensity of UCs C. May need to rely on the woman to indicate when UA is occurring D. Can be used during the antepartum and intrapartum periods

D A. Incorrect: For internal monitoring, the membranes must have ruptured and the cervix must be sufficiently dilated. B. Incorrect: Internal monitoring measures the intensity of contractions; external monitoring cannot do this. C. Incorrect: With external monitoring, the woman may need to alert the nurse that UA is occurring; internal monitoring does not require this. D. Correct: External monitoring can be used in both periods; internal monitoring can be used only in the intrapartum period. p. 500

Nurses should be aware that accelerations in the fetal heart rate: A. Are indications of fetal well-being when they are periodic B. Are greater and longer in preterm gestations C. Are usually seen with breech presentations when they are episodic D. May visibly resemble the shape of the uterine contraction

D A. Incorrect: Periodic accelerations occur with UC and usually are seen with breech presentations. Episodic accelerations occur during fetal movement and are indications of fetal well-being. B. Incorrect: Preterm accelerations peak at 10 beats/min above the baseline and last for at least 10 seconds, not 15 seconds. C. Incorrect: Periodic accelerations occur with UC and usually are seen with breech presentations. Episodic accelerations occur during fetal movement and are indications of fetal well-being. D. Correct: They may resemble the shape of the uterine contraction or may be spikelike. p. 507

While evaluating an external monitor tracing of a woman in active labor whose labor is being induced, the nurse notes that the FHR begins to decelerate at the onset of several contractions and returns to baseline before each contraction ends. The nurse should: A. Change the woman's position B. Discontinue the oxytocin infusion C. Insert an internal monitor D. Document the finding in the client's record

D A. Incorrect: The FHR indicates early decelerations, which are not an ominous sign and do not require any intervention. The nurse should simply document these findings. B. Incorrect: The presence of early decelerations is not an ominous sign and does not require any intervention. C. Incorrect: The presence of early decelerations is not an ominous sign and does not require any intervention. D. Correct: The FHR indicates early decelerations, which are not an ominous sign and do not require any intervention. The nurse should simply document these findings. p. 509

You are evaluating the fetal monitor tracing of your client, who is in active labor. Suddenly, you see the FHR drop from its baseline of 125 down to 80. You reposition the mother, provide oxygen, increase intravenous (IV) fluid, and perform a vaginal exam. The cervix has not changed. Five minutes have passed, and the fetal heart rates remain in the 80s. What additional nursing measures should you take? A. Scream for help B. Insert a Foley catheter C. Start pitocin D. Notify the care provider immediately

D A. Incorrect: This is an inappropriate nursing action. B. Incorrect: If the FHR were to continue in a nonreassuring pattern, a cesarean section may be warranted, which would require a Foley catheter. However, the physician must make that determination. C. Incorrect: Pitocin may put additional stress on the fetus. D. Correct: To relieve an FHR deceleration, the nurse can reposition the mother, increase IV fluid, and provide oxygen. Also, if oxytocin is infusing, it should be discontinued. If the FHR does not resolve, the primary care provider should be notified immediately. p. 511

When using IA for FHR, nurses should be aware that: A. They can be expected to cover only two or three clients when IA is the primary method of fetal assessment. B. The best course is to use the descriptive terms associated with EFM when documenting results. C. If the heartbeat cannot be found immediately, a shift must be made to electronic monitoring. D. Ultrasound can be used to find the fetal heartbeat and reassure the mother if initial difficulty was a factor.

D A. Incorrect: When used as the primary method of fetal assessment, auscultation requires a nurse-to-client ratio of one to one. B. Incorrect: Documentation should use only terms that can be numerically defined; the usual visual descriptions of EFM are inappropriate. C. Incorrect: Locating fetal heartbeats often takes time. Mothers can be reassured verbally and by the ultrasound pictures if that device is used to help locate the heartbeat. D. Correct: Locating fetal heartbeats often takes time. Mothers can be reassured verbally and by the ultrasound pictures if that device is used to help locate the heartbeat. p. 500

As part of their teaching function at discharge, nurses should educate parents regarding safe sleep. Based on the most recent evidence, which information is incorrect and should be discussed with parents? a. Keep the infant away from secondhand smoke. b. Prevent exposure to people with upper respiratory tract infections. c. Avoid loose bedding, water beds, and beanbag chairs. d. Place the infant on his or her abdomen to sleep

D Back to Sleep- Infants should always be placed on their backs to sleep.

A 25-year-old gravida 1 para 1 who had an emergency cesarean birth 3 days ago is scheduled for discharge. As the nurse prepares her for discharge, she begins to cry. The nurse's next action should be what? a. Explain that she is experiencing postpartum blues. b. Point out how lucky she is to have a healthy baby. c. Assess her for pain. d. Allow her time to express her feelings

D The nurse needs to hear from the patient why she is crying before offering a response.

Parents have been asked by the neonatologist to provide breast milk for their newborn son, who was born prematurely at 32 weeks of gestation. The nurse who instructs them regarding pumping, storing, and transporting the milk needs to assess their knowledge of lactation. Which statement is valid? a. The mother should pump every 2 to 3 hours, including during the night. b. The mother should only pump as much milk as the infant can drink. c. A glass of wine just before pumping will help reduce stress and anxiety. d. Premature infants more easily digest breast milk than formula.

D The question asks- The nurse who instructs them regarding pumping, storing, and transporting the milk needs to assess their knowledge of lactation. Which statement is valid? This answer best reflects knowledge of lactation, as indicated in this statement.

A woman gave birth to an infant boy 10 hours ago. Where does the nurse expect to locate this woman's fundus? a. Midway between the umbilicus and the symphysis pubis b. 2 centimeters below the umbilicus c. Nonpalpable abdominally d. 1 centimeter above the umbilicus

D Within 12 hours the fundus can rise to approximately 1 cm above the umbilicus. By 24 hours after birth the uterus is about the same size as it was at 20 weeks of gestation. The fundus descends 1 to 2cm every 24 hours. The uterus should not be palpable abominally after 2 weeks and should have returned to its nonpregannt location by 6 weeks after birth.

The priority nursing diagnosis for a newborn diagnosed with a diaphragmatic hernia would be: A. Risk for impaired parent-infant attachment B. Imbalanced nutrition: less than body requirements C. Risk for infection D. Impaired gas exchange

D A. Incorrect: Although this issue may be a factor in providing care to a newborn with a diaphragmatic hernia, the priority nursing diagnosis relates to the oxygenation issues arising from the lung hypoplasia that occurs with diaphragmatic hernia. B. Incorrect: Although this issue may be a factor in providing care to a newborn with a diaphragmatic hernia, the priority nursing diagnosis relates to the oxygenation issues arising from the lung hypoplasia that occurs with diaphragmatic hernia. C. Incorrect: Although this issue may be a factor in providing care to a newborn with a diaphragmatic hernia, the priority nursing diagnosis relates to the oxygenation issues arising from the lung hypoplasia that occurs with diaphragmatic hernia. D. Correct: Herniation of the abdominal viscera into the thoracic cavity may cause severe respiratory distress and represent a neonatal emergency. Oxygen therapy, mechanical ventilation, and the correction of acidosis are necessary in infants with large defects. p. 1038

What bacterial infection is definitely decreasing because of effective drug treatment? A. Escherichia coli infection B. Tuberculosis C. Candidiasis D. Group B streptococcal infection

D A. Incorrect: E. coli may be increasing, perhaps because of the increasing use of ampicillin (resulting in a more virulent E. coli resistant to the drug). Group B streptococcus has been beaten back by penicillin. B. Incorrect: Tuberculosis is increasing in the United States and in Canada. Group B streptococcus has been beaten back by penicillin. C. Incorrect: Candidiasis is a fairly benign fungal infection. Group B streptococcus has been beaten back by penicillin. D. Correct: Penicillin has significantly decreased the incidence of group B streptococcal infection. pp. 1010-1011

A macrosomic infant is born after a difficult, forceps-assisted delivery. After stabilization, the infant is weighed, and the birth weight is 4550 g (9 pounds, 6 ounces). The nurse's most appropriate action is to: A. Leave the infant in the room with the mother B. Take the infant immediately to the nursery C. Perform a gestational age assessment to determine whether the infant is large for gestational age D. Monitor blood glucose levels frequently and observe closely for signs of hypoglycemia

D A. Incorrect: Macrosomic infants are at high risk for hypoglycemia after birth and need to be observed closely. This can be achieved in the mother's room with nursing interventions, depending on the condition of the fetus. It may be more appropriate for observation to occur in the nursery. B. Incorrect: Macrosomic infants are at high risk for hypoglycemia after birth and need to be observed closely. Observation may occur in the nursery or in the mother's room, depending on the condition of the fetus. C. Incorrect: Regardless of gestational age, this infant is macrosomic. Macrosomia is defined as fetal weight over 4000 g. Hypoglycemia affects many macrosomic infants. Blood glucose levels should be observed closely. D. Correct: This infant is macrosomic (over 4000 g) and is at high risk for hypoglycemia. Blood glucose levels should be monitored frequently, and the infant should be observed closely for signs of hypoglycemia. p. 997

With regard to hemolytic diseases of the newborn, nurses should be aware that: A. Rh incompatibility matters only when an Rh-negative offspring is born to an Rh-positive mother. B. ABO incompatibility is more likely than Rh incompatibility to precipitate significant anemia. C. Exchange transfusions frequently are required in the treatment of hemolytic disorders. D. The indirect Coombs' test is performed on the mother before birth; the direct Coombs' test is performed on the cord blood after birth.

D A. Incorrect: Only the Rh-positive offspring of an Rh-negative mother is at risk. B. Incorrect: ABO incompatibility is more common than Rh incompatibility but causes less severe problems; significant anemia, for instance, is rare with ABO. C. Incorrect: Exchange transfers are needed infrequently because of the decrease in the incidence of severe hemolytic disease in newborns from Rh incompatibility. D. Correct: An indirect Coombs' test may be performed on the mother a few times during pregnancy. p. 1028

What finding supports the diagnosis of pathologic jaundice? A. Serum bilirubin concentrations greater than 2 mg/dl in cord blood B. Serum bilirubin levels increasing more than 1 mg/dl in 24 hours C. Serum bilirubin levels greater than 10 mg/dl in a full-term newborn D. Clinical jaundice evident within 24 hours of birth

D A. Incorrect: Serum bilirubin concentrations greater than 4 mg/dl in cord blood would support a diagnosis of pathologic jaundice. B. Incorrect: Total serum bilirubin levels that increase by more than 5 mg/dl in 24 hours would support a diagnosis of pathologic jaundice. C. Incorrect: A serum bilirubin level in a preterm newborn that exceeds 10 mg/dl would support a diagnosis of pathologic jaundice. D. Correct: Clinical jaundice evident within 24 hours of birth would support a diagnosis of pathologic jaundice. p. 1025

As a home care nurse, you are visiting a 5-day-old male infant for a scheduled follow-up appointment to ensure that he is responding to home phototherapy for treatment of jaundice. Based on the diagnosis of hyperbilirubinemia, you are aware that the development of acute bilirubin encephalopathy is a risk for this infant. This disease process occurs after the bilirubin level has peaked. After completing a thorough assessment and obtaining a history from the parents, you recognize that this infant is in the first phase of encephalopathy when he exhibits: A. A high-pitched cry B. Severe muscle spasms (opisthotonos) C. Fever and seizures D. Hypotonia, lethargy, and poor suck

D A. Incorrect: Should the infant display symptoms such as a high-pitched cry, severe muscle spasms, hyperreflexia, or arching of the back, the nurse should be aware that the baby has progressed beyond the more subtle signs of the first phase. Medical attention is necessary immediately. B. Incorrect: Should the infant display symptoms such as a high-pitched cry, severe muscle spasms, hyperreflexia, or arching of the back, the nurse should be aware that the baby has progressed beyond the more subtle signs of the first phase. Medical attention is necessary immediately. C. Incorrect: Symptoms may progress from the subtle indications of the first phase to fever and seizures in as little as 24 hours. Only about half of these infants survive and will have permanent sequelae including auditory deficiencies, intellectual deficits, and movement abnormalities. D. Correct: The early and most subtle symptoms of bilirubin encephalopathy include hypotonia, lethargy, poor suck, and depressed or absent Moro reflex. pp. 999, 1027, 1028

When attempting to diagnose and treat developmental dysplasia of the hip (DDH), the nurse should: A. Be able to perform the Ortolani and Barlow tests B. Teach double or triple diapering for added support C. Explain to the parents the need for serial casting D. Carefully monitor infants for DDH at follow-up visits

D A. Incorrect: The Ortolani and Barlow tests must be performed by experienced clinicians to prevent fracture or other damage to the hip. B. Incorrect: Double or triple diapering is not recommended, because it promotes hip extension, thus worsening the problem. C. Incorrect: Serial casting is done for clubfeet, not DDH. D. Correct: Because DDH often is not detected at birth, infants should be carefully monitored at follow-up visits. p. 1044

A pregnant woman at 37 weeks of gestation has had ruptured membranes for 26 hours. A cesarean section is performed for failure to progress. The fetal heart rate before birth is 180 beats/min with limited variability. At birth, the newborn has Apgar scores of 6 and 7 at 1 and 5 minutes and is noted to be pale and tachypneic. Based on the maternal history, the cause of this newborn's distress is most likely to be: A. Hypoglycemia B. Phrenic nerve injury C. Respiratory distress syndrome D. Sepsis

D A. Incorrect: The prolonged rupture of membranes is the most indicative clinical cue to this infant's condition. An FHR of 180 beats/min is also indicative. This infant is at high risk for sepsis. B. Incorrect: The prolonged rupture of membranes is the most indicative clinical cue to this infant's condition. An FHR of 180 beats/min is also indicative. This infant is at high risk for sepsis. C. Incorrect: The prolonged rupture of membranes is the most indicative clinical cue to this infant's condition. An FHR of 180 beats/min is also indicative. This infant is at high risk for sepsis. D. Correct: The prolonged rupture of membranes and the tachypnea (before and after birth) both suggest sepsis. p. 1001

In order to provide comprehensive newborn care, the nurse should understand that kernicterus occurs if: A. The kidney excretes bilirubin. B. Bilirubin collects in the liver. C. Bilirubin deposits are concentrated in the cardiac muscle. D. Bilirubin deposits are in the brain.

D A. Incorrect: The term kernicterus is synonymous with bilirubin encephalopathy. It is caused by the deposition of bilirubin in the brain. B. Incorrect: The term kernicterus is synonymous with bilirubin encephalopathy. It is caused by the deposition of bilirubin in the brain. C. Incorrect: The term kernicterus is synonymous with bilirubin encephalopathy. It is caused by the deposition of bilirubin in the brain. D. Correct: Kernicterus describes the chronic and permanent results of bilirubin toxicity. p. 1025

With smaller families and increased genetic screening, many couples have come to expect a perfect baby. Mothers tend to have the greatest and most difficult adjustment to a child with unexpected disabilities. A metaanalysis of families in the United States and Canada has revealed that there are four developmental milestones that the mothers of "differently abled" children need to achieve. At a follow-up office visit, the nurse knows that she needs to listen carefully to the mother's cues in order to determine how well she is coping. Which phase has this mother reached when she states, "Don't you agree that my daughter has made a lot of progress since her last visit?" A. Becoming the mother of a disabled child B. Learning a new maternal role C. Realizing that daily life will never be the same D. Acceptance/denial

D A. Incorrect: This phase includes solving the puzzle of what is wrong, diminished interest in the mothering role, grief for loss of an ideal, learning to trust the health care system, and looking for blame. B. Incorrect: In the second phase the mother has to come to grips with the role of caregiver burden, finding support, protecting the child against prejudice, and the intensity of mothering a disabled child. C. Incorrect: This third phase includes adaptation of routine, control, change, mastering uncertainty, grief for lost choices, and identifying realistic goals. D. Correct: This is the fourth phase and is evidenced by the mother redefining normal, looking for progress, hope, strength, and life enrichment. The paradox is accepting the child for who she is, while never giving up hope. p. 1032

HIV may be perinatally transmitted: A. Only in the third trimester from the maternal circulation B. By a needlestick injury at birth from unsterile instruments C. Only through the ingestion of amniotic fluid D. Through the ingestion of breast milk from an infected mother

D A. Incorrect: Transmission of HIV from the mother to the infant may occur transplacentally at various gestational ages. Transmission close to or at the time of birth is thought to account for 50% to 80% of cases. B. Incorrect: Transmission of HIV may occur through the placenta from the mother to the fetus or through breast milk postnatally. C. Incorrect: Transmission of HIV may occur through the placenta from the mother to the fetus or through breast milk postnatally. D. Correct: Postnatal transmission of HIV through breastfeeding may occur. p. 1006

19. As part of the infant discharge instructions, the nurse is reviewing the use of the infant car safety seat. Which information is the highest priority for the nurse to share? a. Infant carriers are okay to use until an infant car safety seat can be purchased. b. For traveling on airplanes, buses, and trains, infant carriers are satisfactory. c. Infant car safety seats are used for infants only from birth to 15 pounds. d. Infant car seats should be rear facing and placed in the back seat of the car.

D (An infant placed in the front seat could be severely injured by an air bag that deploys during an automobile accident. Infants should travel only in federally approved, rear-facing safety seats secured in the rear seat and only in federally approved safety seats even when traveling on a commercial vehicle. Infants should use a rear-facing car seat from birth to 20 pounds and to age 1 year.)

24. How should the nurse interpret an Apgar score of 10 at 1 minute after birth? a. The infant is having no difficulty adjusting to extrauterine life and needs no further testing. b. The infant is in severe distress and needs resuscitation. c. The nurse predicts a future free of neurologic problems. d. The infant is having no difficulty adjusting to extrauterine life but should be assessed again at 5 minutes after birth.

D (An initial Apgar score of 10 is a good sign of healthy adaptation; however, the test must be repeated at the 5-minute mark.)

29. Screening for critical congenital heart disease (CCHD) was added to the uniform screening panel in 2011. The nurse has explained this testing to the new mother. Which action by the nurse related to this test is correct? a. Screening is performed when the infant is 12 hours of age. b. Testing is performed with an electrocardiogram. c. Oxygen (O2) is measured in both hands and in the right foot. d. A passing result is an O2 saturation of 95%.

D (Screening is performed when the infant is between 24 and 48 hours of age. The test is performed using pulse oximetry technology. O2 is measured in the right hand and one foot. A passing result is an O2 saturation of 95% with a 3% absolute difference between upper and lower extremity readings.)

26. As part of their teaching function at discharge, nurses should educate parents regarding safe sleep. Based on the most recent evidence, which information is incorrect and should be discussed with parents? a. Prevent exposure to people with upper respiratory tract infections. b. Keep the infant away from secondhand smoke. c. Avoid loose bedding, water beds, and beanbag chairs. d. Place the infant on his or her abdomen to sleep.

D (The infant should be laid down to sleep on his or her back for better breathing and to prevent sudden infant death syndrome (SIDS). Grandmothers may encourage the new parents to place the infant on the abdomen; however, evidence shows back to sleep reduces SIDS. Infants are vulnerable to respiratory infections; therefore, infected people must be kept away. Secondhand smoke can damage lungs. Infants can suffocate in loose bedding and in furniture that can trap them. Per AAP guidelines, infants should always be placed back to sleep and allowed tummy time to play to prevent plagiocephaly.)

The most common cause of decreased variability in the fetal heart rate (FHR) that lasts 30 minutes or less is: a. Altered cerebral blood flow. c. Umbilical cord compression. b. Fetal hypoxemia. d. Fetal sleep cycles.

D A temporary decrease in variability can occur when the fetus is in a sleep state. These sleep states do not usually last longer than 30 minutes. Altered fetal cerebral blood flow would result in early decelerations in the FHR. Fetal hypoxemia would be evidenced by tachycardia initially and then bradycardia. A persistent decrease or loss of FHR variability may be seen. Umbilical cord compression would result in variable decelerations in the FHR.

When assessing the relative advantages and disadvantages of internal and external electronic fetal monitoring, nurses comprehend that both: a. Can be used when membranes are intact. b. Measure the frequency, duration, and intensity of uterine contractions. c. May need to rely on the woman to indicate when uterine activity (UA) is occurring. d. Can be used during the antepartum and intrapartum periods.

D External monitoring can be used in both periods; internal monitoring can be used only in the intrapartum period. For internal monitoring the membranes must have ruptured, and the cervix must be sufficiently dilated. Internal monitoring measures the intensity of contractions; external monitoring cannot do this. With external monitoring, the woman may need to alert the nurse that UA is occurring; internal monitoring does not require this.

Increasing the infusion rate of nonadditive intravenous fluids can increase fetal oxygenation primarily by: a. Maintaining normal maternal temperature. b. Preventing normal maternal hypoglycemia. c. Increasing the oxygen-carrying capacity of the maternal blood. d. Expanding maternal blood volume.

D Filling the mother's vascular system makes more blood available to perfuse the placenta and may correct hypotension. Increasing fluid volume may alter the maternal temperature only if she is dehydrated. Most intravenous fluids for laboring women are isotonic and do not provide extra glucose. Oxygen-carrying capacity is increased by adding more red blood cells.

Which fetal heart rate (FHR) finding would concern the nurse during labor? a. Accelerations with fetal movement c. An average FHR of 126 beats/min b. Early decelerations d. Late decelerations

D Late decelerations are caused by uteroplacental insufficiency and are associated with fetal hypoxemia. They are considered ominous if persistent and uncorrected. Accelerations in the FHR are an indication of fetal well-being. Early decelerations in the FHR are associated with head compression as the fetus descends into the maternal pelvic outlet; they generally are not a concern during normal labor.

When using intermittent auscultation (IA) for fetal heart rate, nurses should be aware that: a. They can be expected to cover only two or three clients when IA is the primary method of fetal assessment. b. The best course is to use the descriptive terms associated with electronic fetal monitoring (EFM) when documenting results. c. If the heartbeat cannot be found immediately, a shift must be made to EFM. d. Ultrasound can be used to find the fetal heartbeat and reassure the mother if initial difficulty was a factor.

D Locating fetal heartbeats often takes time. Mothers can be reassured verbally and by the ultrasound pictures if ultrasound is used to help locate the heartbeat. When used as the primary method of fetal assessment, auscultation requires a nurse-to-client ratio of one to one. Documentation should use only terms that can be numerically defined; the usual visual descriptions of EFM are inappropriate.

As a perinatal nurse you realize that a fetal heart rate that is tachycardic, is bradycardic, or has late decelerations or loss of variability is nonreassuring and is associated with: a. Hypotension. c. Maternal drug use. b. Cord compression. d. Hypoxemia.

D Nonreassuring heart rate patterns are associated with fetal hypoxemia. Fetal bradycardia may be associated with maternal hypotension. Fetal variable decelerations are associated with cord compression. Maternal drug use is associated with fetal tachycardia.

While evaluating an external monitor tracing of a woman in active labor whose labor is being induced, the nurse notes that the fetal heart rate (FHR) begins to decelerate at the onset of several contractions and returns to baseline before each contraction ends. The nurse should: a. Change the woman's position. b. Discontinue the oxytocin infusion. c. Insert an internal monitor. d. Document the finding in the client's record.

D The FHR indicates early decelerations, which are not an ominous sign and do not require any intervention. The nurse should simply document these findings.

A nurse may be called on to stimulate the fetal scalp: a. As part of fetal scalp blood sampling. b. In response to tocolysis. c. In preparation for fetal oxygen saturation monitoring. d. To elicit an acceleration in the fetal heart rate (FHR).

D The scalp can be stimulated using digital pressure during a vaginal examination. Fetal scalp blood sampling involves swabbing the scalp with disinfectant before a sample is collected. The nurse would stimulate the fetal scalp to elicit an acceleration of the FHR. Tocolysis is relaxation of the uterus. Fetal oxygen saturation monitoring involves the insertion of a sensor.

You are evaluating the fetal monitor tracing of your client, who is in active labor. Suddenly you see the fetal heart rate (FHR) drop from its baseline of 125 down to 80. You reposition the mother, provide oxygen, increase intravenous (IV) fluid, and perform a vaginal examination. The cervix has not changed. Five minutes have passed, and the fetal heart rate remains in the 80s. What additional nursing measures should you take? a. Scream for help. b. Insert a Foley catheter. c. Start Pitocin. d. Notify the care provider immediately.

D To relieve an FHR deceleration, the nurse can reposition the mother, increase IV fluid, and provide oxygen. If oxytocin is infusing, it should be discontinued. If the FHR does not resolve, the primary care provider should be notified immediately. Inserting a Foley catheter is an inappropriate nursing action. If the FHR were to continue in a nonreassuring pattern, a cesarean section could be warranted, which would require a Foley catheter. However, the physician must make that determination. Pitocin may place additional stress on the fetus.

3. A postpartum woman telephones the provider regarding her 5-day-old infant. The client is not scheduled for another weight check until the infant is 14 days old. The new mother is worried about whether breastfeeding is going well. Which statement indicates that breastfeeding is effective for meeting the infants nutritional needs? a.Sleeps for 6 hours at a time between feedings b.Has at least one breast milk stool every 24 hours c.Gains 1 to 2 ounces per week d.Has at least six to eight wet diapers per day

D (After day 4, when the mothers milk comes in, the infant should have six to eight wet diapers every 24 hours. Typically, infants sleep 2 to 4 hours between feedings, depending on whether they are being fed on a 2- to 3-hour schedule or cluster-fed. The infants sleep pattern is not an indication whether the infant is breastfeeding well. The infant should have a minimum of three bowel movements in a 24-hour period. Breastfed infants typically gain 15 to 30 g/day.)

18. The nurse is providing instruction to the newly delivered client regarding postbirth uterine and vaginal discharge, called lochia. Which statement is the most appropriate? a. Lochia is similar to a light menstrual period for the first 6 to 12 hours. b. It is usually greater after cesarean births. c. Lochia will usually decrease with ambulation and breastfeeding. d. It should smell like normal menstrual flow unless an infection is present.

D (An offensive odor usually indicates an infection. Lochia flow should approximate a heavy menstrual period for the first 2 hours and then steadily decrease. Less lochia is usually seen after cesarean births and usually increases with ambulation and breastfeeding.)

9. A client is concerned that her breasts are engorged and uncomfortable. What is the nurses explanation for this physiologic change? a. Overproduction of colostrum b. Accumulation of milk in the lactiferous ducts and glands c. Hyperplasia of mammary tissue d. Congestion of veins and lymphatic vessels

D (Breast engorgement is caused by the temporary congestion of veins and lymphatic vessels. An overproduction of colostrum, an accumulation of milk in the lactiferous ducts and glands, and hyperplasia of mammary tissue do not cause breast engorgement.)

14. Which condition, not uncommon in pregnancy, is likely to require careful medical assessment during the puerperium? a. Varicosities of the legs b. Carpal tunnel syndrome c. Periodic numbness and tingling of the fingers d. Headaches

D (Headaches in the postpartum period can have a number of causes, some of which deserve medical attention. Total or nearly total regression of varicosities is expected after childbirth. Carpal tunnel syndrome is relieved in childbirth when the compression on the median nerve is lessened. Periodic numbness of the fingers usually disappears after childbirth unless carrying the baby aggravates the condition.)

4. A woman gave birth to a healthy infant boy 5 days ago. What type of lochia does the nurse expect to find when evaluating this client? a. Lochia rubra b. Lochia sangra c. Lochia alba d. Lochia serosa

D (Lochia serosa, which consists of blood, serum, leukocytes, and tissue debris, generally occurs around day 3 or 4 after childbirth. Lochia rubra consists of blood and decidual and trophoblastic debris. The flow generally lasts 3 to 4 days and pales, becoming pink or brown. Lochia sangra is not a real term. Lochia alba occurs in most women after day 10 and can continue up to 6 weeks after childbirth.)

2. What are the most common causes for subinvolution of the uterus? a. Postpartum hemorrhage and infection b. Multiple gestation and postpartum hemorrhage c. Uterine tetany and overproduction of oxytocin d. Retained placental fragments and infection

D (Subinvolution is the failure of the uterus to return to a nonpregnant state. The most common causes of subinvolution are retained placental fragments and infection. Subinvolution may be caused by an infection and result in hemorrhage. Multiple gestations may cause uterine atony, resulting in postpartum hemorrhaging. Uterine tetany and overproduction of oxytocin do not cause subinvolution.)

22. Nurses should be able to teach breastfeeding mothers the signs that the infant has correctly latched on. Which client statement indicates a poor latch? a.I feel a firm tugging sensation on my nipples but not pinching or pain. b.My baby sucks with cheeks rounded, not dimpled. c.My babys jaw glides smoothly with sucking. d.I hear a clicking or smacking sound.

D (The clicking or smacking sound may indicate that the baby is having difficulty keeping the tongue out over the lower gum ridge. The mother should hope to hear the sound of swallowing. The tugging sensation without pinching is a good sign. Rounded cheeks are a positive indicator of a good latch. A smoothly gliding jaw also is a good sign.)

12. A client asks the nurse when her ovaries will begin working again. Which explanation by the nurse is most accurate? a. Almost 75% of women who do not breastfeed resume menstruating within 1 month after birth. b. Ovulation occurs slightly earlier for breastfeeding women. c. Because of menstruation and ovulation schedules, contraception considerations can be postponed until after the puerperium. d. The first menstrual flow after childbirth usually is heavier than normal.

D (The first flow is heavier, but within three or four cycles, the flow is back to normal. Ovulation can occur within the first month, but for 70% of nonlactating women, it returns in approximately 3 months. Women who are breastfeeding take longer to resume ovulation. Because many women ovulate before their first postpartum menstrual period, contraceptive options need to be discussed early in the puerperium.)

4. A primiparous woman is delighted with her newborn son and wants to begin breastfeeding as soon as possible. How should the client be instructed to position the infant to facilitate correct latch-on? a.The infant should be positioned with his or her arms folded together over the chest. b.The infant should be curled up in a fetal position. c.The woman should cup the infants head in her hand. d.The infants head and body should be in alignment with the mother.

D (The infants head and body should be in correct alignment with the mother and the breast during latch-on and feeding. The infant should be facing the mother with his arms hugging the breast. The babys body should be held in correct alignment (i.e., ears, shoulder, and hips in a straight line) during feedings. The mother should support the babys neck and shoulders with her hand and not push on the occiput.)

16. A newly delivered mother who intends to breastfeed tells her nurse, I am so relieved that this pregnancy is over so that I can start smoking again. The nurse encourages the client to refrain from smoking. However, this new mother is insistent that she will resume smoking. How will the nurse adapt her health teaching with this new information? a.Smoking has little-to-no effect on milk production. b.No relationship exists between smoking and the time of feedings. c.The effects of secondhand smoke on infants are less significant than for adults. d.The mother should always smoke in another room.

D (The new mother should be encouraged not to smoke. If she continues to smoke, she should be encouraged to always smoke in another room, removed from the baby. Smoking may impair milk production. When the products of tobacco are broken down, they cross over into the breast milk. Tobacco also results in a reduction of the antiinfective properties of breast milk. Research supports the conclusion that mothers should not smoke within 2 hours before a feeding (AAP Committee on Drugs, 2001). The effects of secondhand smoke on infants include excessive crying, colic, upper respiratory infections, and an increased risk of sudden infant death syndrome (SIDS).)

7. Which term best describes the interval between the birth of the newborn and the return of the reproductive organs to their normal nonpregnant state? a. Involutionary period because of what happens to the uterus b. Lochia period because of the nature of the vaginal discharge c. Mini-tri period because it lasts only 3 to 6 weeks d. Puerperium, or fourth trimester of pregnancy

D (The puerperium, also called the fourth trimester or the postpartum period of pregnancy, is the final period of pregnancy and lasts approximately 3 to 6 weeks. Involution marks the end of the puerperium. Lochia refers to the various vaginal discharges during the puerperium.)

20. A nurse is responsible for teaching new parents regarding the hygienic care of their newborn. Which instruction should the nurse provide regarding bathing? a. Avoid washing the head for at least 1 week to prevent heat loss. b. Sponge bathe the newborn for the first month of life. c. Cleanse the ears and nose with cotton-tipped swabs, such as Q-tips. d. Create a draft-free environment of at least 24 C (75 F) when bathing the infant.

D (The temperature of the room should be 24 C (75 F), and the bathing area should be free of drafts. To prevent heat loss, the infants head should be bathed before unwrapping and undressing. Tub baths may be initiated from birth. Ensure that the infant is fully immersed. Q-tips should not be used; they may cause injury. A corner of a moistened washcloth should be twisted into shape so that it can be used to cleanse the ears and nose.)

16. Pelvic floor exercises, also known as Kegel exercises, will help to strengthen the perineal muscles and encourage healing after childbirth. The nurse requests the client to repeat back instructions for this exercise. Which response by the client indicates successful learning? a. I contract my thighs, buttocks, and abdomen. b. I perform 10 of these exercises every day. c. I stand while practicing this new exercise routine. d. I pretend that I am trying to stop the flow of urine in midstream.

D (The woman can pretend that she is attempting to stop the passing of gas or the flow of urine midstream, which will replicate the sensation of the muscles drawing upward and inward. Each contraction should be as intense as possible without contracting the abdomen, buttocks, or thighs. Guidelines suggest that these exercises should be performed 24 to 100 times per day. Positive results are shown with a minimum of 24 to 45 repetitions per day. The best position to learn Kegel exercises is to lie supine with the knees bent. A secondary position is on the hands and knees.)

While evaluating an external monitor tracing of a woman in active labor whose labor is being induced, the nurse notes that the fetal heart rate (FHR) begins to decelerate at the onset of several contractions and returns to baseline before each contraction ends. The nurse should:

Document the finding in the client's record [The FHR indicates early decelerations, which are not an ominous sign and do not require any intervention]

The uterine contractions of a woman early in the active phase of labor are assessed by an internal uterine pressure catheter (IUPC). The nurse notes that the intrauterine pressure at the peak of the contraction ranges from 65 to 70 mm Hg and the resting tone range is 6 to 10 mm Hg. The uterine contractions occur every 3 to 4 minutes and last an average of 55 to 60 seconds. On the basis of this information, the nurse should:

Document the findings because they reflect the expected contraction pattern for the active phase of labor.

Remaining fairly stable throughout the first and second stages

Duration

Allison is 32 weeks pregnant presents to L&D and tells the nurse she has not felt her baby move in 48 hours. The nurse begins electronic fetal monitoring and notes the baby's heart rate is in the 130's. The nurse contacts Allison's physician to obtain an order for which of the following tests? a. Fetal fibronectin b. Maternal Serum Screening Test c. Biophysical profile d. Leopolds Maneuvers e. Non stress Test

E

In assisting with the two factors that have an effect on fetal status pushing and positioning , nurses should:

Encourage the woman's cooperation in avoiding the supine position.

Increasing the infusion rate of nonadditive intravenous fluids can increase fetal oxygenation primarily by:

Expanding maternal blood volume. [Filling the mother's vascular system makes more blood available to perfuse the placenta and may correct hypotension]

Generally ranging from two to five contractions per 10 minutes of labor

Frequency

As a perinatal nurse you realize that a fetal heart rate that is tachycardic, is bradycardic, or has late decelerations or loss of variability is nonreassuring and is associated with:

Hypoxemia. [Nonreassuring heart rate patterns are associated with fetal hypoxemia]

PPH may be sudden and result in rapid blood loss. The nurse must be alert to the symptoms of hemorrhage and hypovolemic shock and be prepared to act quickly to minimize blood loss. Astute assessment of the client's circulatory status can be performed with noninvasive monitoring. Match the type of noninvasive assessment that the nurse would perform with the appropriate clinical manifestation or body system. Inspection Skin color, temp, tugor Palpation Arterial pulses Observation Presence or absence of anxiety Measurement Pulse oximetry Auscultation Heart pulses

Inspection Skin color, temp, tugor Palpation Arterial pulses Observation Presence or absence of anxiety Measurement Pulse oximetry Auscultation Heart pulses

Which fetal heart rate (FHR) finding would concern the nurse during labor?

Late decelerations [Late decelerations are caused by uteroplacental insufficiency and are associated with fetal hypoxemia. They are considered ominous if persistent and uncorrected.]

The baseline fetal heart rate (FHR) is the average rate during a 10-minute segment. Changes in FHR are categorized as periodic or episodic. These patterns include both accelerations and decelerations. The labor nurse is evaluating the patient's most recent 10-minute segment on the monitor strip and notes a late deceleration. It is likely to be caused by which physiologic alteration?

Late decelerations are almost always caused by uteroplacental insufficiency. [Insufficiency is caused by uterine tachysystole, maternal hypotension, epidural or spinal anesthesia, IUGR, intraamniotic infection, or placental abruption.]

What correctly matches the type of deceleration with its likely cause?

Late deceleration—uteroplacental inefficiency

____________________, a synthetic opiate, has been the therapy of choice for heroin addiction. It crosses the placenta, leading to significant neonatal abstinence syndrome after birth.

Methadone p. 1015

The nurse knows that proper placement of the tocotransducer for electronic fetal monitoring is located:

Over the uterine fundus [The tocotransducer monitors uterine activity and should be placed over the fundus, where the most intensive uterine contractions occur.]

Fetal bradycardia is most common during:

Prolonged umbilical cord compression. [Fetal bradycardia can be considered a later sign of fetal hypoxia and is known to occur before fetal death. Bradycardia can result from placental transfer of drugs, prolonged compression of the umbilical cord, maternal hypothermia, and maternal hypotension.]

Commonly 45 seconds or more in the second stage of labor

Relaxation time

What three measures should the nurse implement to provide intrauterine resuscitation? Select the response that best indicates the priority of actions that should be taken.

Reposition the mother, increase intravenous (IV) fluid, and provide oxygen via face mask.

Average of 10 mm Hg

Resting tone

When using intermittent auscultation (IA) for fetal heart rate, nurses should be aware that:

Ultrasound can be used to find the fetal heartbeat and reassure the mother if initial difficulty was a factor.

The nurse providing care for the laboring woman realizes that variable fetal heart rate decelerations are caused by:

Umbilical cord compression. [Variable decelerations can occur any time during the uterine contracting phase and are caused by compression of the umbilical cord.]

Which maternal condition is considered a contraindication for the application of internal monitoring devices?

Unruptured membranes [In order to apply internal monitoring devices, the membranes must be ruptured.]

The nurse caring for a woman in labor understands that prolonged decelerations:

Usually are isolated events that end spontaneously.

Why is continuous electronic fetal monitoring usually used when oxytocin is administered?

Uteroplacental exchange may be compromised. [The uterus may contract more firmly, and the resting tone may be increased with oxytocin use]

The nurse caring for the woman in labor should understand that maternal hypotension can result in:

Uteroplacental insufficiency. [Low maternal blood pressure reduces placental blood flow during uterine contractions and results in fetal hypoxemia.]

The nurse providing care for the laboring woman should understand that late fetal heart rate decelerations are the result of:

Uteroplacental insufficiency. [Uteroplacental insufficiency would result in late decelerations in the FHR.]

The nurse providing care for the laboring woman should understand that amnioinfusion is used to treat:

Variable decelerations [Amnioinfusion is used during labor either to dilute meconium-stained amniotic fluid or to supplement the amount of amniotic fluid to reduce the severity of variable decelerations caused by cord compression]

The nurse caring for a laboring woman is aware that maternal cardiac output can be increased by: a) Change in position b) Oxytocin administration c) Regional anesthesia d) Intravenous analgesic

a) Change in position

When using intermittent auscultation to assess uterine activity, nurses should be aware that: a) The examiner's hand should be placed over the fundus before, during, and after contractions b) The frequency and duration of contractions are measured in seconds for consistency c) Contraction intensity is given a judgment number of 1 to 7 by the nurse and client together d) The resting tone between contractions is described as either placid or turbulent

a) The examiner's hand should be placed over the fundus before, during, and after contractions

Fetal well-being during labor is assessed by: a) The response of the fetal heart rate (FHR) to uterine contractions (UCs) b) Maternal pain control c) Accelerations in the FHR d) An FHR greater than 110 beats/min

a) The response of the fetal heart rate (FHR) to uterine contractions (UCs)

A group of fetal monitoring experts (National Institute of Child Health and Human Development, 2008) recommends that fetal heart rate (FHR) tracings demonstrate certain characteristics to be described as reassuring or normal (category I). This includes: a) Bradycardia not accompanied by baseline variability b) Early decelerations, either present or absent c) Sinusoidal pattern d) Tachycardia

b) Early decelerations, either present or absent

In documenting labor experiences, nurses should know that a uterine contraction is described according to all of these characteristics except: a) Frequency (how often contractions occur) b) Intensity (the strength of the contraction at its peak) c) Resting tone (The tension in the uterine muscle) d) Appearance (shape and height)

d) Appearance (shape and height)

The most common cause of decreased variability in the FHR that lasts 30 minutes or less is: a) Altered cerebral blood flow b) Fetal hypoxemia c) Umbilical cord compression d) Fetal sleep cycles

d) Fetal sleep cycles

You are evaluating the fetal monitor tracing of your client, who is in active labor. Suddenly you see the fetal heart rate (FHR) drop from its baseline of 125 down to 80. You reposition the mother, provide oxygen, increase IV fluid, and perform a vaginal exam. The cervix has not changed. Five minutes have passed, and the FHR remains in the 80s. What additional nursing measures should you take? a) Call for help b) Insert a foley catheter c) Start oxytocin (Pitocin) d) Notify the primary health care provider immediately

d) Notify the primary health care provider immediately

External fetal monitoring cannot detect the ____________________ of uterine contractions.

intensity p. 500


Set pelajaran terkait

AP Psych: Personality - Psychoanalytic and Humanistic

View Set

RN Pediatric Nursing Online Practice 2023 A

View Set

Determinants of Short-run Aggregate Supply (SRAS)

View Set